SlideShare a Scribd company logo
1 of 93
ĐẠI HỌC QUỐC GIA HÀ NỘI
TRƯỜNG ĐẠI HỌC KHOA HỌC TỰ NHIÊN
- - - - - - - - - - - - - - - - - - - - - - -
ĐINH THỊ BÍCH NGỌC
ĐỀ TÀI
MỘT SỐ PHƯƠNG PHÁP
GIẢI BÀI TOÁN KHÔNG MẪU MỰC
Chuyên ngành: Phương pháp toán sơ cấp
Mã số: 60.46.01.13
LUẬN VĂN THẠC SĨ KHOA HỌC
Người hướng dẫn khoa học: GS.TS Đặng Huy Ruận
HÀ NỘI - 2015
Mục lục
Lời nói đầu 3
1 Phương pháp quy nạp toán học 4
1.1 Nguyên lý quy nạp . . . . . . . . . . . . . . . . . . . . . 4
1.2 Phương pháp chứng minh bằng quy nạp . . . . . . . . . 4
1.2.1 Cơ sở quy nạp . . . . . . . . . . . . . . . . . . . . 4
1.2.2 Quy nạp . . . . . . . . . . . . . . . . . . . . . . . 5
1.3 Vận dụng phương pháp quy nạp để giải bài toán không
mẫu mực . . . . . . . . . . . . . . . . . . . . . . . . . . . 6
1.4 Bài tập tự giải . . . . . . . . . . . . . . . . . . . . . . . . 23
2 Phương pháp phản chứng 25
2.1 Phép suy luận phản chứng . . . . . . . . . . . . . . . . . 25
2.2 Phương pháp chứng minh bằng phản chứng . . . . . . . 25
2.3 Các bước suy luận trong chứng minh phản chứng . . . . 26
2.4 Vận dụng phương pháp phản chứng để giải các bài toán
không mẫu mực . . . . . . . . . . . . . . . . . . . . . . . 27
2.5 Bài tập tự giải . . . . . . . . . . . . . . . . . . . . . . . . 37
3 Phương pháp suy luận 39
3.1 Vài nét về phương pháp suy luận . . . . . . . . . . . . . 39
3.2 Các ví dụ về vận dụng phương pháp suy luận. . . . . . . 40
3.3 Bài tập tự giải . . . . . . . . . . . . . . . . . . . . . . . . 46
4 Phương pháp bảng 50
4.1 Vài nét về phương pháp bảng . . . . . . . . . . . . . . . 50
4.2 Vận dụng phương pháp bảng để giải bài toán không mẫu
mực . . . . . . . . . . . . . . . . . . . . . . . . . . . . . 50
4.3 Bài tập tự giải . . . . . . . . . . . . . . . . . . . . . . . . 59
1
5 Phương pháp sơ đồ 63
5.1 Giới thiệu về phương pháp sơ đồ . . . . . . . . . . . . . 63
5.2 Vận dụng phương pháp sơ đồ để giải các bài toán không
mẫu mực. . . . . . . . . . . . . . . . . . . . . . . . . . . 63
5.3 Bài tập tự giải . . . . . . . . . . . . . . . . . . . . . . . . 69
6 Phương pháp đồ thị 73
6.1 Một số khái niệm và kết quả cơ bản của lý thuyết đồ thị 73
6.2 Phương pháp đồ thị . . . . . . . . . . . . . . . . . . . . . 74
6.3 Vận dụng phương pháp đồ thị để giải bài toán không mẫu
mực . . . . . . . . . . . . . . . . . . . . . . . . . . . . . 75
6.4 Bài tập tự giải . . . . . . . . . . . . . . . . . . . . . . . . 87
Kết luận 91
Tài liệu tham khảo 92
2
LỜI NÓI ĐẦU
Các bài toán không mẫu mực là các bài toán mà việc giải chúng đòi
hỏi suy luận, tư duy độc đáo. Việc giải các bài toán không mẫu mực
giúp người thực hiện nâng cao nhanh chóng khả năng tư duy, suy luận
và nhiều khi phát hiện ra những phương pháp giải toán độc đáo không
ngờ. Bởi vậy rất nhiều em học sinh, đặc biệt là học sinh trường chuyên,
lớp chọn thích làm quen với các bài toán này.
Luận văn "Một số phương pháp giải bài toán không mẫu mực" trình
bày sáu phương pháp chủ yếu để giải các bài toán không mẫu mực.
Nhưng do một bài toán không mẫu mực có thể giải đồng thời bằng
nhiều phương pháp khác nhau và một vài phương pháp có phần "tương
tự" nên việc phân loại phương pháp, ví dụ và bài tập chỉ là tương đối.
Các bài toán không mẫu mực là mảng khá lý thú trong toán học nói
chung cũng như toán phổ thông nói riêng. Vì vậy, tác giả hi vọng luận
văn sẽ trở thành tài liệu có ích cho các em học sinh phổ thông, đặc biệt
các em học sinh trường chuyên, lớp chọn, các thầy cô giáo dạy ở cuối
cấp tiểu học, các thầy cô giáo dạy toán ở trường phổ thông, các bạn sinh
viên và những ai quan tâm đến mảng toán lý thú này.
Luận văn được chia làm sáu chương:
Chương 1 trình bày về phương pháp quy nạp toán học.
Chương 2 trình bày về phương pháp phản chứng.
Chương 3 trình bày về phương pháp suy luận.
Chương 4 trình bày về phương pháp bảng.
Chương 5 trình bày về phương pháp sơ đồ.
Chương 6 trình bày về phương pháp đồ thị.
Luận văn được hoàn thành dưới sự hướng dẫn, giúp đỡ tận tình của
GS.TS Đặng Huy Ruận, em xin gửi tới thầy lòng biết ơn sâu sắc. Em
xin gửi lời cảm ơn chân thành đến Ban chủ nhiệm khoa cùng các thầy
cô giáo khoa Toán - Cơ - Tin học, Trường Đại học Khoa Học Tự Nhiên
- Đại Học Quốc Gia Hà Nội đã tạo điều kiện, dạy bảo và dìu dắt em
trong những năm học vừa qua. Xin chân thành cảm ơn sự giúp đỡ của
bạn bè, người thân trong thời gian học tập và làm luận văn.
Do khả năng nhận thức của bản thân tác giả, luận văn còn nhiều hạn
chế, thiếu sót. Kính mong nhận được các ý kiến đóng góp của thầy cô
cùng các bạn đọc.
Xin chân thành cảm ơn!
Hà Nội, tháng 7 năm 2015
3
Chương 1
Phương pháp quy nạp toán học
Phương pháp quy nạp toán học là một công cụ rất có hiệu lực trong
việc chứng minh nhiều bài toán thuộc các lĩnh vực khác nhau của toán
học như: số học, đại số, hình học... và đặc biệt là các bài toán không
mẫu mực. Đây là một phương pháp chứng minh toán học đặc biệt cho
phép ta rút ra những quy luật tổng quát dựa trên cơ sở những trường
hợp riêng. Quá trình quy nạp ngược với quá trình suy diễn. Từ "tính
chất" của một số cá thể suy ra "tính chất" của tập thể, nên không phải
lúc nào cũng đúng. Nó chỉ đúng khi thỏa mãn nguyên lý quy nạp.
1.1 Nguyên lý quy nạp
Cho n0 là một số nguyên dương và P(n) là một mệnh đề có nghĩa với
mọi số tự nhiên n ≥ n0. Nếu
1.P(n0) đúng và
2. Nếu P(k) đúng từ đó suy ra được P(k + 1) cũng đúng với mọi số
tự nhiên k ≥ n0 thì P(n) đúng với mọi số tự nhiên n ≥ n0.
1.2 Phương pháp chứng minh bằng quy nạp
Giả sử khẳng định P(n) xác định ∀n ≥ n0. Để chứng minh P(n) đúng
∀n ≥ n0 bằng quy nạp, ta cần thực hiện 2 bước
1.2.1 Cơ sở quy nạp
Kiểm tra sự đúng đắn của P(n) với n = n0, nghĩa là xét P(n0) có
đúng không.
4
1.2.2 Quy nạp
Chứng minh rằng: Nếu với mỗi k ≥ n0, P(k) là mệnh đề đúng, thì
suy ra P(k + 1) cũng đúng.
Nếu cả 2 bước trên đều thỏa mãn, thì theo nguyên lý quy nạp P(n) đúng
với mọi n ≥ n0.
Chú ý
Trong quá trình quy nạp, nếu không thực hiện đầy đủ cả 2 bước: Cơ sở
quy nạp và quy nạp thì có thể dẫn đến kết luận sai lầm. Một số ví dụ
sau sẽ chứng tỏ điều này.
- Do bỏ qua bước cơ sở quy nạp, ta đưa ra kết luận không đúng: Mọi số
tự nhiên đều bằng nhau! Bằng cách quy nạp như sau: Giả sử các số tự
nhiên không vượt quá k + 1 đã bằng nhau. Khi đó ta có:
k = k + 1
Thêm vào mỗi vế của đẳng thức trên 1 đơn vị sẽ có:
k + 1 = k + 1 + 1 = k + 2
Cứ như vậy suy ra mọi số tự nhiên không nhỏ hơn k đều bằng nhau. Kết
hợp với giả thiết quy nạp: Mọi số tự nhiên không vượt quá k đều bằng
nhau, đi đến kết luận sai lầm: Tất cả các số tự nhiên đều bằng nhau!
- Do bỏ qua bước quy nạp nên nhà toán học Pháp P.Fermat (1601 -
1665) đã cho rằng số dạng 22n
+ 1 đều là số nguyên tố.
P.Fermat xét 5 số đầu tiên:
Với n = 0 cho 220
+ 1 = 21
+ 1 = 3 là số nguyên tố.
Với n = 1 cho 221
+ 1 = 22
+ 1 = 5 là số nguyên tố.
Với n = 2 cho 222
+ 1 = 24
+ 1 = 17 là số nguyên tố.
Với n = 3 cho 223
+ 1 = 28
+ 1 = 257 là số nguyên tố.
Với n = 4 cho 224
+ 1 = 216
+ 1 = 65537 là số nguyên tố.
Nhưng vào thế kỷ XVIII, L.Euler (1707 - 1783) đã phát hiện với n = 5
khẳng định trên không đúng, bởi vì:
225
+ 1 = 4294967297 = 641.6700417
là hợp số.
5
1.3 Vận dụng phương pháp quy nạp để giải bài toán
không mẫu mực
Phương pháp quy nạp được sử dụng trong tính toán, trong chứng
minh và suy luận dưới nhiều dạng khác nhau, nhưng trong phần này chỉ
trình bày việc vận dụng phương pháp quy nạp để giải bài toán không
mẫu mực.
Bài toán 1.3.1. (IMO 1998) Với mọi số nguyên dương n, ta kí hiệu
d(n) là số tất cả các ước dương của n (kể cả 1 và n). Hãy xác định tất
cả các số nguyên dương k, sao cho d(n2
) = kd(n), với n là số nguyên
dương nào đó.
Chứng minh.
Giả sử khi phân tích ra thừa số nguyên tố, số n có dạng:
n = pa1
1 .pa2
2 ...par
r
Ta có:
d(n) = (a1 + 1)(a2 + 1)...(ar + 1)
d(n2
) = (2a1 + 1)(2a2 + 1)...(2ar + 1)
Để d(n2
) = kd(n) thì ta phải chọn các số ai sao cho:
(2a1 + 1)(2a2 + 1)...(2ar + 1) = k(a1 + 1)(a2 + 1)...(ar + 1) (∗)
Do (2ai + 1)(1 ≤ i ≤ r) đều là các số lẻ nên k phải là các số lẻ. Ta sẽ
chứng minh mệnh đề đảo lại rằng: "Với số lẻ k bất kỳ, ta có thể tìm
được các số ai thỏa mãn (*) (tức là tìm được n)".
Dùng phương pháp quy nạp theo k.
1. Với k = 1, mệnh đề đúng (n = 1, ai = 0)
2. Giả sử mệnh đề đúng với số k nào đó, ta chứng minh nó cũng đúng
cho (2m
.k − 1)(m ≥ 1). Lúc đó mệnh đề đúng cho mọi số lẻ vì mọi số lẻ
l đều viết được dưới dạng: (2m
.l − 1) (với l là số nhỏ hơn l).
Đặt ai = 2i−1
[(2m
− 1).k − 1], với i = 1, 2, ..., m.
Khi đó: 2ai + 1 = 2i
(2m
− 1)k − (2i
− 1)
ai + 1 = 2i−1
(2m
− 1)k − (2i−1
− 1) = 2ai−1 + 1
Do vậy, tích của các số (2ai + 1) chia hết cho tích các số (ai + 1) với
i = 1, m khi (2am + 1) chia hết cho (a1 + 1) hay:
[2m
(2m
− 1)k − (2m
− 1)] = (2m
− 1).(2m
.k − 1)
6
chia hết cho (2m
− 1)k có nghĩa là: (2m
.k − 1) chia hết cho k. Vậy nếu ta
chọn các ai như trên với k đã cho, thì mệnh đề trên đúng cho (2m
.k −1).
Ta có điều phải chứng minh!
Bài toán 1.3.2. (USAMTS, 2000 - 2001, Cuộc thi tài năng toán học
Mỹ) Hãy tìm số dư khi chia số 17761492!
cho 2000.
Chứng minh.
Trước hết ta chứng minh bổ đề: "Với mọi số nguyên dương n, ta có:
1376n
≡ 1376(mod 2000)"
Dùng phương pháp quy nạp:
1. Với n = 1, hiển nhiên có: 13761
≡ 1376(mod 2000)
Với n = 2, ta có: 13762
= 1893376 ≡ 1376(mod 2000)
2. Giả sử mệnh đề đúng với n = k(k ∈ N, k ≥ 1),
tức là: 1376k
≡ 1376(mod 2000).
Ta chứng minh bổ đề đúng với n = k +1. Thật vậy, từ giả thiết quy nạp
ta có:
1376k+1
≡ 13762
(mod 2000), mà 13762
≡ 1376(mod 2000), nên
1376k+1
≡ 1376(mod 2000)
Bổ đề được chứng minh!
Quay lại bài toán, ta có: 17765
= 1376(mod 2000), nên
17761492!
= (17765
)
1492!
5
Vậy khi chia số 17761492!
cho 2000 được số dư là 1376.
Bài toán 1.3.3. Hãy tìm chữ số tận cùng của số: An = 22n
+ 1 với mọi
số nguyên n, n ≥ 2.
Chứng minh.
1. Với n = 2, số A2 = 222
+ 1 = 17, có chữ số tận cùng là 7.
2. Giả sử với n = k, số Ak = 22k
+ 1 có chữ số tận cùng là 7. Ta sẽ
chứng minh Ak+1 có chữ số tận cùng là 7.
Thật vậy, do Ak có chữ số tận cùng là 7, nên tồn tại số nguyên dương
m để: Ak = 10m + 7, hay:
22k
+ 1 = 10m + 7
7
Tức là: 22k
= 10m + 6. Từ đó:
Ak+1 = 22k+1
+ 1
= 22k
.2
+ 1
= (22k
)
2
+ 1
= (10m + 6)2
+ 1
= 100m2
+ 120m + 37
= 10(10m2
+ 12m + 3) + 7
nên Ak+1 cũng có chữ số tận cùng là 7.
Vậy với mọi số nguyên n, n ≥ 2, thì An = 22n
+ 1 có chữ số tận cùng
là 7.
Bài toán 1.3.4. (Vô địch toán Canada, 1982) Cho a, b và c là những
nghiệm của phương trình:
x3
− x2
− x − 1 = 0
Chứng minh rằng số:
b1982
− c1982
b − c
+
c1982
− a1982
c − a
+
a1982
− b1982
a − b
là một số nguyên.
Chứng minh.
Đặt un = bn
−cn
b−c , vn = cn
−an
c−a , wn = an
−bn
a−b , với n nguyên, n ≥ 1.
Ta sẽ chứng minh: un + vn + wn nguyên với mọi n nguyên, n ≥ 1(∗).
Trước hết ta thấy rằng: un+3 = un+3 + un+3 + un, ∀n, n = 1.
Thật vậy, do b, c là nghiệm của phương trình x3
− x2
− x − 1 = 0 nên:
b3
= b2
+ b + 1, c3
= c3
+ c + 1
Do đó:
un+3 =
bn+3
− cn+3
b − c
=
bn
.b3
− cn
.c3
b − c
=
bn
(b2
+ b + 1) − cn
(c2
+ c + 1)
b − c
=
bn+2
− cn+2
b − c
+
bn+1
− cn+1
b − c
+
bn
− cn
b − c
= un+2 + un+1 + un
8
Tương tự ta có:
vn+3 = vn+2 + vn+1 + vn
wn+3 = wn+2 + wn+1 + wn
Tiếp theo, ta sẽ dùng phương pháp quy nạp để chứng minh khẳng định
(*)
1. Với n = 1, ta có: u1 + v1 + w1 = 1 + 1 + 1 = 3 ∈ Z
Với n = 2, ta có:
u2 + v2 + w2 =
b2
− c2
b − c
+
c2
− a2
c − a
+
a2
− b2
a − b
= 2(a + b + c) = 2(−
−1
1
) = 2 ∈ Z
Với n = 3 ta có:
u2 + v2 + w2 =
b3
− c3
b − c
+
c3
− a3
c − a
+
a3
− b3
a − b
= 2(a2
+ b2
+ c2
) + (bc + ca + ab)
= 2(a + b + c)2
− 3(bc + ca + ab)
= 2(−
−1
1
)2
− 3(
−1
1
) = 5 ∈ Z
Vậy khẳng định đúng với n = 1, 2, 3.
2. Giả sử khẳng định đúng với n = k, k +1, k +2(k ≥ 1). Ta chứng minh
khẳng định đúng với n = k + 3. Thật vậy, ta có:
uk+3 +vk+3 +wk+3 = (uk+2 +uk+1 +uk)+(vk+2 +vk+1 +vk)+(wk+2 +
wk+1 + wk)
(uk+2 + vk+2 + wk+2) + ((uk+1 + vk+1 + wk+1) + (uk + vk + wk)
Theo giả thiết quy nạp, cả ba số hạng của tổng trên đều nguyên nên
(uk+3 + vk+3 + wk+3) cũng nguyên. Khẳng định được chứng minh. Từ đó
hiển nhiên tổng ở đề bài là số nguyên.
Bài toán 1.3.5. (IMO 1973) Cho
−−→
OP1,
−−→
OP2, ...,
−−−−→
OP2n+1 là các vecto đơn
vị trong mặt phẳng. Các điểm P1, P2, P2n+1 đều cùng nằm về 1 phía của
đường thẳng qua O. Chứng minh rằng:
|
−−→
OP1 +
−−→
OP2 + ... +
−−−−→
OP2n+1| ≥ 1
Chứng minh.
9
1. Với n = 1, mệnh đề hiển nhiên đúng do:
|
−−→
OP1| = 1 ≥ 1
2. Giả sử mệnh đề đúng với n = k − 1(k ≥ 2), tức là với hệ vecto
đơn vị:
−−→
OP1,
−−→
OP2, ...,
−−−−→
OP2k−1 cùng nằm về 1 phía của 1 đường thẳng qua
O, ta đã có:
|
−−→
OP1 +
−−→
OP2 + ... +
−−−−→
OP2k−1| ≥ 1
Ta chứng minh mệnh đề đúng với n = k, tức là với hệ (2k + 1) vecto
−−→
OP1,
−−→
OP2, ...,
−−−−→
OP2k+1 thỏa mãn các điều kiện trên, ta cũng có:
|
−−→
OP1 +
−−→
OP2 + ... +
−−−−→
OP2k+1| ≥ 1
Thật vậy, do vai trò của
−−→
OPi(1 ≤ i ≤ 2k + 1) như nhau nên ta có thể
sắp xếp lại sao cho
−−→
OPi(1 ≤ i ≤ 2k − 1) nằm giữa
−−−→
OP2k và
−−−−→
OP2k+1
Đặt:
−→u =
−−−→
OP2k +
−−−−→
OP2k+1
−→v =
−−→
OP1 +
−−→
OP2 + ... +
−−−−→
OP2k−1
Khi đó −→u có phương nằm trên tia phân giác góc P2kOP2k+1. Áp dụng
quy tắc hình bình hành nhiều lần, ta được −→v nằm giữa
−−→
OP1 và
−−−−→
OP2k−1,
nên nó nằm giữa
−−−→
OP2k và
−−−−→
OP2k+1.
Vậy góc giữa −→u và −→v bé hơn hoặc bằng
π
2
Ta lại có:
(−→u + −→v )2
= −→u
2
+ −→v
2
+ 2−→u −→v
= −→u
2
+ −→v
2
+ 2|−→u ||−→v |cos(−→u , −→v )
≥ −→v
2
(do cos(−→u , −→v ) ≥ 0)
Do đó: |−→u + −→v | ≥ |−→v |
Mà theo giả thiết quy nạp ta có |−→v | ≥ 1.
Vậy |−→u + −→v | ≥ 1 hay
|
−−→
OP1 +
−−→
OP2 + ... +
−−−−→
OPn+1| ≥ 1
Mệnh đề đúng với n = k + 1, ta có điều phải chứng minh!
Bài toán 1.3.6. (Chứng minh tính chia hết bằng quy nạp)
(Định lý Fermat nhỏ) Chứng minh rằng: Nếu p là số nguyên tố, thì
với mọi số nguyên dương n, hiệu np
− n chia hết cho p.
10
Chứng minh.
1. Với n = 1 ta có: 1p
− 1 = 0 chia hết cho p.
2. Giả sử khẳng định đúng với số nguyên n = a ≥ 1, nghĩa là ap
− a
chia hết cho p. Ta chứng minh: (a + 1)p
− (a + 1) cũng chia hết cho p.
Theo khai triển nhị thức Newton ta có:
(a + 1)p
− (a + 1) = C0
p.ap
+ C1
p.ap−1
+ C2
p.ap−2
+ ... + +Cp−1
p .a + Cp
p − a − 1
= (ap
− a) + C1
p.ap−1
+ C2
p.ap−2
+ ... + +Cp−1
p .a
Ta có:
Ck
p =
p!
k!(p − k)!
=
p(p − 1)...(p − k − 1)
1.2.3...k
(với 1 ≤ k ≤ p − 1)
Do p là số nguyên tố nên (p, k) = 1; ∀k, 1 ≥ k ≥ p − 1, suy ra Ck
p chia
hết cho p với mọi k, 1 ≤ k ≤ p − 1.
Mà ap
− a cũng chia hết cho p (theo giả thiết quy nạp)
Vậy (a+1)p
−(a+1) cũng chia hết cho p, định lý được chứng minh!
Bài toán 1.3.7. (Vô địch toán Hungari 1932) Chứng minh rằng nếu
a, b, n là những số tự nhiên và b chia hết cho an
, thì số (a + 1)b
− 1 chia
hết cho an+1
Chứng minh. (Quy nạp theo n)
1. Với n = 0, ta có (a + 1)0
− 1 luôn chia hết cho a0+1
= a, nên mệnh
đề đúng với n = 0.
2. Giả sử mệnh đề đúng với số tự nhiên n = k nào đó, tức là nếu b
chia hết cho ak
thì (a + 1)b
− 1 chia hết cho ak+1
.
Ta chứng minh mệnh đề đúng với n = k +1, tức là nếu b1 là số tự nhiên,
b1 chia hết cho ak+1
thì (a + 1)b1
− 1 chia hết cho ak+2
.
Thật vậy, đặt c =
b1
a
, thì c chia hết cho ak
.
Ta có:
(a + 1)b1
− 1 = (a + 1)ca
− 1
= [(a + 1)c
]a
− 1
= [(a + 1)c
− 1][(a + 1)c(a−1)
+ (a + 1)c(a−2)
+ ... + (a + 1)c
+ 1]
Biểu thức trong dấu ngoặc vuông thứ nhất chia hết cho ak+1
(theo giả
thiết quy nạp).
11
Biểu thức trong dấu ngoặc vuông thứ hai chia hết cho a vì ta có thể biểu
diễn nó dưới dạng:
[(a + 1)c(a−1)
− 1] + [(a + 1)c(a−2)
− 1] + ... + [(a + 1)c
− 1] + a
(Mỗi số hạng tổng này đều chia hết cho a)
Vậy (a + 1)b1
− 1 chia hết cho ak+2
. Mệnh đề được chứng minh!
Bài toán 1.3.8. (Chứng minh đẳng thức bằng quy nạp)
(Công thức nhị thức Newton) Chứng minh rằng:
(a + b)n
=
n
k=0
ck
n.ak
.bn−k
Chứng minh.
1. Với n = 1, dễ thấy công thức đúng.
2. Giả sử công thức đúng với số nguyên dương n, tức là ta có:
(a + b)n
=
n
k=0
Ck
n.ak
.bn−k
Ta chứng minh công thức đúng với n + 1. Thật vậy, ta có:
(a + b)n+1
= (a + b)n
.(a + b)
= (an
+ C1
n.an−1
.b + ... + Ck
n.an−k
.bk
+ ... + bn
)(a + b)
= an+1
+ C1
n.an
.b + ... + Ck
n.an+1−k
.bk
+ ... + a.bn
+ an
.b
+ C1
n.an−1
.b2
+ ... + Ck
n.an−k
.bk+1
+ ... + bn+1
= an+1
+ (C0
n + C1
n).an
.b + (C1
n + C2
n).an−1
.b2
+ ... + (Ck−1
n + Ck
n).aa+1−k
.bk
+ ... + bn+1
= an+1
+ C1
n+1.an
.b + C2
n+1.an−1
.b2
+ Ck
n+1.an+1−k
.bk
+ ... + bn−1
=
n+1
k=0
Ck
n+1.ak
.bn+1−k
Công thức được chứng minh!
Bài toán 1.3.9. (Chứng minh bất đẳng thức bằng quy nạp)
(Bất đẳng thức Bernoulli) Chứng minh rằng với mọi x > −1, x = 0
và mọi số tự nhiên n ≥ 2, ta có:
(1 + x)n
> 1 + nx
12
Chứng minh.
1. Với n = 2, bất đẳng thức có dạng: (1 + x)2
> 1 + 2x hay x2
> 0.
Điều này đúng do x = 0.
2. Giả sử bất đẳng thức đúng với n = k(k ≥ 2), tức là đã có: (1+x)k
>
(1 + kx)
Ta chứng minh nó cũng đúng với n = k + 1. Thật vậy, ta có:
(1 + x)k+1
= (1 + x)k
(1 + x) > (1 + kx)(1 + x)
= 1 + (k + 1)x + kx2
> 1 + (k + 1)x
(Vì k > 0 và x = 0)
Vậy bất đẳng thức được chứng minh!
Chú ý: Bất đẳng thức Bernoulli còn đúng cho mọi số thực α > 1:
(1 + x)α
> l + α; ∀x, x > −1, x = 0
Bài toán 1.3.10. (Vô địch toán Matxcova 1984) Cho x1, x2, ..., xn là n
số không âm (n ≥ 4), tổng của chúng bằng 1. Chứng minh rằng:
x1x2 + x2x3 + ... + xnx1 ≤ 4
Chứng minh. Ta sẽ chứng minh bất đẳng thức sau bằng phương pháp
quy nạp:
(x1 + x2 + .... + xn)2
≥ 4(x1x2 + x2x3 + .... + xnx1)
Với xi ≥ 0, i = 1, n và n ≥ 4
1. Với n = 4, ta có:
(x1 − x2 + x3 − x4)2
= x2
1 + x2
2 + x2
3 + x2
4
− 2x1x2 + 2x1x3 − 2x1x4 − 2x2x3 + 2x2x4 − 2x3x4
= (x1 + x2 + x3 + x4)2
− 4(x1x2 + x2x3 + x3x4 + x4x1) ≥ 0
Do đó:
(x1 + x2 + x3 + x4)2
≥ 4(x1x2 + x2x3 + x3x4 + x4x1)
nên bất đẳng thức đúng với n = 4.
2. Giả sử bất đẳng thức đúng với n = k(k ≥ 4), tức là ta có:
(x1 + x2 + ... + xk)2
≥ 4(x1x2 + x2x3 + ... + xkx1)
Ta chứng minh bất đẳng thức đúng với n = k + 1:
(x1 + x2 + ... + xk + xk+1)2
≥ 4(x1x2 + x2x3 + ... + xkxk+1 + xk+1x1)
13
Vì tổng 2 vế của bất đẳng thức này là vòng tròn theo chỉ số, ta có thể
giả thiết xk+1 ≥ xi, i = 1, k.
Khi đó, từ giả thiết quy nạp ta có:
(x1 + x2 + ... + xk + xk+1)2
= (x1 + x2 + ... + (xk + xk+1))2
≥ 4[x1x2 + x2x3 + ... + xk−1(xk + xk+1) + (xk + xk+1)x1]
Mà:
[x1x2 + x2x3 + ... + xk−1(xk + xk+1) + (xk + xk+1)x1]
= (x1x2 + x2x3 + ... + xkxk+1 + xk+1x1) + xk−1xk+1 + xk(x1 − xk+1)
Vì xi ≥ 0 và x1 − xk+1 ≥ 0 nên ta có:
[x1x2 + x2x3 + ... + xk−1(xk + xk+1) + (xk + xk+1)x1]
≥ (x1x2 + x2x3 + ... + xk−1xk + xkxk+1 + xk+1x1)
Vậy
(x1 + x2 + ... + xk + xk+1)2
≥ 4(x1x2 + x2x3 + ... + xkxk+1 + xk+1x1)
nên ta có điều phải chứng minh!
Bài toán 1.3.11. (Tính toán bằng quy nạp)
Tính bán kính rn, Rn của đường tròn nội tiếp và ngoại tiếp 2n
- giác
đều chu vi p cho trước (n ≥ 2).
Chứng minh.
1. Với n = 2, ta có hình vuông chu vi p. Dễ dàng tính được:
r2 =
p
8
; R2 =
p
√
2
8
2. Giả sử biết bán kính rn, Rn của các đường tròn nội tiếp và ngoại
tiếp 2n
- giác đều chu vi p, ta tính các bán kính rn+1, Rn+1 của các đường
tròn nội tiếp và ngoại tiếp 2n+1
- giác đều chu vi p.
Gọi AB là cạnh của 2n
- giác đều chu vi p tâm O. Gọi C, D, E, F lần
lượt là trung điểm của cung AB, dây AB, AC, BC, G là trung điểm của
EF. Vì
(OE, OF) = (OE, OC) + (OF, OC)
=
1
2
(OA, OC) +
1
2
(OB, OC) =
1
2
(OA, OB)
14
Hình 1.3.11
và EF bằng cạnh của 2n+1
- giác đều nội tiếp đường tròn bán kính OE,
nên chu vi của 2n+1
- giác này là:
2n+1
.EF = 2n+1 AB
2
= 2n
.AB = p
Do đó rn+1 = OG và Rn+1 = OE
Mặt khác, ta dễ thấy
OC − OG = OG − OD(=
1
2
CD)
nên Rn − rn+1 = rn+1 − rn hay rn+1 = Rn+rn
2
Cuối cùng, từ tam giác vuông OEC ta có: OE2
= OC.OG. Nghĩa là:
R2
n+1 = Rnrn+1, nên Rn+1 =
√
Rnrn+1.
Vậy rn+1 = Rn+rn
2 , Rn+1 =
√
Rnrn+1.
Bài toán 1.3.12. Trên một hình phẳng cho n đường tròn phân biệt, đôi
một cắt nhau và không có ba đường tròn nào giao nhau tại 1 điểm. Các
mặt phẳng này chia đường tròn thành các miền rời nhau. Tính số miền
thu được.
Chứng minh. Gọi số miền thu được bởi n đường tròn trong mặt phẳng
thỏa mãn điều kiện đề bài là F(n).
1. Với n = 1, dễ thấy F(1) = 2.
Với n = 2, ta có 2 đường tròn cắt nhau và F(2) = 4
2. Giả sử với k đường tròn thỏa mãn điều kiện đề bài, chúng chia mặt
phẳng ra làm F(k) miền. Xét (k + 1) đường tròn thỏa mãn điều kiện đề
bài. Ta tính F(k + 1).
Gọi (k + 1) đường tròn đó là (C1), (C2), ..., (Ck+1). Bỏ đi 1 đường
tròn bất kỳ trong (k + 1) đường tròn đó, chẳng hạn (Ck+1). Khi đó còn
15
Hình 1.3.12
k đường tròn, theo giả thiết quy nạp, số miền thu được là F(k). Bây
giờ ta dựng lại Ck+1. Khi đó đường tròn Ck+1 giao với cả k đường tròn
ban đầu. Trên Ck+1 có k cặp giao điểm nên cho ta thêm 2k miền. Vậy
F(k + 1) = F(k) + 2k. Do đó ta có:
F(k) = F(k − 1) + 2(k − 1)
F(k − 1) = F(k − 2) + 2(k − 2)
...
F(2) = F(1) + 2.1
F(1) = 2
Cộng các đẳng thức trên lại ta được:
F(k) = 2[1 + 1 + 2 + 3 + ... + (k − 2) + (k − 1)]
= 2[1 +
k(k − 1)
2
]
= k2
− k + 2
Vậy số miền thu được từ n đường tròn thỏa mãn đề bài là F(n) =
n2
− n + 2.
Bài toán 1.3.13. (Chứng minh bằng quy nạp)
Chứng minh rằng mọi n - giác lồi(n ≥ 5) đều được chia thành 1 số
hữu hạn các ngũ giác lồi.
Chứng minh.
1. Với n = 5, mệnh đề hiển nhiên đúng.
2. Giả sử mệnh đề đúng với n = k(k ≥ 5), tức là mọi k - giác lồi đều
chia được thành hữu hạn các ngũ giác lồi. Ta chứng minh mệnh đề đúng
với n = k + 1, tức là mọi (k + 1) - giác lồi (H) đều chia được thành một
số hữu hạn các ngũ giác lồi.
16
Thật vậy, xét (k + 1) - giác lồi (H) = A1A2...AkAk+1. Trên các cạnh
A1Ak+1 và A3A4 lần lượt lấy các điểm M, N khác các đỉnh. Đoạn MN
chia (H) thành 2 đa giác:
(H1) = MA1A2A3N và (H2) = MNA4A5...AkAk+1
Rõ ràng (H1) là ngũ giác lồi, còn (H2) là k giác lồi. Theo giả thiết quy
Hình 1.3.13
nạp, (H2) chia được thành 1 số hữu hạn các ngũ giác lồi.
Mệnh đề được chứng minh!
Bài toán 1.3.14. (Chắp hình bằng quy nạp)
Cho n hình vuông bất kỳ. Chứng minh có thể cắt chúng (bằng nhát cắt
thẳng) làm một số mảnh đa giác, để từ đó có thể ghép lại thành 1 hình
vuông lớn.
Chứng minh.
1. Với n = 1, mệnh đề hiển nhiên đúng.
Với n = 2, gọi độ dài các cạnh 2 hình vuông A1B1C1D1 và A2B2C2D2
tương ứng là x1 và x2. Giả sử x1 ≥ x2. Trên các cạnh của hình vuông
A1B1C1D1 ta lấy các điểm M, N, P, Q sao cho
A1M = B1N = C1P = D1Q =
x1 + x2
2
Cắt hình vuông này theo các đoạn MP và NQ thì MP và NQ cắt nhau
tại tâm O của nó và chúng vuông góc với nhau. Các đường đó chia
hình vuông thành 4 phần bằng nhau, ta ghép những phần đó vào hình
vuông A2B2C2D2 như hình bên. Hình nhận được sẽ là hình vuông vì
giá trị tại các góc M, N, P, Q bù nhau; các góc A, B, C, D vuông và
AB = BC = CD = DA.
2. Giả sử mệnh đề đúng với n(n ≥ 1) hình vuông. Ta chứng minh
mệnh đề cũng đúng với (n + 1) hình vuông.
17
Thật vậy, giả sử ta có n + 1 hình vuông (H1), (H2), ..., (Hn), (Hn+1).
Ta lấy ra 2 hình vuông bất kỳ, chẳng hạn (Hn), (Hn+1). Theo phần 1,
ta có thể cắt một trong hai hình vuông này và ghép các mảnh với hình
vuông còn lại để được 1 hình vuông mới (H’). Như vậy, ta có n hình
vuông (H1), (H2), ..., (Hn−1), (H ). Theo giả thiết quy nạp, có thể cắt và
ghép chúng thành một hình vuông mới. Ta có điều phải chứng minh!
Hình 1.3.14
Bài toán 1.3.15. (Tô màu bằng quy nạp)
Trên mặt phẳng cho n(n ≥ 1) hình tròn. Chứng minh có thể với bất
kỳ cách sắp đặt nào, thì hình nhận được cũng có thể tô bằng 2 màu, để
cho 2 phần mặt phẳng kề nhau (có biên chung) cũng được tô bằng hai
màu khác nhau.
Chứng minh.
1. Cơ sở quy nạp. Với n = 1, trên mặt phẳng chỉ có 1 hình tròn. Ta
tô hình tròn bằng màu đen. Khi đó phần mặt phẳng còn lại kề với hình
tròn được để trắng, nên hai phần của mặt phẳng kề nhau có màu khác
nhau.
2. Quy nạp. Giả sử khẳng định đã đúng với bức tranh n hình tròn.
Giả sử trên mặt phẳng cho n + 1 hình tròn tùy ý. Xóa đi 1 trong những
hình tròn, sẽ được bức tranh gồm n hình tròn. Theo giả thiết quy nạp,
bức tranh này chỉ cần tô bằng hai màu, chẳng hạn, đen, trắng mà hai
miền kề nhau đều có màu khác nhau.
Khôi phục lại hình tròn đã xóa đi, tức là trở lại hình xuất phát gồm
n + 1 hình tròn, rồi theo 1 phía đối với hình tròn vừa khôi phục, chẳng
18
Hình 1.5
hạn phía trong của hình tròn này thay đổi các màu đã tô bằng màu
ngược lại, sẽ được: bức tranh gồm n+1 hình tròn được tô bằng hai màu,
mà hai miền kề nhau tùy ý đều có màu khác nhau.
Bài toán 1.3.16. (Dựng hình bằng quy nạp)
Trên mặt phẳng cho (2n + 1) điểm. Hãy dựng một (2n + 1) giác để
các điểm đã cho là trung điểm các cạnh của đa giác.
Chứng minh.
1. Với n = 1, ta dựng tam giác ABC khi biết 3 trung điểm M, N, P
bằng cách qua M, N, P lần lượt dựng các đường thẳng song song với NP,
MP, MN. Chúng cắt nhau cho ta tam giác ABC.
2. Giả sử đối với 2(n − 1) + 1 điểm tùy ý không thẳng hàng dựng
được đa giác (2n − 1) đỉnh có các điểm đã cho là trung điểm các cạnh.
Ta chứng minh có thể dựng được (2n + 1) - giác từ trung điểm các cạnh
của nó.
Xét 2n+1 điểm tùy ý không có ba điểm nào thẳng hàng A1, A2, ..., A2n+1.
Giả sử các điểm này là trung điểm các cạnh của (2n+1) - giác cần dựng
B1, B2, ..., B2n+1
Xét tứ giác B1B2n−1B2nB2n+1 có A2n−1A2nA2n+1 lần lượt là trung điểm
các cạnh B2n−1B2n, B2nB2n+1, B2n+1B1. Gọi A là trung điểm B1B2n−1 thì
AA2n−1A2nA2n+1 là hình bình hành. Vì A2n−1, A2n, A2n+1cho trước nên
ta dựng được A.
19
Hình 1.6
Xét (2n − 1) - giác B1B2...B2n−1 có (2n − 1) trung điểm các cạnh là:
A1, A2, ..., A2n−2, A đã xác định nên theo giả thiết quy nạp ta dựng được
đa giác này. Từ B1, B2n−1 và các trung điểm A2n−1, A2n+1 ta dựng được
nốt B2n vàB2n+1.
Vậy ta dựng được (2n + 1) - giác B1B2...B2n+1 khi biết trung điểm
các cạnh của nó.
Bài toán 1.3.17. (Chia hình bằng quy nạp)
Hãy chia một lục giác thành các tam giác đen, trắng sao cho:
(i) Hai tam giác tùy ý hoặc rời nhau hoặc có đỉnh chung hoặc có cạnh
chung.
(ii) Hai tam giác tùy ý có cạnh chung thì có màu khác nhau.
(iii) Mỗi cạnh của lục giác đồng thời là cạnh của đa giác đen.
Chứng minh rằng đa giác n cạnh có thể chia được theo cách trên khi
và chỉ khi n
...3
Chứng minh. Đối với lục giác, có ít nhất hai cách chia theo hình sau
thỏa mãn 3 điều kiện trên.
Ta chứng minh n - giác được chia theo cách trên khi và chỉ khi n
...3.
a) Điều kiện cần:
Giả sử n - giác đã chia được thành các tam giác thỏa mãn ba điều
kiện trên. Ta chứng minh n
...3.
Gọi tổng số cạnh của tất cả các tam giác đen là x, tổng số cạnh của
tất cả các tam giác trắng là y
20
Hình 1.7
Vì hai tam giác chung cạnh có màu khác nhau và cạnh của đa giác
luôn là cạnh của tam giác đen nên:
x = y + n
Mà x
...3, y
...3 nên n
...3
b) Điều kiện đủ:
Giả sử n - giác T có n
...3. Ta cần chỉ ra rằng có thể chia T thành các
tam giác thỏa mãn 3 điều kiện trên.
Vì n
...3 nên tồn tại số nguyên dương k sao cho n = 3k. Ta chứng minh
bằng quy nạp theo k.
1. Với k = 1, ta có tam giác, có thể chia như sau:
Hình 1.8
2. Giả sử đối với các đa giác có n = 3k cạnh, ta đã chia được thành
các tam giác thỏa mãn 3 điều kiện trên. Ta cần chứng minh đa giác có
n = 3(k + 1) cạnh cũng chia tương tự như vậy.
Xét đa giác có n = 3(k + 1) cạnh: A1A2...A3k+2A3k+3
Theo giả thiết quy nạp, đa giác có 3k cạnh A1A2...A3k chia được thành
các tam giác thỏa mãn ba điều kiện trên. Còn lại ngũ giác A1A3kA3k+1A3k+2A3k+3
trong đó A1A3k đã là cạnh của tam giác đen, ta chia tiếp các ngũ giác
này. Có nhiều cách, chẳng hạn lấy M thuộc miền trong ngũ giác, tô
tam giác A1MA3k màu trắng; lúc đó lục giácA1MA3kA3k+1A3k+2A3k+3
dễ dàng chia được thành các tam giác thỏa mãn 3 điều kiện trên.
Như vậy, đối với đa giác có n = 3(k + 1) cạnh ta cũng chia được theo
cách trên. Bài toán được chứng minh!
21
Hình 1.9
Bài toán 1.3.18. Chứng minh rằng: Nếu có một số tiền nguyên (nghìn)
đồng Việt Nam lớn hơn 6.000đ thì luôn luôn có thể đổi ra những tờ tiền
lẻ loại 2.000đ và 5.000đ.
Chứng minh. Ta sẽ giải quyết bài toán này bằng phương pháp quy nạp:
1. Cơ sở quy nạp. Nếu trong túi có số tiền ít nhất, tức là 7.000đ thì
ta đổi 1 tờ tiền loại 5.000đ và 1 tờ tiền loại 2.000đ. Khi đó:
7.000đ = 5.000đ + 2.000đ (1)
8.000đ = 2.000đ + 2.000đ + 2.000đ + 2.000đ (2)
2. Quy nạp. Giả sử với k(k ≥ 7.000đ) nghìn đồng ta đã đổi được sang
các tờ tiền loại 5.000đ và 2.000đ
Nếu ta thêm vào đẳng thức (1) và (2) tờ 2.000đ thì:
9.000đ = 7.000đ + 2.000đ (3)
10.000đ = 8.000đ + 2.000đ (4)
Tiếp tục thêm 2.000đ vào 2 đẳng thức (3) và (4) thì ta có:
11.000đ = 9.000đ + 2.000đ
12.000đ = 10.000đ + 2.000đ
Ta còn tiếp tục được cho bất kỳ số nguyên nào. Ta thấy rằng ở bước
trước có 2 đẳng thức và suy ra ở bước sau có 2 đẳng thức. Như vậy với
mọi số k nguyên đồng nào đó dù là số chẵn hay số lẻ khi k − 2 cũng sẽ
đổi ra được 2 loại tiền 2.000đ và 5.000đ. Suy ra nó cũng đổi được thành
2.000đ và 5.000đ. Như vậy khẳng định của mệnh đề là đúng.
22
1.4 Bài tập tự giải
Bài toán 1.4.1. Tính số tam giác (T(n)) của một đa giác n đỉnh được
chia bởi các đường chéo không cắt nhau.
Hướng dẫn.
T(n) = n − 2. Tính bằng quy nạp theo số đỉnh của đa giác.
Bài toán 1.4.2. Cho n + 1(n ≥ 1) số nguyên dương a0, a1, a2, ..., an.
Biết rằng a1 ≥ a0, a2 = 3a1 − 2a0, a3 = 3a2 − 2a1, ..., an = 3an−1 − 2an−2.
Chứng minh rằng: an > 2n−1
.
Hướng dẫn.
Theo giả thiết có a1 − a0 ≥ 1, ai = 2(ai−1 − ai−2) + ai−1(2 ≤ i ≤ k)
Nhân vế với vế k + 1 đẳng thức trên ta suy ra ak ≥ 2k−1
.
Nhờ bất đẳng thức vừa nhận được và quy nạp theo k chứng minh được
ak ≥ 2k−1
. Từ đây có an ≥ 2n−1
.
Bài toán 1.4.3. Trong mặt phẳng cho n đường thẳng. Biết rằng hai
đường thẳng bất kỳ không song song, ba đường thẳng bất kỳ không đồng
quy. Hỏi n đường thẳng này chia mặt phẳng thành bao nhiêu miền.
Hướng dẫn.
Dùng phương pháp quy nạp theo n, ta có kết quả số miền có được là
F(n) =
n2
+ n + 2
2
Bài toán 1.4.4. Cho cấp số cộng vô hạn với các thành phần là các số
nguyên dương. Một trong những thành phần của cấp số cộng này là số
chính phương. Chứng minh rằng trong cấp số cộng này có một số vô hạn
các số chính phương.
Hướng dẫn.
Giả sử cấp số cộng có công sai là d và một trong những thành phần
chính phương của nó là a = m2
, trong đó m là một số tự nhiên. Khi đó
∀k ∈ N số
bk = (m + kd)2
= m2
+ 2mdk + k2
d2
= a + d(2mk + k2
d)
cũng là một thành phần của cấp số cộng được xét. Bởi vậy cấp số cộng
đã cho chứa một số vô hạn các thành phần là số chính phương.
23
Bài toán 1.4.5. Chứng minh rằng với mọi số tự nhiên n, số 23n
+1 chia
hết cho 3n+1
và không chia hết cho 3n+2
.
Hướng dẫn.
Chứng minh khẳng định đúng với n = k
23k+1
= (23k
)3
+ 1
= (23k
+ 1)[(23k
)2
− 23k
+ 1]
= M.3k+1
[(23k
+ 1)2
− 3.23k
]
= 3k+2
.M[32k+1
.M2
− 23k
]
Do đó 23k+1
chia hết cho 3k+2
Chứng minh biểu thức [32k+1
.M2
− 23k
] không chia hết cho 3
32k+1
.M2
chia hết cho 3
23k
≡ ±1(mod3) hay 23k
không chia hết cho 3.
Nghĩa là 23k+1
không chia hết cho 3k+3
.
Bài toán 1.4.6. Chứng minh rằng mỗi số tự nhiên không vượt quá n!
đều phân tích được thành tổng gốm không quá n số, sao cho 2 số bất kỳ
đều khác nhau và mỗi số này là ước của n!.
Chứng minh.
1. Cơ sở quy nạp. Với n = 1, khẳng định hiển nhiên thỏa mãn.
2. Giả sử khẳng định đã đúng với n = k.
Giả sử a là số tự nhiên tùy ý và a < (a + 1)!.
Chia a cho n+1 với số dư r và thương d. Khi đó: a = d(n+1)+r, trong
đó d ≤ n!, r < n + 1.
Theo giả thiết quy nạp d = d1 + d2 + ... + dl, trong đó di(1 ≤ i ≤ l) là
các số tự nhiên khác nhau từng đôi 1, và là ước của n!, đồng thời l ≤ n.
Khi đó:
a = d1(n + 1) + ... + dl(n + 1) + r
và trong tổng này có không quá n +1 số khác nhau từng đôi một và đều
là ước của (n + 1)!
24
Chương 2
Phương pháp phản chứng
2.1 Phép suy luận phản chứng
Phép suy luận phản chứng là quá trình ta đưa ra một giả thiết
(giả thiết này đối lập với điều cần tìm) rồi đi tìm đến sự vô lý để loại
trừ giả thiết ta vừa đặt ra.
2.2 Phương pháp chứng minh bằng phản chứng
Phương pháp chứng minh bằng phản chứng là phương pháp sử dụng
phép suy luận phản chứng để chứng minh, diễn giải những khẳng định
toán học.
Trong lịch sử toán học, phương pháp chứng minh bằng phản chứng
đã được sử dụng từ rất sớm. Người ta sử dụng nó trong chứng minh
nguyên lý Dirichlet.
Bài toán 2.2.1. (Nguyên lý Dirichlet) Người ta nhốt m con thỏ vào n
cái lồng, (m > n). Chứng minh rằng có ít nhất 2 con thỏ được nhốt trong
cùng một lồng nào đó.
Chứng minh. Giả sử mi là số con thỏ được nhốt vào lồng thứ i, (i = 1, n).
Khi đó ta có
m1 + m2 + ... + mn = m.
Giả sử ngược lại, mỗi lồng chỉ nhốt nhiều nhất một con thỏ, tức là
0 ≥ mi ≥ 1, (i = 1, n).
Khi đó m = m1 + m2 + ... + mn ≤ 1 + 1 + ... + 1
n số 1
= n
Điều này mâu thuẫn với giả thiết m > n.
25
Vậy điều ta giả sử là sai, nghĩa là phải có ít nhất 2 con thỏ được nhốt
trong một lồng nào đó.
2.3 Các bước suy luận trong chứng minh phản chứng
Trong toán học, phương pháp phản chứng rất thường xuyên được sử
dụng, nó là công cụ đắc lực trong chứng minh một số bài toán khó. Vậy
câu hỏi đặt ra là: Phương pháp chứng minh phản chứng sử dụng khi
nào? Cách chứng minh phản chứng như thế nào?
Phương pháp chứng minh phản chứng sử dụng khi nào?
Khi gặp những bài toán khẳng định một hệ thức đúng, khẳng định
nghiệm của phương trình, hệ phương trình hoặc bất đẳng thức ... trong
đại số, hình học, số học, giải tích hay các bài toán không mẫu mực. Đặc
biệt khi cần chứng minh tính tồn tại duy nhất của một đối tượng người
ta hay dùng phản chứng để chứng minh.
Nhận xét trên đây chỉ là kinh nghiệm của tác giả trong quá trình
giải một số bài toán. Tùy vào từng bài toán, tình huống cụ thể khác mà
người giải toán vận dụng phương pháp này một cách linh hoạt.
Các bước trong chứng minh phản chứng:
Ta chứng minh mệnh đề P là đúng.
Bước 1. Giả sử mệnh đề P là sai (tức là chúng ta đi phủ định mệnh
đề cần chứng minh).
Bước 2. Từ điều giả sử ta suy ra một số tính chất hoặc quan hệ mới
mà những tính chất này dẫn tới điều vô lý.
Bước 3. Ta kết luận điều giả sử ban đầu là sai. Vậy mệnh đề P là
mệnh đề đúng.
Chú ý: Trong ba bước suy luận phản chứng nêu trên, bước 1 rất
quan trọng vì chúng ta cần tạo ra mệnh đề phủ định điều cần chứng
minh phải chính xác.
Ở bước 2 điều vô lý có thể thuộc một trong các dạng sau:
Điều trái với giả thiết đã cho
Điều trái với một trong các kiến thức đã biết.
Điều trái với giả thiết phản chứng đặt ra.
26
2.4 Vận dụng phương pháp phản chứng để giải các
bài toán không mẫu mực
Bài toán 2.4.1. Chứng minh tồn tại vô số số nguyên tố.
Chứng minh.
Giả sử ngược lại, tồn tại hữu hạn số nguyên tố p1, p2, ..., pn. Ta xét tích
N = p1p2...pn + 1.
Hiển nhiên N phải có ít nhất một ước số nguyên tố p nào đó. Khi đó, do
p1, p2, ..., pn là tất cả các số nguyên tố nên tồn tại i, (1 ≤ i ≤ n) sao cho
p = pi.
Từ đó suy ra được p|1 hay p = 1 (mâu thuẫn với p là số nguyên tố).
Vậy điều ta giả sử là sai. Nghĩa là có vô số số nguyên tố.
Bài toán 2.4.2. Trong lớp học có 30 học sinh. Khi viết chính tả, em
An phạm 13 lỗi còn các em khác ít hơn. Chứng minh rằng trong lớp có
ít nhất 3 em học sinh đã mắc một số lỗi như nhau khi viết chính tả (kể
cả những người không mắc lỗi nào).
Chứng minh. Ta có thể vận dụng nguyên lý L. Dirichlet để giải bài toán
này. Ở đây, thỏ tức là các em học sinh còn lồng là số lỗi đã phạm phải
khi các em viết chính tả.
Ta lập 14 lồng được đánh số từ 0 đến 13.
Lồng số 0 "nhốt" các em viết chính tả phạm 0 lỗi;
Lồng số 1 "nhốt" các em viết chính tả phạm 1 lỗi;
Lồng số i, (0 ≤ i ≤ 13) "nhốt" các em viết chính tả phạm i lỗi;
Chỉ có em An phạm 13 lỗi khi viết chính tả nên lồng số 13 chỉ có một
mình em An, 29 em còn lại được "nhốt" vào các lồng từ 0 đến 12, tức
là 29 em được "nhốt" vào 13 lồng, nên phải có ít nhất một lồng nhốt từ
3 em trở lên. Chẳng hạn lồng i(0 ≤ i ≤ 12) có ít nhất 3 em. Khi đó 3
trong các em ở lồng i cùng phạm i lỗi khi viết sai chính tả.
Bài toán được chứng minh!
Bài toán 2.4.3. Tìm các cặp số (m, n) nguyên dương sao cho m!+n! =
mn
.
Chứng minh.
Trước hết ta sẽ chỉ ra rằng các cặp số nguyên dương (m, n) muốn thỏa
27
mãn bài toán thì m ≤ n.
Phản chứng. Giả sử ngược lại, m > n.
Khi đó ta có
m! + n! = n![m(m − 1)...(n − 1) + 1] = mn
.
Do (m, m(m − 1)...(n + 1) + 1) = 1 nên đẳng thức trên không xảy ra.
Vậy điều ta giả sử là sai nên m ≤ n.
Xét trường hợp m > 2.
Vì
m! + n! = (m − 2)!(m − 1)m[1 + (m + 1)(m + 2)...(n − 1)n]
nên ta có
(m − 2)!(m − 1)m[1 + (m + 1)(m + 2)...(n − 1)n] = mn
.
Đẳng thức trên không xảy ra vì (m, m − 1) = 1 nên vế trái của đẳng
thức trên chia hết cho (m − 1), còn vế phải của đẳng thức không chia
hết cho (m − 1). Như vậy m ∈ 1, 2.
Với m = 1 thì 1! + n! = 1. Không tồn tại số n thỏa mãn.
Với m = 2 thì 2! + n! = 2n
.
Bây giờ ta đi tìm các giá trị của n thỏa mãn 2! + n! = 2n
.
Nếu n ≥ 4 thì n! = 1.2.3.4...n > 2.2.22
...2 = 2n
nên không thỏa mãn
2! + n! = 2n
.
Suy ra n = 2 hoặc n = 3. Thử trực tiếp ta có:
2! + 2! = 22
, 2! + 3! = 23
.
Như vậy các cặp số thỏa mãn bài toán là (2, 2) và (2, 3).
Bài toán 2.4.4. (Olympic Châu Á Thái Bình Dương 1998) Chứng minh
rằng với mọi số a, b nguyên dương, số (36a + b)(a + 36b) không thể là
một lũy thừa của 2.
Chứng minh.
Phản chứng. Giả sử ngược lai, tồn tại cặp số nguyên dương (a, b), sao
cho (36a + b)(a + 36b) là một lũy thừa của 2.
Trong số những cặp (a, b) như vậy, ta xét (m, n) là cặp có tổng m+n bé
nhất. Vì (36m+n)(m+36n) là một lũy thừa của 2 nên suy ra (36m+n)
28
và (m+36n) cũng là các lũy thừa của 2. Lúc đó m, n đều là các số chẵn.
Ta cũng thấy rằng
(36m + n) ≥ 37, (m + 36n) ≥ 37
Bây giờ, ta đặt
36m + n = 2r
, m + 36n = 2s
, m = 2p và n = 2q.
trong đó r, s, p, q là các số nguyên dương.
Ta có r, s > 5. Khi đó
(36p + q)(p + 36q) =
36m + n
2
x
m + 36n
2
= 2r+s−2
cũng là một lũy thừa của 2.
Nhưng ta lại có
p + q =
m + n
2
< m + n
Điều này mâu thuẫn với giả thiết về việc chọn cặp số (m, n) có tổng bé
nhất như trên. Mâu thuẫn này chứng tỏ rằng trong các cặp (a, b) mà
(36a + b)(a + 36b) là một lũy thừa của 2, ta không thể chọn được cặp số
có tổng bé nhất: vô lý!
Vậy số (36a + b)(a + 36b) không thể là một lũy thừa của 2.
Bài toán 2.4.5. Cho 20 số tự nhiên a1, a2, ..., a20, không vượt quá 70.
Chứng minh rằng trong các hiệu aj − ak(j > k) tìm được ít nhất 4 số
giống nhau.
Chứng minh.
Phản chứng.Giả sử khẳng định trên không đúng, tức là trong các hiệu
aj − ak(j > k) có không quá 3 hiệu giống nhau. Khi đó trong 19 số tự
nhiên
a20 − a19, a19 − a18..., a3 − a2, a2 − a1 (1)
có không quá 3 hiệu giống nhau. Bởi vậy không một số nào trong các số
1, 2, 3, 4, 5, 6 trong dãy (1) lặp quá 3 lần, mà dãy (1) có 19 số, bởi vậy
phải có ít nhất một trong các số thuộc (1) phải lớn hơn 6 (Trường hợp
ngược lại, nếu tất cả các dãy số thuộc dãy (1) đều không quá 6, khi đó
các số đều không lặp quá 3 lần, nên dãy (1) chỉ có thể có tối đa 18 số
chứ không phải 19 số như thực tế).
29
Khi đó
Ít nhất 3 trong 18 số còn lại lớn hơn 5.
Ít nhất 3 trong 15 số còn lại lớn hơn 4.
Ít nhất 3 trong 12 số còn lại lớn hơn 3.
Ít nhất 3 trong 9 số còn lại lớn hơn 2.
Ít nhất 3 trong 6 số còn lại lớn hơn 1.
Bởi vậy tổng các số thuộc dãy (1)
a20 − a1 = (a20 − a19) + (a19 − a18) + ... + (a3 − a2) + (a2 − a1)
≥ 7 + (6 + 6 + 6) + (5 + 5 + 5) + (2 + 2 + 2) + (1 + 1 + 1) = 70
Mặt khác
a20 − a1 ≤ 70 − 1 = 69 (3)
So sánh (2) và (3) ta đi đến mâu thuẫn. Bởi vậy trong các hiệu aj −ak(j >
k) phải có ít nhất 4 số giống nhau.
Bài toán 2.4.6. (Tuyển tập 5 năm Tạp chí Toán học & Tuổi trẻ 1991 -
1995) Trong một hình vuông cạnh bằng 1 cm, lấy 51 điểm tùy ý. Chứng
minh rằng luôn luôn có 3 điểm đã lấy ra nằm trong một hình tròn bán
kính bằng
1
7
.
Chứng minh. Bài toán được giải theo 2 bước.
1) Chia mỗi cạnh hình vuông thành 5 phần bằng nhau, rồi nối các
điểm chia tương ứng đối diện bằng các đoạn thẳng song song với cạnh
hình vuông. Khi đó hình vuông được chia thành 25 hình vuông nhỏ bằng
nhau và có cạnh bằng
1
5
cm.
Giả sử mỗi hình vuông nhỏ chứa không quá 2 điểm đã được lấy. Khi
đó số điểm đã được lấy sẽ không vượt quá 50, nên nhỏ hơn 51 điểm. Ta
đã đi tới mâu thuẫn với giả thiết: Các điểm đã được lấy bằng 51. Bởi
vậy phải có ít nhất một hình vuông con chứa không ít hơn 3 điểm đã
lấy.
Giả sử hình vuông con ở góc trên tận cùng bên phải chứa 3 điểm đã
chọn ra. Ta ký hiệu hình vuông này bằng ABCD.
2) Bao hình vuông ABCD bằng hình tròn bán kính
1
7
cm.
Hình vuông ABCD có hai đường chéo cắt nhau tại điểm giữa của mỗi
đường và vuông góc với nhau. Dùng O để ký hiệu giao điểm hai đường
chéo và x là độ dài nửa đường chéo.
30
Tam giác AOB vuông tại O có cạnh huyền AB dài
1
5
. Khi đó ta có
OA
2
+ OB
2
= 2x2
= AB
2
=
1
5
2
=
1
25
nên x2
=
1
50
<
1
49
=
1
7
2
Do đó x <
1
7
cm.
Bởi vậy, ta có thể bao hình vuông ABCD bằng hình tròn bán kính
1
7
Hình 2.4.6
cm, nên ba điểm đã chọn ra thuộc hình vuông ABCD nằm trong hình
tròn bán kính
1
7
cm.
Bài toán 2.4.7. Cho ba số a, b, c ∈ (0, 1). Chứng minh rằng ba số a(1 −
a), b(1 − b), c(1 − c) không thể đồng thời lớn hơn
1
4
.
Chứng minh.
Phản chứng. Ta giả sử ngược lại, cả ba số cùng lớn hơn
1
4
, tức là
a(1 − b) >
1
4
, b(1 − c) >
1
4
, c(1 − a) >
1
4
Nhân vế với vế của các bất đẳng thức đó, ta được
a(1 − b)b(1 − c)c(1 − a) >
1
64
(1)
31
Mặt khác, từ giả thiết a ∈ (0, 1) ta có
0 < a(1 − a) =
1
4
− (a −
1
2
)2
≥
1
4
(2)
Tương tự, ta có
0 < b(1 − b) ≥
1
4
; 0 < c(1 − c) ≥
1
4
(3)
Từ (2) và (3) suy ra
a(1 − b)b(1 − c)c(1 − a) ≤
1
64
(4)
Ta thấy (1) và (4) mâu thuẫn nhau, chứng tỏ điều giả sử ban đầu là sai.
Vậy ta có điều phải chứng minh.
Bài toán 2.4.8. Cho n+1 số tự nhiên khác nhau và nhỏ hơn 2n. Chứng
minh rằng từ các số này luôn luôn chọn ra được ba số, mà một trong ba
số này bằng tổng của hai số còn lại
Chứng minh. Giả sử n + 1 số đã cho là a1, a2, ..., an+1 và
a1 < a2 < ... < an < an+1
Ta lập n hiệu từ các số đã cho
h1 = a2 − a1, h2 = a3 − a1, ..., hn = an+1 − a1
Vì a1 nhỏ hơn tất cả các số còn lại, nên tất cả các hiệu hi(1 ≤ i ≤ n)
đều dương.
Do các số a2, a3, ..., an+1 khác nhau từng đôi một, nên hiệu h1, h2, ..., hn
đều khác nhau.
Vì an+1 < 2n, nên tất cả các hiệu hi(1 ≤ i ≤ n) đều nhỏ hơn 2n.
Như vậy, ta có cả thảy 2n + 1 số nguyên dương nhỏ hơn 2n.
a1, a2, ..., an, an+1, h1 = a2 − a1, h2 = a3 − a1, ..., hn = an+1 − a1
nên, theo nguyên lý phản chứng, phải có hai số bằng nhau. Nhưng các
số ai(1 ≤ i ≤ n + 1) và các số ht(1 ≤ t ≤ n) đều khác nhau, nên chỉ có
thể có một số ak nào đó phải bằng một số hs, tức
ak = as − a1
Từ đó ta có: as = ak + a1 và khẳng định được chứng minh.
32
Bài toán 2.4.9. Chứng minh rằng từ một tập bất kỳ gồm n số tự nhiên
đều có thể tách ra một tập con (không trống), mà tổng của các số thuộc
tập con này chia hết cho n.
Chứng minh.
Giả sử A = {a1, a2, ..., an} là tập các số tùy ý đã chọn ra.
Phản chứng. Giả sử khẳng định của đề bài không đúng (*), nghĩa là
không có một tập con nào gồm các phần tử thuộc tập A, mà tổng chia
hết cho n. Khi đó trong n tổng
S1 = a1, S2 = a1 + a2, ..., Sn = a1 + a2 + ... + an−1 + an
không có một tổng nào chia hết cho n. Bởi vậy, khi chia n tổng S1, S2, ..., Sn
cho n chỉ nhận được tối đa n − 1 loại số dư. Bởi vậy, theo nguyên lý
phản chứng, phải có ít nhất hai trong các số trên chia cho n có cùng số
dư. Chẳng hạn Si, Sj với j > i có cùng số dư khi chia cho n. Do đó
ai+1 + ai+2 + ... + aj = Sj − Si
chia hết cho n. Ta đã chứng tỏ khẳng định (*) là sai, nên khẳng định
của đề bài là đúng.
Bài toán 2.4.10. Gọi a là số lớn nhất của các hình tròn rời nhau (không
có điểm chung) có đường kính bằng 1 và tâm nằm trong đa giác F cho
trước, còn b là số nhỏ nhất những hình tròn có bán kính bằng 1 mà bằng
chúng có thể phủ kín đa giác F. Hỏi a ≥ b hay a < b?
Chứng minh.
Bằng phản chứng ta sẽ khẳng định được a ≥ b.
Do a là số lớn nhất các hình tròn rời nhau (từng cặp không có điểm
chung) có đường kính bằng 1, mà tâm nằm trong đa giác F, nghĩa là
trên đa giác F không còn một khoảng nào có thể đặt gọn được một hình
tròn có đường kính bằng 1 nữa.
Ta lấy tâm của mỗi hình tròn này làm tâm của một hình tròn mới
đường kính bằng 2, thì tập tất cả các hình tròn này sẽ phủ kín hình F.
Giả sử ngược lại, có điểm O nào đó thuộc F, nhưng không nằm trên
một hình tròn nào trong các hình tròn ta vừa vẽ. Khi đó O cách tất cả
các tâm của hình tròn bán kính bằng
1
2
một khoảng lớn hơn 1, nên khi
lấy O làm tâm ta có thể đặt thêm 1 hình tròn đường kính bằng 1 vào
hình F, mà nó không trờm lên bất kỳ một hình tròn bán kính
1
2
nào.
33
Như vậy, ta có thể đặt không phải a, mà ít nhất a + 1 hình tròn đường
kính đơn vị vào hình F, mà chúng vẫn rời nhau. Như vậy ta đã đi tới
mâu thuẫn với giả thiết: a là số lớn nhất các hình tròn rời nhau có đường
kính bằng 1 và tâm nằm trong đa giác F. Từ đó suy ra a ≥ b.
Chú ý.
1) Nếu F là hình vuông cạnh bằng
1
2
, thì a = b. Khi đó mỗi loại có
một hình tròn, tâm của hai hình tròn này chung nhau và chung với tâm
hình vuông.
2) Nếu F là hình vuông cạnh bằng 1, thì ta cần phủ bằng một hình
tròn có bán kính bằng 1, tâm trùng với tâm của hình vuông nhưng lại
có thể vẽ được tâm của 4 hình tròn bán kính bằng
1
2
với tâm ở 4 đỉnh.
Vậy khi đó ta có
a = 4 > 1 = b
Hình 2.4.12
Bài toán 2.4.11. Gọi a là số lớn nhất các hình tròn rời nhau (không có
điểm chung), có bán kính bằng 1 và tâm thuộc hình F, còn b là số nhỏ
nhất các hình tròn bán kính bằng 1, mà các hình tròn này có thể phủ kín
hình F. Hỏi a ≤ b hay a > b?
Chứng minh. Bằng phản chứng ta có thể khẳng định được a ≤ b.
Giả sử a > b
Gọi C1, C2, ..., Ca là tập hợp nhiều nhất các hình tròn rời nhau có tâm
tại O1, O2, ..., Oa thuộc hình F và có bán kính bằng 1.
Gọi D1, D2, ..., Db là tập ít nhất các hình tròn bán kính bằng 1 và phủ
kín hình F.
Ta cắt các hình tròn D1, D2, ..., Db bằng giấy xanh rồi đem phủ kín
lên hình F. Sau đó, ta cắt các hình tròn C1, C2, ..., Ca bằng giấy đỏ rồi
đặt chúng lên hình F, sao cho chúng rời nhau.
34
Vì a > b, nên số hình tròn C1, C2, ..., Ca có tâm nằm nằm trên hình
F nhiều hơn số hình tròn D1, D2, ..., Db có ít nhất một phần nằm trên
hình F (Vì b là số ít nhất các hình tròn phủ hình F, nên những hình tròn
không có phần phủ F đã bị loại bỏ).
Bởi vậy sẽ có hình tròn Di(1 ≤ i ≤ b) nào đó chứa tâm của hai hình
tròn Cj, Ck(k = j) nào đó. Khi đó do các hình tròn D1, D2, ..., Db đều có
bán kính bằng 1, nên khoảng cách giữa Oj và Ok nhỏ hơn 2 (OjOk < 2).
Do đó hai hình tròn Cj, Ck không rời nhau. Ta đi tới mâu thuẫn với tính
chất rời nhau của các hình tròn C1, C2, ..., Ca. Bởi vậy phải loại bỏ giả
thiết a > b mà kết luận a ≤ b.
Bài toán 2.4.12. Cho 7 đoạn thẳng. Độ dài mỗi đoạn lớn hơn 10cm,
bé hơn 100cm. Chứng minh rằng luôn luôn tồn tại ba đoạn thẳng là các
cạnh của một tam giác.
Chứng minh.
Dùng ai(1 ≤ i ≤ 7) để ký hiệu 7 đoạn thẳng đã cho và giả sử độ dài của
chúng được xếp theo thứ tự:
10cm < a1 ≤ a2 ≤ a3 ≤ a4 ≤ a5 ≤ a6 ≤ a7 < 100cm
Ta biết rằng ba đoạn thẳng là cạnh của một tam giác khi và chỉ khi tổng
độ dài của hai đoạn tùy ý lớn hơn độ dài đoạn còn lại và trị tuyệt đối
của hiệu độ dài hai cạnh tùy ý không lớn hơn độ dài cạnh còn lại.
Phản chứng. Giả sử không có ba đoạn thẳng nào trong các đoạn
thẳng đã cho lập thành một tam giác. Khi đó bất kỳ bộ ba i, j, k nào
mà 1 ≤ i; j, k ≤ 7, thì ai + aj ≤ ak, nên có:
a1 + a2 ≤ a3, a2 + a3 ≤ a4, a3 + a4 ≤ a5, a4 + a5 ≤ a6, a5 + a6 ≤ a7
Bởi vậy:
100cm ≥ a7 ≥ a5 + a6 ≥ 2a5 + a4 ≥ 2(a3 + a4) + a4 = 2a3 + 3a4 ≥
≥ 2a3 + 3(a2 + a3) = 5a3 + 3a2 ≥ 5(a2 + a1) + 3a2 = 8a2 + 5a1 ≥
≥ 8a1 + 5a1 = 13a1 ≥ 13.10cm = 130cm
Ta đi tới mâu thuẫn, nên phải tồn tại ít nhất một bộ ba p, q, r ∈ N+
(1 ≤
p < q < r ≤ 7) để ap + aq > ar.
Từ đó ap + ar > ar > aq, aq + ar > ar > ap.
Khi đó bộ ba ap, aq, ar lập thành một tam giác.
Khẳng định được chứng minh.
35
Bài toán 2.4.13. Chứng minh rằng một đa giác lồi có 2015 cạnh không
thể chia ra thành hình bình hành.
Chứng minh.
Phản chứng. Ta giả sử ngược lại, có một đa giác lồi 2015 cạnh có thể
chia cắt thành những hình bình hành. Khi đó mỗi cặp cạnh của đa giác
sẽ là đôi một song song cùng nhau (ba cạnh không thể song song cùng
nhau vì đa giác là lồi). Nhưng trong trường hợp như vậy, đa giác sẽ có
số cạnh là số chẵn. Giả thiết bài toán cho cạnh của đa giác lồi là 2015
(là một số lẻ) nên không thỏa mãn. Suy ra điều ta giả sử là sai.
Vậy đã giác lồi có 2015 cạnh không thể chia ra thành những hình
bình hành.
Bài toán 2.4.14. Bảy đa giác có diện tích bằng nhau và bằng 1 nằm
trong một hình vuông có độ dài cạnh là 2. Chứng minh rằng có ít nhất
hai đa giác cắt nhau với diện tích phần chung không nhỏ hơn
1
7
.
Chứng minh.
Ta kí hiệu:
Pi, i = 1, 7 là bảy đa giác có diện tích bằng 1; V là hình vuông có diện
tích bằng 4 chứa bảy đa giác trên.
Phản chứng. Giả sử mệnh đề: "Ít nhất có hai đa giác cắt nhau với
diện tích chung không nhỏ hơn
1
7
" là sai.
Kí hiệu SA là diện tích của hình A.
Ta có
SPi∩Pj
<
1
7
∀i, j : 1 ≤ i < j ≤ 7.
Từ đây suy ra
SPi∪Pj
= SP1
+ SP2
− SP1∩P2
> 1 + 1 −
1
7
.
Hơn nữa
SP1∪P2∪P3
= SP1∪P2
+ SP3
− S(P1∪P2)∩P3
= SP1∪P2
+ SP3
− S(P1∩P3)∪(P2∩P3)
≥ SP1∪P2
+ SP3
− SP1∩P3
− SP2∩P3
> (2 −
1
7
) + 1 − (
1
7
+
1
7
) = 3 − (
1
7
+
2
7
).
36
Lặp lại cách làm trên ta nhận được
SP1∪P2∪P3∪P4∪P5∪P6∪P7
> 7 − (
1
7
+
2
7
+ ... +
6
7
) = 4.
Điều này không thể xảy ra vì các đa giác nằm trong hình vuông V có
diện tích bằng 4.
Vậy phải có ít nhất hai hình đa giác cắt nhau mà phần chung diện
tích lớn hớn hoặc bằng
1
7
.
2.5 Bài tập tự giải
Bài toán 2.5.1. Chứng minh rằng từ 8 số tự nhiên tùy ý luôn luôn chọn
ra được 2 số, mà hiệu của chúng chia hết cho 7.
Hướng dẫn. Chia 8 số tự nhiên tùy ý đã chọn ra cho 7 được 8 số dư
tương ứng, nhưng chỉ thuộc 7 loại: 0, 1, 2, 4, 5, 6 nên phải có ít nhất 2
số dư bằng nhau. Khi đó hiệu của 2 số tương ứng chia hết cho 7.
Bài toán 2.5.2. Chứng minh với mọi số nguyên a, b, c luôn tìm được số
nguyên dương n, sao cho số f(n) = n3
+ an2
+ bn + c không phải là số
chính phương.
Hướng dẫn. Nhận xét rằng khi chia cho 4, số chính phương chỉ có thể
dư 0 hoặc dư 1.
Phản chứng. Giả sử f(n) là số chính phương.
Khi đó
f(4) − f(2) ≡ 2b(mod 4)
Mà 2b là số chẵn, theo nhận xét thì f(4) − f(2) chỉ có thể đồng dư với
0, 1, −1 theo modun 4, nên suy ra 2b ≡ 0(mod 4)
f(3) − f(3) ≡ (2b + 2)(mod 4)
Tương tự trên ta cũng có 2b + 2 ≡ 0(mod 4) nên suy ra 2 ≡ 0(mod 4)
(vô lý). Vậy điều giả sử là sai.
Bài toán 2.5.3. Chứng minh không số hữu tỷ nào là nghiệm của phương
trình
x3
+ x + 1 = 0.
37
Hướng dẫn.
Phản chứng. Giả sử ngược lại, có số hữu tỷ
p
q
(q = 0, (p, q) = 1, p, q ∈ Z)
là nghiệm của phương trình. Khi đó
p3
q3
+
p
q
+ 1 = 0 ⇒ p3
+ p.q2
+ q3
= 0
Xét các trường hợp p, q cùng lẻ, p, q cùng chẵn, p chẵn, q lẻ hoặc p lẻ, q
chẵn để chỉ ra vô lý của điều giả sử.
Bài toán 2.5.4. Chứng minh rằng từ n số tự nhiên bất kỳ a1, a2, ..., an
đều có thể tách ra một nhóm, gồm ít nhất một số, mà tổng tất cả các số
thuộc nhóm này chia hết cho n.
Hướng dẫn.Xét n tổng
S1 = a1, S2 = a1 + a2, ..., Sn = a1 + a2 + ... + an (1)
1) Nếu một trong những tổng thuộc dãy (1) chia hết cho n, thì khẳng
định được chứng minh.
2) Các tổng thuộc dãy (1) đều không chia hết cho n. Khi đó chia tất cả
các số thuộc dãy (1) cho n, được n số dư, nhưng chỉ thuộc tối đa n − 1
loại. Khi đó theo nguyên lý phản chứng, phải có ít nhất 2 số dư bằng
nhau. Giả sử j > i và Si, Sj có cùng số dư khi chia cho n. Khi đó:
A = Sj − Si = ai+1 + ai+2 + ... + aj
chia hết cho n. Vậy khẳng định được chứng minh.
Bài toán 2.5.5. Cho 9 đường thẳng. Mỗi đường đều chia hình vuông
thành 2 tứ giác với tỷ số diện tích bằng
2
3
. Chứng minh rằng có ít nhất
3 trong 9 đường thẳng này phải đi qua 1 điểm.
Hướng dẫn. Mỗi đường trung bình của hình vuông có 2 điểm chia nó
theo tỷ lệ
2
3
. Do đó trong hình vuông có 4 điểm chia đường trung bình
theo tỷ lệ
2
3
. Mặt khác đường thẳng bất kỳ cắt hình vuông thành 2 hình
thang hoặc 2 hình chữ nhật với tỷ lệ diện tích
2
3
.
38
Chương 3
Phương pháp suy luận
3.1 Vài nét về phương pháp suy luận
Các bài toán không mẫu mực (không có cách giải nhất định),
thường có nhiều cách giải khác nhau, trong đó có phương pháp suy
luận.
Phương pháp suy luận đã có từ xa xưa và để giải các bài toán logic
người ta chỉ có duy nhất phương pháp này (sau này mới có thêm các
phương pháp khác). Các bài toán không mẫu mực đa dạng về đề tài,
phong phú về chủng loại đòi hỏi chúng ta phải biết suy luận đúng đắn,
chặt chẽ trên cơ sở vận dụng những kiến thức cơ bản và kinh nghiệm
sống của mình. Vì vậy, cần phải luyện tập óc quan sát, cách lập luận,
cách xem xét các khả năng có thể xảy ra của một sự kiện và vận dụng
những kiến thức đã học vào các tình huống muôn hình muôn vẻ trong
cuộc sống hàng ngày.
Đôi khi để giải những bài toán loại này, chỉ cần những kiến thức toán
học đơn giản nhưng lại đòi hỏi khả năng chọn lọc trường hợp, suy luận
chặt chẽ, rõ ràng. Sự phát triển của toán học, chẳng hạn giải tích tổ hợp,
phương pháp quy nạp, phản chứng góp phần phong phú thêm phương
pháp suy luận logic trực tiếp. Và nhờ những kiến thức toán học này
người ta có thể giải các bài toán không mẫu mực một cách nhanh hơn,
tốt hơn và chặt chẽ hơn.
Điều cơ bản của phương pháp này là thông qua việc phân tích các
điều kiện của bài toán cần tìm ra mối quan hệ logic giữa các mệnh đề.Sau
đây là một vài ví dụ về vận dụng phương pháp suy luận trực tiếp.
39
3.2 Các ví dụ về vận dụng phương pháp suy luận.
Bài toán 3.2.1. (Tạp chí Toán Học & Tuổi Trẻ số 379, tháng 1 - 2009)
Hãy tưởng tượng bạn đang tham gia trò chơi "Chiếc nón kỳ diệu" trên
truyền hình và bạn đang có cơ hội nhận được một phần quà có giá trị.
Phần quà này nằm ở 1 trong 3 chiếc hộp (hai chiếc hộp còn lại rỗng).
Giả sử bạn đã chọn chiếc hộp nào đó nhưng chưa mở ra, lúc này người
dẫn chương trình (là người biết rõ hộp nào chứa phần quà) sẽ mở 1 chiếc
hộp rỗng trong hai chiếc còn lại ra (nếu 2 hộp đều rỗng thì chọn hộp bất
kỳ). Sau đó anh ta hỏi bạn có đổi chiếc hộp vừa chọn lấy chiếc hộp còn
lại chưa mở không?
Hỏi rằng nếu bạn đồng ý thì xác suất chọn đúng chiếc hộp có quà là bao
nhiêu? Theo tôi nghĩ khi người dẫn chương trình bỏ đi một chiếc hộp
rỗng thì sẽ còn lại 2 chiếc hộp. Nên nếu đồng ý đổi thì xác suất chọn
đúng hộp có quà tặng là
1
2
. Bạn có nghĩ như tôi không?
Chứng minh. Rõ ràng xác suất chọn đúng hộp có quà tặng bằng
1
2
là
không đúng.
Có thể thấy rằng xác suất để chiếc hộp chứa quà mà người chơi chọn
ban đầu (khi có 3 hộp) là
1
3
. Xác suất để một trong hai chiếc hộp còn lại
có quà là
2
3
. Khi người dẫn chương trình loại đi một chiếc hộp rỗng thì
chiếc hộp còn lại (mà người chơi không chọn) có xác suất chứa quà là
2
3
.
Do đó xác suất có chứa quà trong hai chiếc hộp còn lại là khác nhau. Vì
vậy nếu người chơi đồng ý đổi chiếc hộp đã chọn với chiếc hộp còn lại
thì xác suất chọn đúng hộp quà của người chơi sẽ tăng lên (=
2
3
). Như
vậy nếu người dẫn chương trình tạo một cơ hội như trên thì người chơi
không nên bỏ lỡ.
Bài toán 3.2.2. Có 12 đội bóng tham gia tranh giải. Mỗi đội phải đấu
với nhau một trận: Thắng được 1 điểm, hòa được
1
2
điểm, thua không
được điểm. Biết rằng số điểm của đội đứng thứ 2 trong giải bằng tổng
số điểm của 5 đội đứng cuối cùng trong giải và số điểm của đội đứng
thứ 9 không nhỏ hơn tổng số điểm của 3 đội đứng cuối cùng trong giải.
Đồng thời không có ba đội nào cùng bằng điểm nhau. Hỏi kết quả của
40
trận giữa đội đứng thứ 8 và đội đứng thứ 9 như thế nào?
Chứng minh. Đội thứ nhất đấu 11 trận với 11 đội còn lại và số điểm
không vượt quá 11 điểm. Do đó, đội thứ 2 có số điểm không quá 10,5
điểm.
Nếu đội thứ 2 được 10,5 điểm, thì đội thứ hai hòa 1 trận và thắng 10
trận, nghĩa là trận đấu giữa đội xếp thứ nhất và đội xếp thứ 2 hòa nhau
vì thế đội xếp thứ nhất chỉ đạt số điểm nhiều nhất là 10,5 điểm, nhưng
như vậy có hai đội đạt cùng một điểm (trái với giả thiết). Vậy số điểm
của đội thứ 2 không vượt quá 10 điểm (1).
Năm đội cuối cùng đã đấu với nhau số trận là
5 × 4
2
= 10 trận và đạt
tổng số điểm là 10 điểm, nên tổng số điểm của 5 đội xếp cuối cùng phải
lớn hơn hoặc bằng 10 điểm (2).
Vì đội thứ hai đạt số điểm bằng số điểm 5 đội xếp cuối bảng, nên từ
(1) và (2) ta có số điểm của 5 đội xếp cuối cùng là 10 điểm. Nghĩa là các
đội này chỉ đạt điểm khi họ đấu với nhau còn họ thua tất cả các đội còn
lại, nghĩa là mỗi đội trong số 5 đội này đều thua các đội còn lại.
Kí hiệu A, B, C, D, E lần lượt là số điểm của đội thứ 8, thứ 9, thứ
10, thứ 11, thứ 12. Ta đã biết A + B + C + D + E = 10. Nếu ba đội đấu
với nhau thì bao giờ cũng có tổng số điểm là 3, nên nếu kể cả điểm của
ba đội đó đạt được khi thi đấu với 2 dội còn lại thì ta có: C +D +E ≥ 3
(3).
Theo giả thiết B ≥ C + D + E (4), suy ra B ≥ 3. Đội đứng thứ 8 có
số điểm A ≤ 4. Từ đó suy ra A = 4 (Vì nếu A = 3 × 5 thì B = 3 × 5),
B = C + D + E = 3.
Vậy đội đứng thứ 8 thi đấu 4 trận với 4 đội xếp cuối cùng, đạt 4 điểm
nên đội đứng thứ 8 đã thắng đội đứng thứ 9.
Bài toán 3.2.3. (Tạp chí Toán Học & Tuổi Trẻ số 388, tháng 10 năm
2009) Trong một cuộc họp gồm 56 đại biểu. Biết rằng mỗi đại biểu đều
quen biết với không ít hơn 8 đại biểu khác (quy ước rằng A quen B thì B
quen A). Chứng minh rằng ban tổ chức có thể sắp xếp ít nhất một bàn
gồm 4 đại biểu cùng bàn với nhau sao cho mỗi đại biểu quen ít nhất 2
trong số 3 đại biểu còn lại.
Chứng minh. Xét 1 đại biểu A bất kỳ. Kí hiệu A1, A2, ..., An là tất cả
các đại biểu quen với A. Theo giả thiết n ≥ 8 và mỗi Ai(i = 1, n) ngoài
A ra đều quen ít nhất 7 đại biểu khác. Do đó nếu với mỗi i = 1, n ta
41
liệt kê tất cả các đại biểu khác A và quen với A1 thì tổng số đại biểu
liệt kê ra được kể cả lặp không ít hơn 7n ≥ 56. Mà ngoài A chỉ còn 55
đại biểu khác nhau nên suy ra có ít nhất một đại biểu được liệt kê ít
nhất hai lần. Nghĩa là phải có hai đại biểu Ai, Aj(i = j, j ∈ {1, 2, ..., n})
cũng quen với đại biểu B, khác A (có thể B ∈ {A1, A2, ..., An}). Hiển
nhiên khi xếp A1, Ai, Aj, B cùng bàn ta sẽ được chiếc bàn thỏa mãn đề
bài.
Bài toán 3.2.4. (Tạp chí Toán Học & Tuổi Trẻ, số 5 năm 1992) Trong
dịp hè Cường, Hùng, Dũng tham gia câu lạc bộ thể dục thể thao, luyện
tập 6 môn bơi, bóng bàn, bóng đá, võ dân tộc, cờ quốc tế và cầu lông.Mỗi
người luyện tập 2 môn và không có 2 người nào luyện tập cùng một môn.
Hãy xác định xem ai luyện tập những môn nào? Biết rằng
1) Bạn luyện tập cờ quốc tế và bạn luyện tập bóng đá là đôi bạn thân từ
nhỏ.
2) Bạn Cường cao to nhất bọn.
3) Bạn Dũng, bạn tạp bóng bàn và bạn tập bóng đá thường rủ nhau đến
câu lạc bộ.
4) So với bạn học võ dân tộc, bạn chơi bóng bàn thấp hơn chút ít.
5) Ba bạn: bạn tập bơi, bạn học võ dân tộc và bạn Cường cùng tham dự
kỳ thi học sinh giỏi toán năm vừa qua.
Chứng minh. Từ giả thiết (2) và (4) ta suy ra bạn Cường không tập
bóng bàn.
Từ (3) ta có Dũng cũng không tập bóng bàn.
Vậy bạn Hùng tập bóng bàn (6) và bạn Cường tập bóng đá (7)
Từ giả thiết (4) và (6) suy ra Hùng không tập võ dân tộc.
Từ (5) ta có Cường không tập võ dân tộc. Vậy Dũng tập võ dân tộc (8).
Từ (5) và (8) suy ra Hùng tập bơi (9).
Từ (1) và (7) suy ra Cường không học chơi cờ quốc tế.
Từ (6) và (9) ta cũng có Hùng không học chơi cờ quốc tế. Vậy Dũng
chơi cờ quốc tế và Cường tập cầu lông.
Kết luận: Cường tập cầu lông và bóng đá. Dũng học võ dân tộc và chơi
cờ quốc tế. Hùng tập bóng bàn và tập bơi.
Bài toán 3.2.5. Có một bảng gồm 3×3 ô vuông và 9 cái thẻ kích thước
bằng một ô vuông có viết các số nào đó. Hai người chơi lần lượt đặt từng
thẻ một lên các ô của bảng. Sau khi đặt hết các thẻ phủ kín bảng, người
thứ nhất (người đặt thẻ đầu tiên) tính tổng của 6 ô đứng ở dòng trên
42
cùng và dưới cùng, người thứ 2 tính tổng của 6 ô đứng ở cột bên trái và
cột bên phải. Ai có tổng của mình lớn hơn là người thắng cuộc. Chứng
minh rằng nếu người thứ nhất chơi đúng thì người thứ hai sẽ không thể
thắng cuộc.
Chứng minh. Rõ ràng kết quả của trò chơi chỉ phụ thuộc vào các số đứng
ở ô số 1, ô số 2, ô số 3, ô số 4. Người thứ nhất thua nếu tổng các chữ số
ở ô số 1 và ô số 3 nhỏ thua tổng các chữ số ở ô số 2 và ô số 4. Ta hãy đề
ra chiến thuật người thứ nhất chơi không thua cuộc. Giả sử các số ghi
trên các thẻ là a1, a2, ..., a9 thỏa mãn a1 ≤ a2 ≤ a3 ≤ ... ≤ a9.
Hình 3.2.5
- Nếu a1 + a9 ≥ a2 + a8, khi đó người thứ nhất đặt a9 ở ô số 1 và lần
thứ hai đặt a2 (hoặc a1) vào ô số 2 (hoặc ô số 4). Rõ ràng tổng ở các ô
số 1 và số 3 sẽ không bé thua a1 + a9, còn tổng các chữ số của ô số 2 và
ô số 4 không vượt quá a2 + a8.
- Nếu a1 + a9 < a2 + a8, khi đó trong lần đầu đi đầu tiên người thứ
nhất đặt a1 vào ô số 2 và lần đi thứ 2 đặt a8 (hoặc a9) vào ô số 1 hoặc ô
số 3. Rõ ràng tổng các số ở ô số 1 và ô số 3 không nhỏ thua a2 + a8 còn
tổng các số ở ô số 2 và ô số 4 không vượt quá a1 + a9.
Như vậy trong cả hai trường hợp thì người thứ nhất không thua, nghĩa
là người thứ 2 không thể thắng cuộc. Tất nhiên kết quả này không phụ
thuộc vào số viết trên thẻ.
Bài toán 3.2.6. Có 3 trường học, mỗi trường học có n học sinh. Mỗi
học sinh bất kỳ có tổng số người quen từ hai trường kia là n + 1. Chứng
minh rằng có thể chọn ở mỗi trường một học sinh, sao cho ba học sinh
này quen lẫn nhau.
Chứng minh. Trong số tất cả 3n học sinh chọn ra một học sinh có số
người quen lớn nhất (k) từ một rong hai trường học khác (hay một từ
các học sinh như vậy). Giả sử đó là học sinh A từ trường số 1 mà học
sinh này quen k học sinh, ví dụ ở trường số 2. Khi đó A quen với n+1−k
học sinh ở trường số 3, ngoài ra n + 1 − k ≥ 1 vì k ≤ n.
43
Xét học sinh B của trường số 3 quen với A. Nếu B quen với một học
sinh C trong số k người quen của A ở trường số 2, thì các học sinh A,
B, C tạo thành bộ ba cần tìm. Nếu B không quen C trong số k người
quen của A ở trường số 2 thì trong trường hợp này B sẽ quen với không
nhiều hơn n − k học sinh ở trường số 2. Nghĩa là trong trường hợp số 1,
B quen với không ít (n + 1) − (n − k) = k + 1 học sinh. Điều này trái
với việc chọn số k. Suy ra khẳng định được chứng minh.
Bài toán 3.2.7. Ai cao ai thấp?
Có 200 học sinh xếp hàng 10. Lần thứ nhất, theo mỗi hàng dọc, người ta
chọn ra một học sinh cao nhất. Sau đó trong số 10 học sinh được chọn
ấy người ta lại chọn ra một học sinh thấp nhất.
Lần thứ hai, theo mỗi hàng ngang, người ta chọn ra một học sinh thấp
nhất. Sau đó trong số 20 học sinh được chọn ấy người ta lại chọn ra 1
học sinh cao nhất. Ai trong số 2 học sinh được chọn ra cao hơn, biết rằng
sau lần chọn thứ nhất các học sinh lại về vị trí cũ.
Chứng minh. Gọi A là học sinh được chọn trong lần chọn thứ nhất và
B là học sinh được chọn trong lần chọn thứ hai. Gọi C là học sinh đứng
cùng hàng dọc với A và cùng hàng ngang với B (như hình vẽ).
Vì A là học sinh cao nhất trong hàng dọc của mình nên A cao hơn
C: h(A) ≥ h(C) (1). Vì B là học sinh thấp nhất trong hàng ngang của
mình nên B thấp hơn C:h(B) ≤ h(C) (2), với h() là kí hiệu chiều cao
của mỗi học sinh. Từ (1) và (2) suy ra h(A) ≥ h(B), nghĩa là học sinh
được chọn lần đầu không thấp hơn học sinh được chọn lần hai. Dấu "="
xảy ra ở cả hai đẳng thức (1) và (2) khi và chỉ khi A cao bằng B.
Bài toán 3.2.8. (Tạp chí Toán học & tuổi trẻ, tháng 10 năm 2007) Ai
bắng trúng vòng mười?
Ba anh bộ đội Mạnh, Hùng, Dũng tập bắn bia. Mỗi người bắn ba phát
vào cùng một bia. Kết quả như sau: Tất cả các vòng bia từ 2 đến 10 đều
44
được bắn trúng và ba người đều có tổng số điểm bằng nhau. Biết rằng
Hùng bắn trúng vòng 4, Dũng bắn trứng vòng 7. Các bạn hãy xem xét
xem ai đã bắn trúng vòng 10 và mỗi người đã bắn trúng những vòng nào?
Chứng minh. Vì có 9 phát bắn và cả 9 vòng bia đều được bắn trúng nên
mỗi vòng bia từ 2 đến 10 chỉ có một phát trúng. Như vậy tổng số điểm
của cả ba người là
2 + 3 + 4 + 5 + 6 + 7 + 8 + 9 + 10 = 54
Số điểm mỗi người đạt được là 54 : 3 = 18.
Vì Hùng bắn trúng vòng 4 nên Hùng có thể bắn trúng vòng 5, 4, 9 hoặc
4, 6 ,8.
Vì Dũng bắn trúng vòng 7 nên Dũng có thể bắn trúng vòng 3, 8, 7 hoặc
7, 2 ,9.
Vậy Mạnh bắn trúng vòng 10 và kết quả có thể theo các khả năng như
bảng sau.
Bài toán 3.2.9. Nhập học vào lớp 10 năm nay có 50 em biết nhạc, 50
em biết đá bóng và 50 em biết đánh bóng bàn (một em có thể biết nhiều
môn). Hãy chia các em học sinh khối 10 thành 5 lớp (có thể sĩ số của
các lớp khác nhau) sao cho mỗi lớp có đúng 10 em biết nhạc, 10 em biết
đá bóng và 10 em biết đánh bóng bàn.
Chứng minh.
1. Chia 50 em biết nhạc thành 50 nhóm A1, A2, ..., A49, A50
2. Bổ sung người biết đá bóng (nếu Ai có em biết đá bóng thì không
bổ sung nữa) để được các nhóm B1, B2, ..., B49, B50 (Bi có đúng một em
biết nhạc, một em biết đá bóng).
3. Bổ sung các em biết bóng bàn.
Ta bổ sung các em biết đánh bóng bàn chưa tham gia nhóm Bi vào
các nhóm Bi sao cho mỗi nhóm nhận được có không quá 2 m biết đánh
45
bóng bàn. Sau 3 bước ta chọn được các nhóm C1, C2, ..., C49, C50 (mỗi
nhóm Ci, 1 ≤ i ≤ 50 có đúng một em biết nhạc, một em biết đá bóng
và không quá 2 em biết đánh bóng bàn.)
Gọi số nhóm Ci có 2 em biết đánh bóng bàn là x.
Gọi số nhóm Ci không có em nào biết đánh bóng bàn là y.
Gọi số nhóm Ci có 1 em biết đánh bóng bàn là z.
Suy ra số nhóm Ci là x + y + z = 50
Số em biết đánh bóng bàn là 2x + 0y + z = 50
Suy ra hệ:
x + y + z = 50
2x + z = 50
⇒
x = y
z chẵn
Vì số nhóm không có em nào biết đánh bóng bàn bằng số nhóm có hai
em biết đánh bóng bàn và số nhóm có một em biết đánh bóng bàn là số
chẵn nên ta nhập một nhóm không em nào biết đánh bóng bàn với một
nhóm có hai em biết đánh bóng bàn. Nhập hai nhóm có một em biết
đánh bóng bàn lại với nhau.
Suy ra có 25 nhóm như vậy và ký hiệu là: D1, D2, ..., D25.
Mỗi nhóm Di có đúng hai em biết đánh bóng bàn, hai em biết nhạc
và hai em biết đá bóng.
Ta lại nhập 5 nhóm Di lại thành một nhóm rồi bổ sung các em còn
lại vào các nhóm này thì sẽ được 5 lớp, mỗi lớp có đúng 10 em biết nhạc,
10 em biết bóng bàn và 10 em biết đá bóng.
3.3 Bài tập tự giải
Bài toán 3.3.1. Điều mâu thuẫn ở đâu?
Trong một tòa nhà chỉ có những cặp vợ chồng và những con nhỏ chưa
lập gia đình. Ban điều tra dân số yêu cầu báo cáo về số người sống trong
tòa nhà, đại diện là một anh thợ thích đùa báo cáo như sau:
Sống trong tòa nhà bố mẹ nhiều hơn con cái. Mỗi con trai đều có 1 chị
hay em gái. Số con trai nhiều hơn số con gái. Mỗi cặp vợ chồng đều có
con.
Người ta không thể chấp nhận được báo cáo đó dù là đùa vui vì trong đó
xảy ra mâu thuẫn.
Bạn hãy chỉ ra điều mâu thuẫn trong báo cáo trên.
Hướng dẫn.
Vì mỗi gia đình đều có con, mỗi con trai đều có một chị hoặc em gái.
46
Suy ra số con gái ít nhất bằng số gia đình.
Mặt khác số con trai nhiều hơn số con gái nên tổng số con nhiều hơn
hai lần số gia đình, hay nhiều hơn số bố mẹ, điều này cho ta thấy mâu
thuẫn trong báo cáo của anh thợ thích đùa ở câu đầu tiên:"bố mẹ nhiều
hơn con cái" với các câu tiếp theo.
Bài toán 3.3.2. Có xếp được không?
Một bàn cờ đô - mi - nô có 64 ô vuông (bàn cờ vuông, mỗi cạnh 8 ô
vuông). Các quân đô - mi - nô hình chữ nhật, có kích thước sao cho có
thể chồng vừa khít lên hai ô vuông liền nhau. Như vậy có thể dễ dàng xếp
được 32 quân cho vừa khít cả bàn cờ. Có thể xếp được 31 quân sao cho
vừa khít 62 ô còn lại không? Nếu được thì xếp như thế nào? Nếu không,
hãy chứng minh là không xếp được.
Hướng dẫn.
Trả lời: Không.
Với bất kỳ cách xếp đặt nào, mỗi quân cờ sẽ phủ kín hai ô khác màu, vì
vậy 30 quân cờ sẽ phủ kín 30 ô đen và 30 ô trắng. Còn lại 2 ô cùng một
màu, do đó hai ô xa nhau sẽ hoặc đen hoặc trắng.
Bài toán 3.3.3. Thứ tự xếp hàng.
Có 5 người là A, B, C, D, E xếp thành hàng mua vé xem phim. Biết
rằng A mua được vé sau E, trước B, C và D không đứng kề sau. D không
đứng kề với E, A và C. Tìm thứ tự xếp hàng của 5 người đó.
Hướng dẫn.
Thứ tự sắp xếp: E, A, C, B, D.
Vì D không đứng kề với E, A và C nên D đứng kề với B. Do đó D đứng
đầu hàng hoặc cuối hàng, nhưng A mua vé trước D. Do vậy D đứng cuối
hàng và B đứng thứ 4. Mà E đứng trước A, C và E không đứng kề nhau
nên C đứng thứ 3. A đứng thứ 2 còn E đứng thứ nhất.
Bài toán 3.3.4. Bao nhiêu đại biểu
Trong một hội nghị có 500 đại biểu tham dự, mỗi đại biểu có thể sử dụng
một trong ba thứ tiếng: Nga, Anh hoặc Pháp. Theo thống kê của ban tổ
chức, có 60 đại biểu chỉ nói được một trong ba thứ tiếng, 180 đại biểu
nói được hai thứ tiếng Anh và Pháp, 150 đại biểu nói được cả hai thứ
tiếng Anh và Nga, 170 đại biểu nói được cả tiếng Nga và tiếng Pháp.
Hỏi có bao nhiêu đại biểu nói được cả ba thứ tiếng trên?
47
Hướng dẫn.
Có 60 đại biểu nói được cả ba thứ tiếng.
Bài toán 3.3.5. Ai đi xem xiếc?
Gia đình Châu có 5 người: bố, mẹ, Châu cùng hai em Anh và Dũng.
Sáng chủ nhật cả nhà thích đi xem xiếc, nhưng chỉ mua được hai vé. Mọi
người trong gia đình đề xuất 5 ý kiến
1. Châu và Dũng đi.
2. Bố và mẹ đi.
3. Bố và Anh đi.
4. Mẹ và Châu đi.
5. Bố và Châu đi.
Cuối cùng mọi người đồng ý với đề nghị của Dũng vì theo đề nghị đó
mỗi đề nghị của bốn người còn lại đều được thỏa mãn một phần và bác
bỏ một phần. Bạn hãy cho biết ai đi xem xiếc hôm đó?
Hướng dẫn.
Nếu chọn đề nghị thứ nhất thì đề nghị thứ 2 không thỏa mãn. Điều này
trái với đề bài. Tương tự cũng không thể chọn đề nghị thứ hai, thứ ba
và thứ tư. Nếu chọn đề nghị thứ 5 thì mỗi ý kiến được thỏa mãn 1 phần
và bác bỏ 1 phần. Vậy bố và Châu đi xem xiếc.
Bài toán 3.3.6. Người không chinh phục được cá sấu.
Cho trước các mệnh đề:
1. Trẻ sơ sinh có tư duy logic.
2. Chúng ta không dám coi thường những người chinh phục được cá sấu.
3. Chúng ta coi thường những người chưa có tư duy logic.
Từ các mệnh đề trên bạn hãy suy ra kết luận: Trẻ sơ sinh không chinh
phục được cá sấu.
Hướng dẫn.
Giả sử "Trẻ sơ sinh chinh phục được cá sấu". Từ (2) suy ra "chúng ta
không dám coi thường trẻ sơ sinh". Từ (3) suy ra "Trẻ sơ sinh có tư duy
logic". Điều này mâu thuẫn với (1).
Bài toán 3.3.7. Các giáo sư dạy môn gì?
Ba giáo sư Kim, Quân, Hà dạy ba môn toán, lý, hóa. Trong ba mệnh đề:
1. Giáo sư Kim dạy môn hóa.
2. Giáo sư Quân không dạy môn hóa.
3. Giáo sư Hà không dạy môn lý.
chỉ có một mệnh đề đúng. Bạn hãy xác định mỗi giáo sư dạy môn gì?
48
Hướng dẫn.
Giả sử mệnh đề (1) đúng suy ra (2) đúng. Điều này vô lý, vậy mệnh đề
(1) là sai.
Nếu (2) đúng ⇒ Giáo sư Hà dạy hóa và (3) đúng. Điều này vô lý, vậy
mệnh đề (2) là sai. Vậy giáo sư Quân dạy hóa, giáo sư Hà dạy toán và
giáo sư Kim dạy lý.
49
Chương 4
Phương pháp bảng
4.1 Vài nét về phương pháp bảng
Nhiều bài toán logic có thể giải bằng cách lập bảng mô tả mối quan
hệ giữa các đối tượng được cho trong bài toán.
Đối với một số bài toán logic trong đó xuất hiện 2 hay nhiều tệp và
các cặp phần tử nói lên mối quan hệ giữa các tệp, người ta có thể thiết
lập một hay nhiều bảng, để mô tả mối quan hệ giữa các tệp.
Mỗi bảng này có hàng trên cùng ghi các phần tử của một tệp, còn
cột tận cùng bên trái ghi các phần tử thuộc tập kia và các vị trí trong
bảng ghi mã số quan hệ giữa các phần tử thuộc các tệp.
Căn cứ vào các điều kiện đã cho trong bài toán gạch bỏ đi những cặp
phần tử không thích hợp, từ đó đi đến lời giải của bài toán.
Giải bài toán logic bằng phương pháp bảng đôi khi vấp phải bảng cần
lập có chiều khá lớn hoặc phải kết hợp nhiều bảng mới đi đến kết quả.
Sau đây là một vài ví dụ vận dụng phương pháp bảng để giải bài toán
không mẫu mực.
4.2 Vận dụng phương pháp bảng để giải bài toán
không mẫu mực
Bài toán 4.2.1. Trong buổi học nữ công, ba bạn Cúc, Đào, Hồng làm
ba bông hoa: cúc, đào, hồng. Bạn làm hoa hồng nói với bạn Cúc:"Thế là
trong chúng ta không có ai làm loại hoa trùng với tên mình". Hãy xác
định tên hoa mà mỗi bạn đã làm.
Chứng minh.
Bài toán này có hai tệp đối tượng. Tệp thứ nhất gồm các bạn làm hoa,
50
tệp thứ 2 gồm các bông hoa được làm. Nó có thể giải bằng phương pháp
bảng.
1) Lập bảng.
Bảng cần lập gồm 4 hàng và 4 cột. Hàng đầu, từ cột thứ 2 ghi lần lượt
tên các bông hoa được làm, còn trên cột tận cùng bên trái từ hàng hai
ghi lần lượt các bạn tham gia làm hoa.
2) Điền mã số quan hệ vào các vị trí của bảng.
a. Căn cứ vào giả thiết: Mỗi bạn đều không làm hoa trùng với tên mình,
mà điền mã "không" vào các ô nằm trên đường chéo chính.
Bảng 4.2.1
b. Căn cứ vào câu "Bạn làm hoa hồng nói với bạn Cúc" suy ra bạn
Cúc không phải làm hoa hồng, mà ghi mã "không" vào ô nằm ở hàng
Cúc, cột hồng.
3) Loại bỏ vị trí không thỏa mãn quan hệ để nhận được lời giải. Trong
bảng trên cột cuối vị trí 1 và 3 bị gạch bỏ nên vị trí duy nhất còn lại là
vị trí 2 phải thỏa mãn quan hệ giữa người làm hoa và hoa được làm. Do
đó bạn Đào làm hoa hồng.
Vì trên hàng 2 (Đào) đã có vị trí thỏa mãn quan hệ thì toàn bộ hàng
này phải loại bỏ ra ngoài diện xét. Bởi vậy cột Cúc chỉ còn vị trí cuối
cùng trong diện xét. Bởi vậy nó phải thỏa mãn quan hệ giữa người làm
hoa và hoa được làm, nên bạn Hồng làm hoa Cúc.
Từ đó suy ra người còn lại là bạn Cúc phải làm hoa đào.
Vậy bạn Cúc làm hoa đào, bạn Đào làm hoa hồng và bạn Hồng làm hoa
cúc.
Bài toán 4.2.2. Trên bàn là 3 cuốn sách giáo khoa: Văn, Toán và Địa
Lí được bọc 3 màu khác nhau: xanh, đỏ, vàng. Cho biết cuốn bọc bìa màu
đỏ nằm giữa cuốn Văn và Địa Lí, cuốn Địa Lí và cuốn màu xanh mua
51
cùng một ngày. Bạn hãy xác định mỗi cuốn sách bọc bìa màu gì?
Chứng minh. Ta có bảng sau
Bảng 4.2.2
Theo đề bài "Cuốn màu đỏ đặt giữa 2 cuốn Văn và Địa Lý". Vậy
cuốn Văn và Địa Lý đều không bọc màu đỏ nên cuốn Toán sẽ bọc màu
đỏ. Ta ghi số 0 vào ô 4 và 6, đánh dấu "x" vào ô 5
Mặt khác "cuốn Địa Lí và cuốn màu xanh mua cùng ngày". Điều đó có
nghĩa là cuốn Địa Lí không bọc màu xanh. Ta ghi số 0 vào ô 3.
Nhìn vào cột 4 ta thấy cuốn Địa Lí không bọc màu xanh, cũng không
bọc màu đỏ. Vậy cuốn Địa Lí bọc màu vàng. Ta đánh dấu "x" vào ô số
9.
Nhìn vào cột 2 ta và ô 9 ta thấy cuốn Văn không bọc màu đỏ cũng không
bọc màu vàng. Vậy cuốn Văn bọc màu xanh, ta đánh dấu "x" vào ô 1.
Kết luận: Cuốn Văn bọc màu xanh, cuốn Toán bọc màu đỏ, cuốn Địa
Lí bọc màu vàng.
Bài toán 4.2.3. Bốn bạn Xuân, Hạ, Thu, Đông nhận được điểm của bài
kiểm tra cuối học kỳ. Bạn Lan cùng lớp muốn biết điểm của từng người.
Khi Lan hỏi thì được các bạn trả lời úp mở như sau:
Xuân nói: "Bạn Đông được 7, Hạ được 8, Thu được 9".
Hạ nói: "Bạn Đông được 10, Thu được 8, Xuân được 9".
Thu nói: "Cả 3 bạn đều được 7".
Đông nói: "Cả 3 bạn đều được 8".
Biết rằng không có 2 bạn nào được 2 bạn nói cùng đúng với số điểm của
mình và mỗi câu trả lời ở trên chỉ nói đúng số điểm của một người mà
thôi. Hãy tìm điểm của mỗi người.
Chứng minh. Ta ghi điểm của từng người trong các câu trả lời vào bảng
4.2.3.
52
Căn cứ vào điều kiện: Không có bạn nào được hai bạn nói đúng điểm
của mình, ta suy ra
Hạ không được điểm 8.
Thu không được điểm 8.
Đông không được điểm 7.
Bảng 4.2.3
Căn cứ vào điều kiện: Mỗi câu trả lời chỉ đúng với số điểm của một
người, ta suy ra: Xuân được 8, Thu được 9, Đông được 10, Hạ được
7.
Bài toán 4.2.4. Trong trại hè thiếu nhi quốc tế, một tổ gồm ba bạn thiếu
niên: Một bạn người Anh, một bạn người Pháp, một bạn người Nga. Mỗi
người trong số ba người này đều đang học một trong ba ngoại ngữ: Anh,
Pháp, Nga. Biết rằng bạn học ngoại ngữ Anh lớn hơn bạn người Pháp 1
tuổi. Hãy xác định xem bạn nước nào học ngoại ngữ gì?
Chứng minh. Trong bài toán này có hai nhóm đối tượng:
Nhóm các bạn thiếu niên: Người Anh, người Pháp, người Nga.
Nhóm các ngoại ngữ: Tiếng Anh, tiếng Pháp, tiếng Nga.
Ta lập một bảng có hàng trên là quốc tịch các bạn thiếu niên và cột bên
trái là các ngoại ngữ.
Theo đề bài: "Bạn học ngoại ngữ Anh lớn hơn người Pháp một tuổi",
ta suy ra: Bạn học người Pháp không học tiếng Anh. Do đó ta điền O
vào ô (tiếng Anh, Pháp).
Rõ ràng tiếng Pháp không phải ngoại ngữ của bạn người Pháp, tương
tự với bạn người Anh và người Nga nên ta điền O vào ô (tiếng Pháp,
Pháp), (tiếng Anh, Anh) và ô (tiếng Nga, Nga).
Tiếp đến, nhìn vào hàng tiếng Anh, ta thấy hai ô O, do đó ô còn lại phải
là có nên ta đánh dấu "x" vào ô (tiếng Anh, Nga).
53
Bảng 4.2.4
Lại thấy cột Pháp đã có 2 ô O, nên ô còn lại phải là có nên ta đánh dấu
"x" vào ô (tiếng Nga, Pháp).
Suy ra ô (tiếng Pháp, Anh) là có nên ta đánh dấu "x".
Vậy bạn người Pháp học tiếng Nga, bạn người Nga học tiếng Anh, bạn
người Anh học tiếng Pháp.
Bài toán 4.2.5. Nhiều nhất bao nhiêu người dự đoán đúng?
Khi đội tuyển của trường Lương Ngọc Quyến đi thi học sinh giỏi văn và
toán toàn quốc, sáu bạn ở nhà đã dự đoán:
An: Trường ta sẽ đạt giải vô địch môn văn.
Hải: Trường ta sẽ không đạt giải vô địch môn toán.
Lan: Trường ta đạt giải vô địch môn toán nhưng không đạt giải vô địch
môn văn.
Phan: Đạt giải vô địch văn nhưng không đạt giải vô địch toán.
Cường: Đạt giải vô dịch văn và đạt giải vô địch toán. Thịnh: Đạt giải
vô địch văn hoặc đạt giải vô địch toán nhưng không đạt cùng một lúc cả
hai giải vô địch.
Bạn hãy cho biết nhiều nhất bao nhiêu bạn dự đoán đúng và bao nhiêu
bạn dự đoán sai?
Chứng minh. Giả sử bạn Lan dự đoán đúng, thế thì An và Hải đều dự
đoán sai. Nếu Lan dự đoán sai và một trong hai bạn An hoặc Hải dự
đoán sai thì cũng có hai bạn dự đoán sai. Còn nếu cả hai bạn dự đoán
đúng, thì Cường dự đoán sai. Vậy trong mọi trường hợp vẫn có 2 bạn
dự đoán sai hay nói một cách khác, số bạn dự đoán đúng không vượt
qua 4.
Mặt khác, nếu An và Hải dự đoán đúng thì cả Phan và Thịnh cũng
dự đoán đúng. Vì vậy số người dự đoán đúng nhiều nhất không nhỏ hơn
54
4. Từ đó suy ra nhiều nhất có 4 bạn dự đoán đúng.
Gọi dự đoán của các bạn An, Hải, Lan, Phan, Cường, Thịnh theo thứ
tự là các mệnh đề A, H, L, P, C, T. Mệnh đề đúng ký hiệu là 1, mệnh
đề sai ký hiệu là 0.
Theo bài ra ta có bảng sau đây:
Bảng 4.2.5
Nhìn vào bảng trên ta cũng thấy số mệnh đề đúng nhiều nhất bằng
4 và số mệnh đề sai nhiều nhất cũng bằng 4.
Bài toán 4.2.6. Có ba cậu bé từ Quy Nhơn, Đà Nẵng và Huế tới dự trại
hè. Chúng bằng tuổi nhau và chúng cùng thích thể thao. Chỉ có Châu và
cậu bé đến từ Quy Nhơn chơi tennis, Tốc và cậu bé tới từ Đà Nẵng chơi
bóng, Dung chơi cờ tướng và lớn hơn cậu bé từ Đà Nẵng. Các cậu bé chơi
tennis không chơi cờ. Cậu bé chơi cờ lớn nhất.
Hỏi cậu bé nào đến từ thành phố nào? Và thích môn thể thao nào? Ai
lớn hơn ai?
Chứng minh. Trước tiên ta xác định xem ai là người thành phố nào?
Thích môn thể thao nào?
Ta lập bảng hai chiều: Chiều thẳng đứng ghi tên môn thể thao và thành
phố, chiều ngang ghi tên các cậu bé và thành phố. Bảng này chứa tất cả
các khả năng có thể đề kết hợp tên người và tên môn thể thao ưa thích,
tên người với tên thành phố, tức là những tích đề các của hai tập hợp:
A × X và A × Y với
A = {Châu, Tốc, Dung}
X = {Tennis, Bóng bàn, Cờ}
Y = {Qui Nhơn, Đà Nẵng, Huế}
Từ giả thiết ta có:
Do Châu và cậu bé từ Quy Nhơn chơi tennis, suy ra Châu không ở Quy
55
Nhơn, vì vậy ô (Quy Nhơn, Châu) ghi O.
Do Tốc và cậu bé từ Đà Nẵng chơi bóng đá, suy ra Tốc không ở Đà
Nẵng, vì vậy ô (Đà Nẵng, Tốc) ghi O.
Suy ra Dung không ở Q vì cậu bé từ Q chơi tennis, mà chơi tennis thì
không chơi cờ, do đó Dung ở H. Vì vậy ô (H, Dung) đánh dấu "x". Suy
ra các ô còn lại ở dòng Huế, các ô (Quy Nhơn, Dung) và (Đà Nẵng,
Dung) ghi O.
Dòng Quy Nhơn còn lại duy nhất ô (Quy Nhơn, Tốc) trống, nên ô này
ghi dấu "x".
Dòng Đà Nẵng còn lại duy nhất ô (Đà Nẵng, Châu) trống nên ô này ghi
dấu "x".
Cũng từ giả thiết ta có: ô (Tennis, Châu), (Tennis, Quy Nhơn) ghi dấu
"x"; ô (Bóng bàn, Tốc), (Bóng bàn, Đà Nẵng) ghi dấu "x"; ô (Cờ tướng,
Dung) ghi dấu "x".
Mặt khác cậu bé chơi cờ lớn nhất, suy ra Dung lớn nhất. Dung lại lớn
hơn cậu bé từ Đà Nẵng, do đó Dung lớn hơn Châu. Suy ra Tốc bé nhất.
Vậy: Châu chơi tennis và đến từ Đà Nẵng. Tốc chơi bóng và đến từ Quy
Nhơn, Dung chơi cờ và đến từ Huế.
Bài toán 4.2.7. Trong một buồng trên toa tàu có 6 hành khách A, B,
V, G, D, E từ 6 thành phố M, L, K, T, X, O. Dọc đường họ phát hiện
ra:
1. A và người từ M là bác sỹ, D và người từ L là giáo viên, V và người
từ T là kỹ sư.
2. B, E và một người từ K là cựu chiến binh, còn người từ T chưa nhập
ngũ bao giờ.
3. Người từ X già hơn A, người từ O già hơn V, còn E là người trẻ nhất
hội.
4. B và người từ M đi K, còn V và người từ X đi O.
56
Luận văn: Phương pháp giải bài toán không mẫu mực, HAY, 9đ
Luận văn: Phương pháp giải bài toán không mẫu mực, HAY, 9đ
Luận văn: Phương pháp giải bài toán không mẫu mực, HAY, 9đ
Luận văn: Phương pháp giải bài toán không mẫu mực, HAY, 9đ
Luận văn: Phương pháp giải bài toán không mẫu mực, HAY, 9đ
Luận văn: Phương pháp giải bài toán không mẫu mực, HAY, 9đ
Luận văn: Phương pháp giải bài toán không mẫu mực, HAY, 9đ
Luận văn: Phương pháp giải bài toán không mẫu mực, HAY, 9đ
Luận văn: Phương pháp giải bài toán không mẫu mực, HAY, 9đ
Luận văn: Phương pháp giải bài toán không mẫu mực, HAY, 9đ
Luận văn: Phương pháp giải bài toán không mẫu mực, HAY, 9đ
Luận văn: Phương pháp giải bài toán không mẫu mực, HAY, 9đ
Luận văn: Phương pháp giải bài toán không mẫu mực, HAY, 9đ
Luận văn: Phương pháp giải bài toán không mẫu mực, HAY, 9đ
Luận văn: Phương pháp giải bài toán không mẫu mực, HAY, 9đ
Luận văn: Phương pháp giải bài toán không mẫu mực, HAY, 9đ
Luận văn: Phương pháp giải bài toán không mẫu mực, HAY, 9đ
Luận văn: Phương pháp giải bài toán không mẫu mực, HAY, 9đ
Luận văn: Phương pháp giải bài toán không mẫu mực, HAY, 9đ
Luận văn: Phương pháp giải bài toán không mẫu mực, HAY, 9đ
Luận văn: Phương pháp giải bài toán không mẫu mực, HAY, 9đ
Luận văn: Phương pháp giải bài toán không mẫu mực, HAY, 9đ
Luận văn: Phương pháp giải bài toán không mẫu mực, HAY, 9đ
Luận văn: Phương pháp giải bài toán không mẫu mực, HAY, 9đ
Luận văn: Phương pháp giải bài toán không mẫu mực, HAY, 9đ
Luận văn: Phương pháp giải bài toán không mẫu mực, HAY, 9đ
Luận văn: Phương pháp giải bài toán không mẫu mực, HAY, 9đ
Luận văn: Phương pháp giải bài toán không mẫu mực, HAY, 9đ
Luận văn: Phương pháp giải bài toán không mẫu mực, HAY, 9đ
Luận văn: Phương pháp giải bài toán không mẫu mực, HAY, 9đ
Luận văn: Phương pháp giải bài toán không mẫu mực, HAY, 9đ
Luận văn: Phương pháp giải bài toán không mẫu mực, HAY, 9đ
Luận văn: Phương pháp giải bài toán không mẫu mực, HAY, 9đ
Luận văn: Phương pháp giải bài toán không mẫu mực, HAY, 9đ
Luận văn: Phương pháp giải bài toán không mẫu mực, HAY, 9đ
Luận văn: Phương pháp giải bài toán không mẫu mực, HAY, 9đ

More Related Content

What's hot

Các chuyên đề bồi dưỡng HSG môn Toán THCS hay nhất
Các chuyên đề bồi dưỡng HSG môn Toán THCS hay nhấtCác chuyên đề bồi dưỡng HSG môn Toán THCS hay nhất
Các chuyên đề bồi dưỡng HSG môn Toán THCS hay nhấtBồi dưỡng Toán lớp 6
 
Xstk 07 12_2015_9914
Xstk 07 12_2015_9914Xstk 07 12_2015_9914
Xstk 07 12_2015_9914Nam Cengroup
 
chuyên đề cực trị GTLN và GTNN , rất chi tiết và đầy đủ
chuyên đề cực trị GTLN và GTNN , rất chi tiết và đầy đủ chuyên đề cực trị GTLN và GTNN , rất chi tiết và đầy đủ
chuyên đề cực trị GTLN và GTNN , rất chi tiết và đầy đủ Jackson Linh
 
9 phương pháp giải phương trình nghiệm nguyên
9 phương pháp giải phương trình nghiệm nguyên9 phương pháp giải phương trình nghiệm nguyên
9 phương pháp giải phương trình nghiệm nguyênThấy Tên Tao Không
 
Bài tập về cấp của một số nguyên modulo n
Bài tập về cấp của một số nguyên modulo nBài tập về cấp của một số nguyên modulo n
Bài tập về cấp của một số nguyên modulo nLuu Tuong
 
Công thức vật lý lớp 11
Công thức vật lý lớp 11Công thức vật lý lớp 11
Công thức vật lý lớp 11Vô Ngã
 
Chuong 2 dai so tuyen tinh 2
Chuong 2   dai so tuyen tinh 2Chuong 2   dai so tuyen tinh 2
Chuong 2 dai so tuyen tinh 2Trương Huỳnh
 
chuong 4. dai so boole
chuong 4.  dai so boolechuong 4.  dai so boole
chuong 4. dai so boolekikihoho
 
Ứng dụng đồng dư vào giải toán chia hết lớp 9
Ứng dụng đồng dư vào giải toán chia hết lớp 9Ứng dụng đồng dư vào giải toán chia hết lớp 9
Ứng dụng đồng dư vào giải toán chia hết lớp 9youngunoistalented1995
 
Bài toán số học liên quan tới lũy thữa
Bài toán số học liên quan tới lũy thữaBài toán số học liên quan tới lũy thữa
Bài toán số học liên quan tới lũy thữaThế Giới Tinh Hoa
 
Ebook 4000 câu hỏi luyện thi Olympia 2016-Thiều Đình Chung
Ebook 4000 câu hỏi luyện thi Olympia 2016-Thiều Đình ChungEbook 4000 câu hỏi luyện thi Olympia 2016-Thiều Đình Chung
Ebook 4000 câu hỏi luyện thi Olympia 2016-Thiều Đình ChungChung Đình
 
Một số bất đẳng thức hình học luận văn của thầy hoàng ngọc quang
Một số bất đẳng thức hình học   luận văn của thầy hoàng ngọc quangMột số bất đẳng thức hình học   luận văn của thầy hoàng ngọc quang
Một số bất đẳng thức hình học luận văn của thầy hoàng ngọc quangThế Giới Tinh Hoa
 
Cđ đồng dư thức trong toán 7
Cđ đồng dư thức trong toán 7Cđ đồng dư thức trong toán 7
Cđ đồng dư thức trong toán 7Cảnh
 
Cong thuc luong giac day du
Cong thuc luong giac  day duCong thuc luong giac  day du
Cong thuc luong giac day duLe Nguyen
 
Cac cong thuc luong giac day du chinh xac
Cac cong thuc luong giac day du chinh xacCac cong thuc luong giac day du chinh xac
Cac cong thuc luong giac day du chinh xacb00mx_xb00m
 
302 BÀI TẬP ÔN TẬP CHƯƠNG 1 - SỐ THẬP PHÂN - TOÁN 5
302 BÀI TẬP ÔN TẬP CHƯƠNG 1 - SỐ THẬP PHÂN - TOÁN 5302 BÀI TẬP ÔN TẬP CHƯƠNG 1 - SỐ THẬP PHÂN - TOÁN 5
302 BÀI TẬP ÔN TẬP CHƯƠNG 1 - SỐ THẬP PHÂN - TOÁN 5Bồi Dưỡng HSG Toán Lớp 3
 

What's hot (20)

Bất đẳng thức hình học
Bất đẳng thức hình họcBất đẳng thức hình học
Bất đẳng thức hình học
 
Các chuyên đề bồi dưỡng HSG môn Toán THCS hay nhất
Các chuyên đề bồi dưỡng HSG môn Toán THCS hay nhấtCác chuyên đề bồi dưỡng HSG môn Toán THCS hay nhất
Các chuyên đề bồi dưỡng HSG môn Toán THCS hay nhất
 
Xstk 07 12_2015_9914
Xstk 07 12_2015_9914Xstk 07 12_2015_9914
Xstk 07 12_2015_9914
 
chuyên đề cực trị GTLN và GTNN , rất chi tiết và đầy đủ
chuyên đề cực trị GTLN và GTNN , rất chi tiết và đầy đủ chuyên đề cực trị GTLN và GTNN , rất chi tiết và đầy đủ
chuyên đề cực trị GTLN và GTNN , rất chi tiết và đầy đủ
 
9 phương pháp giải phương trình nghiệm nguyên
9 phương pháp giải phương trình nghiệm nguyên9 phương pháp giải phương trình nghiệm nguyên
9 phương pháp giải phương trình nghiệm nguyên
 
BỒI DƯỠNG NĂNG LỰC MÔ HÌNH HÓA TOÁN HỌC CÁC BÀI TOÁN THỰC TIỄN CHO HỌC SINH T...
BỒI DƯỠNG NĂNG LỰC MÔ HÌNH HÓA TOÁN HỌC CÁC BÀI TOÁN THỰC TIỄN CHO HỌC SINH T...BỒI DƯỠNG NĂNG LỰC MÔ HÌNH HÓA TOÁN HỌC CÁC BÀI TOÁN THỰC TIỄN CHO HỌC SINH T...
BỒI DƯỠNG NĂNG LỰC MÔ HÌNH HÓA TOÁN HỌC CÁC BÀI TOÁN THỰC TIỄN CHO HỌC SINH T...
 
Dãy số và giới hạn
Dãy số và giới hạnDãy số và giới hạn
Dãy số và giới hạn
 
Luận văn: Một số lớp bài toán về phương trình hàm, HAY, 9đ
Luận văn: Một số lớp bài toán về phương trình hàm, HAY, 9đLuận văn: Một số lớp bài toán về phương trình hàm, HAY, 9đ
Luận văn: Một số lớp bài toán về phương trình hàm, HAY, 9đ
 
Bài tập về cấp của một số nguyên modulo n
Bài tập về cấp của một số nguyên modulo nBài tập về cấp của một số nguyên modulo n
Bài tập về cấp của một số nguyên modulo n
 
Công thức vật lý lớp 11
Công thức vật lý lớp 11Công thức vật lý lớp 11
Công thức vật lý lớp 11
 
Chuong 2 dai so tuyen tinh 2
Chuong 2   dai so tuyen tinh 2Chuong 2   dai so tuyen tinh 2
Chuong 2 dai so tuyen tinh 2
 
chuong 4. dai so boole
chuong 4.  dai so boolechuong 4.  dai so boole
chuong 4. dai so boole
 
Ứng dụng đồng dư vào giải toán chia hết lớp 9
Ứng dụng đồng dư vào giải toán chia hết lớp 9Ứng dụng đồng dư vào giải toán chia hết lớp 9
Ứng dụng đồng dư vào giải toán chia hết lớp 9
 
Bài toán số học liên quan tới lũy thữa
Bài toán số học liên quan tới lũy thữaBài toán số học liên quan tới lũy thữa
Bài toán số học liên quan tới lũy thữa
 
Ebook 4000 câu hỏi luyện thi Olympia 2016-Thiều Đình Chung
Ebook 4000 câu hỏi luyện thi Olympia 2016-Thiều Đình ChungEbook 4000 câu hỏi luyện thi Olympia 2016-Thiều Đình Chung
Ebook 4000 câu hỏi luyện thi Olympia 2016-Thiều Đình Chung
 
Một số bất đẳng thức hình học luận văn của thầy hoàng ngọc quang
Một số bất đẳng thức hình học   luận văn của thầy hoàng ngọc quangMột số bất đẳng thức hình học   luận văn của thầy hoàng ngọc quang
Một số bất đẳng thức hình học luận văn của thầy hoàng ngọc quang
 
Cđ đồng dư thức trong toán 7
Cđ đồng dư thức trong toán 7Cđ đồng dư thức trong toán 7
Cđ đồng dư thức trong toán 7
 
Cong thuc luong giac day du
Cong thuc luong giac  day duCong thuc luong giac  day du
Cong thuc luong giac day du
 
Cac cong thuc luong giac day du chinh xac
Cac cong thuc luong giac day du chinh xacCac cong thuc luong giac day du chinh xac
Cac cong thuc luong giac day du chinh xac
 
302 BÀI TẬP ÔN TẬP CHƯƠNG 1 - SỐ THẬP PHÂN - TOÁN 5
302 BÀI TẬP ÔN TẬP CHƯƠNG 1 - SỐ THẬP PHÂN - TOÁN 5302 BÀI TẬP ÔN TẬP CHƯƠNG 1 - SỐ THẬP PHÂN - TOÁN 5
302 BÀI TẬP ÔN TẬP CHƯƠNG 1 - SỐ THẬP PHÂN - TOÁN 5
 

Similar to Luận văn: Phương pháp giải bài toán không mẫu mực, HAY, 9đ

Sáng kiến kinh nghiệm đổi mới phương pháp giảng dạy Toán học
Sáng kiến kinh nghiệm đổi mới phương pháp giảng dạy Toán họcSáng kiến kinh nghiệm đổi mới phương pháp giảng dạy Toán học
Sáng kiến kinh nghiệm đổi mới phương pháp giảng dạy Toán họcHọc Tập Long An
 
SH_Lien_ND_Dinh ly thang du Trung Hoa_VP_2016_08_16.pdf
SH_Lien_ND_Dinh ly thang du Trung Hoa_VP_2016_08_16.pdfSH_Lien_ND_Dinh ly thang du Trung Hoa_VP_2016_08_16.pdf
SH_Lien_ND_Dinh ly thang du Trung Hoa_VP_2016_08_16.pdfNguyenTanBinh4
 
Luận văn: Bài toán cực trị với đa thức đối xứng ba biến, HAY - Gửi miễn phí q...
Luận văn: Bài toán cực trị với đa thức đối xứng ba biến, HAY - Gửi miễn phí q...Luận văn: Bài toán cực trị với đa thức đối xứng ba biến, HAY - Gửi miễn phí q...
Luận văn: Bài toán cực trị với đa thức đối xứng ba biến, HAY - Gửi miễn phí q...Dịch vụ viết bài trọn gói ZALO: 0909232620
 
Luận văn: Một số lớp bài toán về loại phương trình hàm, HAY - Gửi miễn phí qu...
Luận văn: Một số lớp bài toán về loại phương trình hàm, HAY - Gửi miễn phí qu...Luận văn: Một số lớp bài toán về loại phương trình hàm, HAY - Gửi miễn phí qu...
Luận văn: Một số lớp bài toán về loại phương trình hàm, HAY - Gửi miễn phí qu...Dịch vụ viết bài trọn gói ZALO: 0909232620
 
17_skkn_toan_9_ren_ki_nang_giai_bai_toan_bang_cach_lap_phuong_trinh_va_he_phu...
17_skkn_toan_9_ren_ki_nang_giai_bai_toan_bang_cach_lap_phuong_trinh_va_he_phu...17_skkn_toan_9_ren_ki_nang_giai_bai_toan_bang_cach_lap_phuong_trinh_va_he_phu...
17_skkn_toan_9_ren_ki_nang_giai_bai_toan_bang_cach_lap_phuong_trinh_va_he_phu...https://dichvuvietluanvan.com/
 
skkn toan 9 ren ki nang giai bai toan bang cach lap phuong trinh va he phuong...
skkn toan 9 ren ki nang giai bai toan bang cach lap phuong trinh va he phuong...skkn toan 9 ren ki nang giai bai toan bang cach lap phuong trinh va he phuong...
skkn toan 9 ren ki nang giai bai toan bang cach lap phuong trinh va he phuong...https://dichvuvietluanvan.com/
 
Svtoantin com-chuyendecuctri-121205215618-phpapp012020
Svtoantin com-chuyendecuctri-121205215618-phpapp012020Svtoantin com-chuyendecuctri-121205215618-phpapp012020
Svtoantin com-chuyendecuctri-121205215618-phpapp012020LngVnGiang
 
Cđ dãy số viết theo quy luật
Cđ dãy số viết theo quy luậtCđ dãy số viết theo quy luật
Cđ dãy số viết theo quy luậtCảnh
 

Similar to Luận văn: Phương pháp giải bài toán không mẫu mực, HAY, 9đ (20)

Luận văn: Sáu phương pháp giải các bài toán phổ thông, HAY, 9đ
Luận văn: Sáu phương pháp giải các bài toán phổ thông, HAY, 9đLuận văn: Sáu phương pháp giải các bài toán phổ thông, HAY, 9đ
Luận văn: Sáu phương pháp giải các bài toán phổ thông, HAY, 9đ
 
Luận văn: Sáu phương pháp giải các bài toán phổ thông, HOT
Luận văn: Sáu phương pháp giải các bài toán phổ thông, HOTLuận văn: Sáu phương pháp giải các bài toán phổ thông, HOT
Luận văn: Sáu phương pháp giải các bài toán phổ thông, HOT
 
Sáng kiến kinh nghiệm đổi mới phương pháp giảng dạy Toán học
Sáng kiến kinh nghiệm đổi mới phương pháp giảng dạy Toán họcSáng kiến kinh nghiệm đổi mới phương pháp giảng dạy Toán học
Sáng kiến kinh nghiệm đổi mới phương pháp giảng dạy Toán học
 
SH_Lien_ND_Dinh ly thang du Trung Hoa_VP_2016_08_16.pdf
SH_Lien_ND_Dinh ly thang du Trung Hoa_VP_2016_08_16.pdfSH_Lien_ND_Dinh ly thang du Trung Hoa_VP_2016_08_16.pdf
SH_Lien_ND_Dinh ly thang du Trung Hoa_VP_2016_08_16.pdf
 
Luận văn thạc sĩ: Quy hoạch toàn phương, HAY, 9đ
Luận văn thạc sĩ: Quy hoạch toàn phương, HAY, 9đLuận văn thạc sĩ: Quy hoạch toàn phương, HAY, 9đ
Luận văn thạc sĩ: Quy hoạch toàn phương, HAY, 9đ
 
Luận văn: Phương pháp giải phương trình chứa ẩn dưới dấu căn
Luận văn: Phương pháp giải phương trình chứa ẩn dưới dấu cănLuận văn: Phương pháp giải phương trình chứa ẩn dưới dấu căn
Luận văn: Phương pháp giải phương trình chứa ẩn dưới dấu căn
 
Luận văn: Các dạng phương trình lượng giác, HAY, 9đ
Luận văn: Các dạng phương trình lượng giác, HAY, 9đLuận văn: Các dạng phương trình lượng giác, HAY, 9đ
Luận văn: Các dạng phương trình lượng giác, HAY, 9đ
 
Luận văn: Các dạng phương trình lượng giác, HOT - Gửi miễn phí qua zalo=> 090...
Luận văn: Các dạng phương trình lượng giác, HOT - Gửi miễn phí qua zalo=> 090...Luận văn: Các dạng phương trình lượng giác, HOT - Gửi miễn phí qua zalo=> 090...
Luận văn: Các dạng phương trình lượng giác, HOT - Gửi miễn phí qua zalo=> 090...
 
Đề tài: Lớp bất đẳng thức, bài toán cực trị với đa thức đối xứng, 9đ
Đề tài: Lớp bất đẳng thức, bài toán cực trị với đa thức đối xứng, 9đĐề tài: Lớp bất đẳng thức, bài toán cực trị với đa thức đối xứng, 9đ
Đề tài: Lớp bất đẳng thức, bài toán cực trị với đa thức đối xứng, 9đ
 
Luận văn: Bài toán cực trị với đa thức đối xứng ba biến, HAY - Gửi miễn phí q...
Luận văn: Bài toán cực trị với đa thức đối xứng ba biến, HAY - Gửi miễn phí q...Luận văn: Bài toán cực trị với đa thức đối xứng ba biến, HAY - Gửi miễn phí q...
Luận văn: Bài toán cực trị với đa thức đối xứng ba biến, HAY - Gửi miễn phí q...
 
Dtth
DtthDtth
Dtth
 
Dang thuc to hop
Dang thuc to hopDang thuc to hop
Dang thuc to hop
 
BaiGiang_2.pdf
BaiGiang_2.pdfBaiGiang_2.pdf
BaiGiang_2.pdf
 
Luận văn: Một số lớp bài toán về loại phương trình hàm, HAY - Gửi miễn phí qu...
Luận văn: Một số lớp bài toán về loại phương trình hàm, HAY - Gửi miễn phí qu...Luận văn: Một số lớp bài toán về loại phương trình hàm, HAY - Gửi miễn phí qu...
Luận văn: Một số lớp bài toán về loại phương trình hàm, HAY - Gửi miễn phí qu...
 
17_skkn_toan_9_ren_ki_nang_giai_bai_toan_bang_cach_lap_phuong_trinh_va_he_phu...
17_skkn_toan_9_ren_ki_nang_giai_bai_toan_bang_cach_lap_phuong_trinh_va_he_phu...17_skkn_toan_9_ren_ki_nang_giai_bai_toan_bang_cach_lap_phuong_trinh_va_he_phu...
17_skkn_toan_9_ren_ki_nang_giai_bai_toan_bang_cach_lap_phuong_trinh_va_he_phu...
 
skkn toan 9 ren ki nang giai bai toan bang cach lap phuong trinh va he phuong...
skkn toan 9 ren ki nang giai bai toan bang cach lap phuong trinh va he phuong...skkn toan 9 ren ki nang giai bai toan bang cach lap phuong trinh va he phuong...
skkn toan 9 ren ki nang giai bai toan bang cach lap phuong trinh va he phuong...
 
Svtoantin com-chuyendecuctri-121205215618-phpapp012020
Svtoantin com-chuyendecuctri-121205215618-phpapp012020Svtoantin com-chuyendecuctri-121205215618-phpapp012020
Svtoantin com-chuyendecuctri-121205215618-phpapp012020
 
Thpt hoang le kha-MVN 2
Thpt hoang le kha-MVN 2Thpt hoang le kha-MVN 2
Thpt hoang le kha-MVN 2
 
Cđ dãy số viết theo quy luật
Cđ dãy số viết theo quy luậtCđ dãy số viết theo quy luật
Cđ dãy số viết theo quy luật
 
Luận văn: Kết quả về nghiệm của phương trình Cauchy-Riemann
Luận văn: Kết quả về nghiệm của phương trình Cauchy-RiemannLuận văn: Kết quả về nghiệm của phương trình Cauchy-Riemann
Luận văn: Kết quả về nghiệm của phương trình Cauchy-Riemann
 

More from Dịch vụ viết bài trọn gói ZALO: 0909232620

Danh Sách 200 Đề Tài Tiểu Luận Chuyên Viên Chính Về Bảo Hiểm Xã Hội Mới Nhất
Danh Sách 200 Đề Tài Tiểu Luận Chuyên Viên Chính Về Bảo Hiểm Xã Hội Mới NhấtDanh Sách 200 Đề Tài Tiểu Luận Chuyên Viên Chính Về Bảo Hiểm Xã Hội Mới Nhất
Danh Sách 200 Đề Tài Tiểu Luận Chuyên Viên Chính Về Bảo Hiểm Xã Hội Mới NhấtDịch vụ viết bài trọn gói ZALO: 0909232620
 
Danh Sách 200 Đề Tài Báo Cáo Thực Tập Luật Phòng, Chống Hiv, Mới Nhất, Điểm Cao
Danh Sách 200 Đề Tài Báo Cáo Thực Tập Luật Phòng, Chống Hiv, Mới Nhất, Điểm CaoDanh Sách 200 Đề Tài Báo Cáo Thực Tập Luật Phòng, Chống Hiv, Mới Nhất, Điểm Cao
Danh Sách 200 Đề Tài Báo Cáo Thực Tập Luật Phòng, Chống Hiv, Mới Nhất, Điểm CaoDịch vụ viết bài trọn gói ZALO: 0909232620
 

More from Dịch vụ viết bài trọn gói ZALO: 0909232620 (20)

Danh Sách 200 Đề Tài Tiểu Luận Chuyên Viên Chính Về Bảo Hiểm Xã Hội Mới Nhất
Danh Sách 200 Đề Tài Tiểu Luận Chuyên Viên Chính Về Bảo Hiểm Xã Hội Mới NhấtDanh Sách 200 Đề Tài Tiểu Luận Chuyên Viên Chính Về Bảo Hiểm Xã Hội Mới Nhất
Danh Sách 200 Đề Tài Tiểu Luận Chuyên Viên Chính Về Bảo Hiểm Xã Hội Mới Nhất
 
Danh Sách 200 Đề Tài Luận Văn Thạc Sĩ Quản Trị Nguồn Nhân Lực, 9 Điểm
Danh Sách 200 Đề Tài Luận Văn Thạc Sĩ Quản Trị Nguồn Nhân Lực, 9 ĐiểmDanh Sách 200 Đề Tài Luận Văn Thạc Sĩ Quản Trị Nguồn Nhân Lực, 9 Điểm
Danh Sách 200 Đề Tài Luận Văn Thạc Sĩ Quản Trị Nguồn Nhân Lực, 9 Điểm
 
Danh Sách 200 Đề Tài Luận Văn Thạc Sĩ Quản Lý Văn Hóa Giúp Bạn Thêm Ý Tưởng
Danh Sách 200 Đề Tài Luận Văn Thạc Sĩ Quản Lý Văn Hóa Giúp Bạn Thêm Ý TưởngDanh Sách 200 Đề Tài Luận Văn Thạc Sĩ Quản Lý Văn Hóa Giúp Bạn Thêm Ý Tưởng
Danh Sách 200 Đề Tài Luận Văn Thạc Sĩ Quản Lý Văn Hóa Giúp Bạn Thêm Ý Tưởng
 
Danh Sách 200 Đề Tài Báo Cáo Thực Tập Quản Lý Giáo Dục Dễ Làm Điểm Cao
Danh Sách 200 Đề Tài Báo Cáo Thực Tập Quản Lý Giáo Dục Dễ Làm Điểm CaoDanh Sách 200 Đề Tài Báo Cáo Thực Tập Quản Lý Giáo Dục Dễ Làm Điểm Cao
Danh Sách 200 Đề Tài Báo Cáo Thực Tập Quản Lý Giáo Dục Dễ Làm Điểm Cao
 
Danh Sách 200 Đề Tài Báo Cáo Thực Tập Quan Hệ Lao Động Từ Sinh Viên Giỏi
Danh Sách 200 Đề Tài Báo Cáo Thực Tập Quan Hệ Lao Động Từ Sinh Viên GiỏiDanh Sách 200 Đề Tài Báo Cáo Thực Tập Quan Hệ Lao Động Từ Sinh Viên Giỏi
Danh Sách 200 Đề Tài Báo Cáo Thực Tập Quan Hệ Lao Động Từ Sinh Viên Giỏi
 
Danh Sách 200 Đề Tài Báo Cáo Thực Tập Nuôi Trồng Thủy Sản Dễ Làm Nhất
Danh Sách 200 Đề Tài Báo Cáo Thực Tập Nuôi Trồng Thủy Sản Dễ Làm NhấtDanh Sách 200 Đề Tài Báo Cáo Thực Tập Nuôi Trồng Thủy Sản Dễ Làm Nhất
Danh Sách 200 Đề Tài Báo Cáo Thực Tập Nuôi Trồng Thủy Sản Dễ Làm Nhất
 
Danh Sách 200 Đề Tài Báo Cáo Thực Tập Luật Sư, Mới Nhất, Điểm Cao
Danh Sách 200 Đề Tài Báo Cáo Thực Tập Luật Sư, Mới Nhất, Điểm CaoDanh Sách 200 Đề Tài Báo Cáo Thực Tập Luật Sư, Mới Nhất, Điểm Cao
Danh Sách 200 Đề Tài Báo Cáo Thực Tập Luật Sư, Mới Nhất, Điểm Cao
 
Danh Sách 200 Đề Tài Báo Cáo Thực Tập Luật Phòng, Chống Hiv, Mới Nhất, Điểm Cao
Danh Sách 200 Đề Tài Báo Cáo Thực Tập Luật Phòng, Chống Hiv, Mới Nhất, Điểm CaoDanh Sách 200 Đề Tài Báo Cáo Thực Tập Luật Phòng, Chống Hiv, Mới Nhất, Điểm Cao
Danh Sách 200 Đề Tài Báo Cáo Thực Tập Luật Phòng, Chống Hiv, Mới Nhất, Điểm Cao
 
Danh Sách 200 Đề Tài Báo Cáo Thực Tập Luật Phá Sản, Mới Nhất
Danh Sách 200 Đề Tài Báo Cáo Thực Tập Luật Phá Sản, Mới NhấtDanh Sách 200 Đề Tài Báo Cáo Thực Tập Luật Phá Sản, Mới Nhất
Danh Sách 200 Đề Tài Báo Cáo Thực Tập Luật Phá Sản, Mới Nhất
 
Danh Sách 200 Đề Tài Báo Cáo Thực Tập Luật Nhà Ở, Điểm Cao
Danh Sách 200 Đề Tài Báo Cáo Thực Tập Luật Nhà Ở, Điểm CaoDanh Sách 200 Đề Tài Báo Cáo Thực Tập Luật Nhà Ở, Điểm Cao
Danh Sách 200 Đề Tài Báo Cáo Thực Tập Luật Nhà Ở, Điểm Cao
 
Danh Sách 200 Đề Tài Báo Cáo Thực Tập Luật Ngân Hàng, Mới Nhất
Danh Sách 200 Đề Tài Báo Cáo Thực Tập Luật Ngân Hàng, Mới NhấtDanh Sách 200 Đề Tài Báo Cáo Thực Tập Luật Ngân Hàng, Mới Nhất
Danh Sách 200 Đề Tài Báo Cáo Thực Tập Luật Ngân Hàng, Mới Nhất
 
Danh Sách 200 Đề Tài Báo Cáo Thực Tập Luật Môi Trường, Mới Nhất
Danh Sách 200 Đề Tài Báo Cáo Thực Tập Luật Môi Trường, Mới NhấtDanh Sách 200 Đề Tài Báo Cáo Thực Tập Luật Môi Trường, Mới Nhất
Danh Sách 200 Đề Tài Báo Cáo Thực Tập Luật Môi Trường, Mới Nhất
 
Danh Sách 200 Đề Tài Báo Cáo Thực Tập Luật Hộ Tịch, Điểm Cao
Danh Sách 200 Đề Tài Báo Cáo Thực Tập Luật Hộ Tịch, Điểm CaoDanh Sách 200 Đề Tài Báo Cáo Thực Tập Luật Hộ Tịch, Điểm Cao
Danh Sách 200 Đề Tài Báo Cáo Thực Tập Luật Hộ Tịch, Điểm Cao
 
Danh Sách 200 Đề Tài Báo Cáo Thực Tập Luật Hình Sự , Dễ Làm Điểm Cao
Danh Sách 200 Đề Tài Báo Cáo Thực Tập Luật Hình Sự , Dễ Làm Điểm CaoDanh Sách 200 Đề Tài Báo Cáo Thực Tập Luật Hình Sự , Dễ Làm Điểm Cao
Danh Sách 200 Đề Tài Báo Cáo Thực Tập Luật Hình Sự , Dễ Làm Điểm Cao
 
Danh Sách 200 Đề Tài Báo Cáo Thực Tập Luật Hành Chính, Dễ Làm Điểm Cao
Danh Sách 200 Đề Tài Báo Cáo Thực Tập Luật Hành Chính, Dễ Làm Điểm CaoDanh Sách 200 Đề Tài Báo Cáo Thực Tập Luật Hành Chính, Dễ Làm Điểm Cao
Danh Sách 200 Đề Tài Báo Cáo Thực Tập Luật Hành Chính, Dễ Làm Điểm Cao
 
Danh Sách 200 Đề Tài Báo Cáo Thực Tập Luật Giáo Dục, Điểm Cao
Danh Sách 200 Đề Tài Báo Cáo Thực Tập Luật Giáo Dục, Điểm CaoDanh Sách 200 Đề Tài Báo Cáo Thực Tập Luật Giáo Dục, Điểm Cao
Danh Sách 200 Đề Tài Báo Cáo Thực Tập Luật Giáo Dục, Điểm Cao
 
Danh Sách 200 Đề Tài Báo Cáo Thực Tập Luật Đấu Thầu, Từ Sinh Viên Khá Giỏi
Danh Sách 200 Đề Tài Báo Cáo Thực Tập Luật Đấu Thầu, Từ Sinh Viên Khá GiỏiDanh Sách 200 Đề Tài Báo Cáo Thực Tập Luật Đấu Thầu, Từ Sinh Viên Khá Giỏi
Danh Sách 200 Đề Tài Báo Cáo Thực Tập Luật Đấu Thầu, Từ Sinh Viên Khá Giỏi
 
Danh Sách 200 Đề Tài Báo Cáo Thực Tập Luật Đầu Tư, Dễ Làm Điểm Cao
Danh Sách 200 Đề Tài Báo Cáo Thực Tập Luật Đầu Tư, Dễ Làm Điểm CaoDanh Sách 200 Đề Tài Báo Cáo Thực Tập Luật Đầu Tư, Dễ Làm Điểm Cao
Danh Sách 200 Đề Tài Báo Cáo Thực Tập Luật Đầu Tư, Dễ Làm Điểm Cao
 
Danh Sách 200 Đề Tài Báo Cáo Thực Tập Luật Đầu Tư Công, Dễ Làm Điểm Cao
Danh Sách 200 Đề Tài Báo Cáo Thực Tập Luật Đầu Tư Công, Dễ Làm Điểm CaoDanh Sách 200 Đề Tài Báo Cáo Thực Tập Luật Đầu Tư Công, Dễ Làm Điểm Cao
Danh Sách 200 Đề Tài Báo Cáo Thực Tập Luật Đầu Tư Công, Dễ Làm Điểm Cao
 
Danh Sách 200 Đề Tài Báo Cáo Thực Tập Luật Đất Đai, Từ Sinh Viên Khá Giỏi
Danh Sách 200 Đề Tài Báo Cáo Thực Tập Luật Đất Đai, Từ Sinh Viên Khá GiỏiDanh Sách 200 Đề Tài Báo Cáo Thực Tập Luật Đất Đai, Từ Sinh Viên Khá Giỏi
Danh Sách 200 Đề Tài Báo Cáo Thực Tập Luật Đất Đai, Từ Sinh Viên Khá Giỏi
 

Recently uploaded

BỘ ĐỀ KIỂM TRA CUỐI KÌ 2 VẬT LÝ 11 - KẾT NỐI TRI THỨC - THEO CẤU TRÚC ĐỀ MIN...
BỘ ĐỀ KIỂM TRA CUỐI KÌ 2 VẬT LÝ 11 - KẾT NỐI TRI THỨC - THEO CẤU TRÚC ĐỀ MIN...BỘ ĐỀ KIỂM TRA CUỐI KÌ 2 VẬT LÝ 11 - KẾT NỐI TRI THỨC - THEO CẤU TRÚC ĐỀ MIN...
BỘ ĐỀ KIỂM TRA CUỐI KÌ 2 VẬT LÝ 11 - KẾT NỐI TRI THỨC - THEO CẤU TRÚC ĐỀ MIN...Nguyen Thanh Tu Collection
 
BỘ ĐỀ PHÁT TRIỂN THEO CẤU TRÚC ĐỀ MINH HỌA BGD NGÀY 22-3-2024 KỲ THI TỐT NGHI...
BỘ ĐỀ PHÁT TRIỂN THEO CẤU TRÚC ĐỀ MINH HỌA BGD NGÀY 22-3-2024 KỲ THI TỐT NGHI...BỘ ĐỀ PHÁT TRIỂN THEO CẤU TRÚC ĐỀ MINH HỌA BGD NGÀY 22-3-2024 KỲ THI TỐT NGHI...
BỘ ĐỀ PHÁT TRIỂN THEO CẤU TRÚC ĐỀ MINH HỌA BGD NGÀY 22-3-2024 KỲ THI TỐT NGHI...Nguyen Thanh Tu Collection
 
Chuong trinh dao tao Su pham Khoa hoc tu nhien, ma nganh - 7140247.pdf
Chuong trinh dao tao Su pham Khoa hoc tu nhien, ma nganh - 7140247.pdfChuong trinh dao tao Su pham Khoa hoc tu nhien, ma nganh - 7140247.pdf
Chuong trinh dao tao Su pham Khoa hoc tu nhien, ma nganh - 7140247.pdfhoangtuansinh1
 
Trích dẫn trắc nghiệm tư tưởng HCM5.docx
Trích dẫn trắc nghiệm tư tưởng HCM5.docxTrích dẫn trắc nghiệm tư tưởng HCM5.docx
Trích dẫn trắc nghiệm tư tưởng HCM5.docxnhungdt08102004
 
ôn tập lịch sử hhhhhhhhhhhhhhhhhhhhhhhhhh
ôn tập lịch sử hhhhhhhhhhhhhhhhhhhhhhhhhhôn tập lịch sử hhhhhhhhhhhhhhhhhhhhhhhhhh
ôn tập lịch sử hhhhhhhhhhhhhhhhhhhhhhhhhhvanhathvc
 
10 ĐỀ KIỂM TRA + 6 ĐỀ ÔN TẬP CUỐI KÌ 2 VẬT LÝ 11 - KẾT NỐI TRI THỨC - THEO C...
10 ĐỀ KIỂM TRA + 6 ĐỀ ÔN TẬP CUỐI KÌ 2 VẬT LÝ 11 - KẾT NỐI TRI THỨC - THEO C...10 ĐỀ KIỂM TRA + 6 ĐỀ ÔN TẬP CUỐI KÌ 2 VẬT LÝ 11 - KẾT NỐI TRI THỨC - THEO C...
10 ĐỀ KIỂM TRA + 6 ĐỀ ÔN TẬP CUỐI KÌ 2 VẬT LÝ 11 - KẾT NỐI TRI THỨC - THEO C...Nguyen Thanh Tu Collection
 
30 ĐỀ PHÁT TRIỂN THEO CẤU TRÚC ĐỀ MINH HỌA BGD NGÀY 22-3-2024 KỲ THI TỐT NGHI...
30 ĐỀ PHÁT TRIỂN THEO CẤU TRÚC ĐỀ MINH HỌA BGD NGÀY 22-3-2024 KỲ THI TỐT NGHI...30 ĐỀ PHÁT TRIỂN THEO CẤU TRÚC ĐỀ MINH HỌA BGD NGÀY 22-3-2024 KỲ THI TỐT NGHI...
30 ĐỀ PHÁT TRIỂN THEO CẤU TRÚC ĐỀ MINH HỌA BGD NGÀY 22-3-2024 KỲ THI TỐT NGHI...Nguyen Thanh Tu Collection
 
Sơ đồ tư duy môn sinh học bậc THPT.pdf
Sơ đồ tư duy môn sinh học bậc THPT.pdfSơ đồ tư duy môn sinh học bậc THPT.pdf
Sơ đồ tư duy môn sinh học bậc THPT.pdftohoanggiabao81
 
Ma trận - định thức và các ứng dụng trong kinh tế
Ma trận - định thức và các ứng dụng trong kinh tếMa trận - định thức và các ứng dụng trong kinh tế
Ma trận - định thức và các ứng dụng trong kinh tếngTonH1
 
SÁNG KIẾN “THIẾT KẾ VÀ SỬ DỤNG INFOGRAPHIC TRONG DẠY HỌC ĐỊA LÍ 11 (BỘ SÁCH K...
SÁNG KIẾN “THIẾT KẾ VÀ SỬ DỤNG INFOGRAPHIC TRONG DẠY HỌC ĐỊA LÍ 11 (BỘ SÁCH K...SÁNG KIẾN “THIẾT KẾ VÀ SỬ DỤNG INFOGRAPHIC TRONG DẠY HỌC ĐỊA LÍ 11 (BỘ SÁCH K...
SÁNG KIẾN “THIẾT KẾ VÀ SỬ DỤNG INFOGRAPHIC TRONG DẠY HỌC ĐỊA LÍ 11 (BỘ SÁCH K...Nguyen Thanh Tu Collection
 
bài 5.1.docx Sinh học di truyền đại cương năm nhất của học sinh y đa khoa
bài 5.1.docx Sinh học di truyền đại cương năm nhất của học sinh y đa khoabài 5.1.docx Sinh học di truyền đại cương năm nhất của học sinh y đa khoa
bài 5.1.docx Sinh học di truyền đại cương năm nhất của học sinh y đa khoa2353020138
 
Tư tưởng Hồ Chí Minh về độc lập dân tộc và CNXH
Tư tưởng Hồ Chí Minh về độc lập dân tộc và CNXHTư tưởng Hồ Chí Minh về độc lập dân tộc và CNXH
Tư tưởng Hồ Chí Minh về độc lập dân tộc và CNXHThaoPhuong154017
 
50 ĐỀ ĐỀ XUẤT THI VÀO 10 THPT SỞ GIÁO DỤC THANH HÓA MÔN TIẾNG ANH 9 CÓ TỰ LUẬ...
50 ĐỀ ĐỀ XUẤT THI VÀO 10 THPT SỞ GIÁO DỤC THANH HÓA MÔN TIẾNG ANH 9 CÓ TỰ LUẬ...50 ĐỀ ĐỀ XUẤT THI VÀO 10 THPT SỞ GIÁO DỤC THANH HÓA MÔN TIẾNG ANH 9 CÓ TỰ LUẬ...
50 ĐỀ ĐỀ XUẤT THI VÀO 10 THPT SỞ GIÁO DỤC THANH HÓA MÔN TIẾNG ANH 9 CÓ TỰ LUẬ...Nguyen Thanh Tu Collection
 
Bài giảng về vật liệu ceramic ( sứ vệ sinh, gạch ốp lát )
Bài giảng về vật liệu ceramic ( sứ vệ sinh, gạch ốp lát )Bài giảng về vật liệu ceramic ( sứ vệ sinh, gạch ốp lát )
Bài giảng về vật liệu ceramic ( sứ vệ sinh, gạch ốp lát )lamdapoet123
 
Chàm - Bệnh án (da liễu - bvdlct ctump) .pptx
Chàm - Bệnh án (da liễu - bvdlct ctump) .pptxChàm - Bệnh án (da liễu - bvdlct ctump) .pptx
Chàm - Bệnh án (da liễu - bvdlct ctump) .pptxendkay31
 
ĐỀ THAM KHẢO THEO HƯỚNG MINH HỌA 2025 KIỂM TRA CUỐI HỌC KÌ 2 NĂM HỌC 2023-202...
ĐỀ THAM KHẢO THEO HƯỚNG MINH HỌA 2025 KIỂM TRA CUỐI HỌC KÌ 2 NĂM HỌC 2023-202...ĐỀ THAM KHẢO THEO HƯỚNG MINH HỌA 2025 KIỂM TRA CUỐI HỌC KÌ 2 NĂM HỌC 2023-202...
ĐỀ THAM KHẢO THEO HƯỚNG MINH HỌA 2025 KIỂM TRA CUỐI HỌC KÌ 2 NĂM HỌC 2023-202...Nguyen Thanh Tu Collection
 
[GIẢI PHẪU BỆNH] Tổn thương cơ bản của tb bào mô
[GIẢI PHẪU BỆNH] Tổn thương cơ bản của tb bào mô[GIẢI PHẪU BỆNH] Tổn thương cơ bản của tb bào mô
[GIẢI PHẪU BỆNH] Tổn thương cơ bản của tb bào môBryan Williams
 
Sáng kiến “Sử dụng ứng dụng Quizizz nhằm nâng cao chất lượng ôn thi tốt nghiệ...
Sáng kiến “Sử dụng ứng dụng Quizizz nhằm nâng cao chất lượng ôn thi tốt nghiệ...Sáng kiến “Sử dụng ứng dụng Quizizz nhằm nâng cao chất lượng ôn thi tốt nghiệ...
Sáng kiến “Sử dụng ứng dụng Quizizz nhằm nâng cao chất lượng ôn thi tốt nghiệ...Nguyen Thanh Tu Collection
 
Slide Webinar Hướng dẫn sử dụng ChatGPT cho người mới bắt đầ...
Slide Webinar Hướng dẫn sử dụng ChatGPT cho người mới bắt đầ...Slide Webinar Hướng dẫn sử dụng ChatGPT cho người mới bắt đầ...
Slide Webinar Hướng dẫn sử dụng ChatGPT cho người mới bắt đầ...Học viện Kstudy
 
QUẢN LÝ HOẠT ĐỘNG GIÁO DỤC KỸ NĂNG SỐNG CHO HỌC SINH CÁC TRƯỜNG TRUNG HỌC CƠ ...
QUẢN LÝ HOẠT ĐỘNG GIÁO DỤC KỸ NĂNG SỐNG CHO HỌC SINH CÁC TRƯỜNG TRUNG HỌC CƠ ...QUẢN LÝ HOẠT ĐỘNG GIÁO DỤC KỸ NĂNG SỐNG CHO HỌC SINH CÁC TRƯỜNG TRUNG HỌC CƠ ...
QUẢN LÝ HOẠT ĐỘNG GIÁO DỤC KỸ NĂNG SỐNG CHO HỌC SINH CÁC TRƯỜNG TRUNG HỌC CƠ ...ThunTrn734461
 

Recently uploaded (20)

BỘ ĐỀ KIỂM TRA CUỐI KÌ 2 VẬT LÝ 11 - KẾT NỐI TRI THỨC - THEO CẤU TRÚC ĐỀ MIN...
BỘ ĐỀ KIỂM TRA CUỐI KÌ 2 VẬT LÝ 11 - KẾT NỐI TRI THỨC - THEO CẤU TRÚC ĐỀ MIN...BỘ ĐỀ KIỂM TRA CUỐI KÌ 2 VẬT LÝ 11 - KẾT NỐI TRI THỨC - THEO CẤU TRÚC ĐỀ MIN...
BỘ ĐỀ KIỂM TRA CUỐI KÌ 2 VẬT LÝ 11 - KẾT NỐI TRI THỨC - THEO CẤU TRÚC ĐỀ MIN...
 
BỘ ĐỀ PHÁT TRIỂN THEO CẤU TRÚC ĐỀ MINH HỌA BGD NGÀY 22-3-2024 KỲ THI TỐT NGHI...
BỘ ĐỀ PHÁT TRIỂN THEO CẤU TRÚC ĐỀ MINH HỌA BGD NGÀY 22-3-2024 KỲ THI TỐT NGHI...BỘ ĐỀ PHÁT TRIỂN THEO CẤU TRÚC ĐỀ MINH HỌA BGD NGÀY 22-3-2024 KỲ THI TỐT NGHI...
BỘ ĐỀ PHÁT TRIỂN THEO CẤU TRÚC ĐỀ MINH HỌA BGD NGÀY 22-3-2024 KỲ THI TỐT NGHI...
 
Chuong trinh dao tao Su pham Khoa hoc tu nhien, ma nganh - 7140247.pdf
Chuong trinh dao tao Su pham Khoa hoc tu nhien, ma nganh - 7140247.pdfChuong trinh dao tao Su pham Khoa hoc tu nhien, ma nganh - 7140247.pdf
Chuong trinh dao tao Su pham Khoa hoc tu nhien, ma nganh - 7140247.pdf
 
Trích dẫn trắc nghiệm tư tưởng HCM5.docx
Trích dẫn trắc nghiệm tư tưởng HCM5.docxTrích dẫn trắc nghiệm tư tưởng HCM5.docx
Trích dẫn trắc nghiệm tư tưởng HCM5.docx
 
ôn tập lịch sử hhhhhhhhhhhhhhhhhhhhhhhhhh
ôn tập lịch sử hhhhhhhhhhhhhhhhhhhhhhhhhhôn tập lịch sử hhhhhhhhhhhhhhhhhhhhhhhhhh
ôn tập lịch sử hhhhhhhhhhhhhhhhhhhhhhhhhh
 
10 ĐỀ KIỂM TRA + 6 ĐỀ ÔN TẬP CUỐI KÌ 2 VẬT LÝ 11 - KẾT NỐI TRI THỨC - THEO C...
10 ĐỀ KIỂM TRA + 6 ĐỀ ÔN TẬP CUỐI KÌ 2 VẬT LÝ 11 - KẾT NỐI TRI THỨC - THEO C...10 ĐỀ KIỂM TRA + 6 ĐỀ ÔN TẬP CUỐI KÌ 2 VẬT LÝ 11 - KẾT NỐI TRI THỨC - THEO C...
10 ĐỀ KIỂM TRA + 6 ĐỀ ÔN TẬP CUỐI KÌ 2 VẬT LÝ 11 - KẾT NỐI TRI THỨC - THEO C...
 
30 ĐỀ PHÁT TRIỂN THEO CẤU TRÚC ĐỀ MINH HỌA BGD NGÀY 22-3-2024 KỲ THI TỐT NGHI...
30 ĐỀ PHÁT TRIỂN THEO CẤU TRÚC ĐỀ MINH HỌA BGD NGÀY 22-3-2024 KỲ THI TỐT NGHI...30 ĐỀ PHÁT TRIỂN THEO CẤU TRÚC ĐỀ MINH HỌA BGD NGÀY 22-3-2024 KỲ THI TỐT NGHI...
30 ĐỀ PHÁT TRIỂN THEO CẤU TRÚC ĐỀ MINH HỌA BGD NGÀY 22-3-2024 KỲ THI TỐT NGHI...
 
Sơ đồ tư duy môn sinh học bậc THPT.pdf
Sơ đồ tư duy môn sinh học bậc THPT.pdfSơ đồ tư duy môn sinh học bậc THPT.pdf
Sơ đồ tư duy môn sinh học bậc THPT.pdf
 
Ma trận - định thức và các ứng dụng trong kinh tế
Ma trận - định thức và các ứng dụng trong kinh tếMa trận - định thức và các ứng dụng trong kinh tế
Ma trận - định thức và các ứng dụng trong kinh tế
 
SÁNG KIẾN “THIẾT KẾ VÀ SỬ DỤNG INFOGRAPHIC TRONG DẠY HỌC ĐỊA LÍ 11 (BỘ SÁCH K...
SÁNG KIẾN “THIẾT KẾ VÀ SỬ DỤNG INFOGRAPHIC TRONG DẠY HỌC ĐỊA LÍ 11 (BỘ SÁCH K...SÁNG KIẾN “THIẾT KẾ VÀ SỬ DỤNG INFOGRAPHIC TRONG DẠY HỌC ĐỊA LÍ 11 (BỘ SÁCH K...
SÁNG KIẾN “THIẾT KẾ VÀ SỬ DỤNG INFOGRAPHIC TRONG DẠY HỌC ĐỊA LÍ 11 (BỘ SÁCH K...
 
bài 5.1.docx Sinh học di truyền đại cương năm nhất của học sinh y đa khoa
bài 5.1.docx Sinh học di truyền đại cương năm nhất của học sinh y đa khoabài 5.1.docx Sinh học di truyền đại cương năm nhất của học sinh y đa khoa
bài 5.1.docx Sinh học di truyền đại cương năm nhất của học sinh y đa khoa
 
Tư tưởng Hồ Chí Minh về độc lập dân tộc và CNXH
Tư tưởng Hồ Chí Minh về độc lập dân tộc và CNXHTư tưởng Hồ Chí Minh về độc lập dân tộc và CNXH
Tư tưởng Hồ Chí Minh về độc lập dân tộc và CNXH
 
50 ĐỀ ĐỀ XUẤT THI VÀO 10 THPT SỞ GIÁO DỤC THANH HÓA MÔN TIẾNG ANH 9 CÓ TỰ LUẬ...
50 ĐỀ ĐỀ XUẤT THI VÀO 10 THPT SỞ GIÁO DỤC THANH HÓA MÔN TIẾNG ANH 9 CÓ TỰ LUẬ...50 ĐỀ ĐỀ XUẤT THI VÀO 10 THPT SỞ GIÁO DỤC THANH HÓA MÔN TIẾNG ANH 9 CÓ TỰ LUẬ...
50 ĐỀ ĐỀ XUẤT THI VÀO 10 THPT SỞ GIÁO DỤC THANH HÓA MÔN TIẾNG ANH 9 CÓ TỰ LUẬ...
 
Bài giảng về vật liệu ceramic ( sứ vệ sinh, gạch ốp lát )
Bài giảng về vật liệu ceramic ( sứ vệ sinh, gạch ốp lát )Bài giảng về vật liệu ceramic ( sứ vệ sinh, gạch ốp lát )
Bài giảng về vật liệu ceramic ( sứ vệ sinh, gạch ốp lát )
 
Chàm - Bệnh án (da liễu - bvdlct ctump) .pptx
Chàm - Bệnh án (da liễu - bvdlct ctump) .pptxChàm - Bệnh án (da liễu - bvdlct ctump) .pptx
Chàm - Bệnh án (da liễu - bvdlct ctump) .pptx
 
ĐỀ THAM KHẢO THEO HƯỚNG MINH HỌA 2025 KIỂM TRA CUỐI HỌC KÌ 2 NĂM HỌC 2023-202...
ĐỀ THAM KHẢO THEO HƯỚNG MINH HỌA 2025 KIỂM TRA CUỐI HỌC KÌ 2 NĂM HỌC 2023-202...ĐỀ THAM KHẢO THEO HƯỚNG MINH HỌA 2025 KIỂM TRA CUỐI HỌC KÌ 2 NĂM HỌC 2023-202...
ĐỀ THAM KHẢO THEO HƯỚNG MINH HỌA 2025 KIỂM TRA CUỐI HỌC KÌ 2 NĂM HỌC 2023-202...
 
[GIẢI PHẪU BỆNH] Tổn thương cơ bản của tb bào mô
[GIẢI PHẪU BỆNH] Tổn thương cơ bản của tb bào mô[GIẢI PHẪU BỆNH] Tổn thương cơ bản của tb bào mô
[GIẢI PHẪU BỆNH] Tổn thương cơ bản của tb bào mô
 
Sáng kiến “Sử dụng ứng dụng Quizizz nhằm nâng cao chất lượng ôn thi tốt nghiệ...
Sáng kiến “Sử dụng ứng dụng Quizizz nhằm nâng cao chất lượng ôn thi tốt nghiệ...Sáng kiến “Sử dụng ứng dụng Quizizz nhằm nâng cao chất lượng ôn thi tốt nghiệ...
Sáng kiến “Sử dụng ứng dụng Quizizz nhằm nâng cao chất lượng ôn thi tốt nghiệ...
 
Slide Webinar Hướng dẫn sử dụng ChatGPT cho người mới bắt đầ...
Slide Webinar Hướng dẫn sử dụng ChatGPT cho người mới bắt đầ...Slide Webinar Hướng dẫn sử dụng ChatGPT cho người mới bắt đầ...
Slide Webinar Hướng dẫn sử dụng ChatGPT cho người mới bắt đầ...
 
QUẢN LÝ HOẠT ĐỘNG GIÁO DỤC KỸ NĂNG SỐNG CHO HỌC SINH CÁC TRƯỜNG TRUNG HỌC CƠ ...
QUẢN LÝ HOẠT ĐỘNG GIÁO DỤC KỸ NĂNG SỐNG CHO HỌC SINH CÁC TRƯỜNG TRUNG HỌC CƠ ...QUẢN LÝ HOẠT ĐỘNG GIÁO DỤC KỸ NĂNG SỐNG CHO HỌC SINH CÁC TRƯỜNG TRUNG HỌC CƠ ...
QUẢN LÝ HOẠT ĐỘNG GIÁO DỤC KỸ NĂNG SỐNG CHO HỌC SINH CÁC TRƯỜNG TRUNG HỌC CƠ ...
 

Luận văn: Phương pháp giải bài toán không mẫu mực, HAY, 9đ

  • 1. ĐẠI HỌC QUỐC GIA HÀ NỘI TRƯỜNG ĐẠI HỌC KHOA HỌC TỰ NHIÊN - - - - - - - - - - - - - - - - - - - - - - - ĐINH THỊ BÍCH NGỌC ĐỀ TÀI MỘT SỐ PHƯƠNG PHÁP GIẢI BÀI TOÁN KHÔNG MẪU MỰC Chuyên ngành: Phương pháp toán sơ cấp Mã số: 60.46.01.13 LUẬN VĂN THẠC SĨ KHOA HỌC Người hướng dẫn khoa học: GS.TS Đặng Huy Ruận HÀ NỘI - 2015
  • 2. Mục lục Lời nói đầu 3 1 Phương pháp quy nạp toán học 4 1.1 Nguyên lý quy nạp . . . . . . . . . . . . . . . . . . . . . 4 1.2 Phương pháp chứng minh bằng quy nạp . . . . . . . . . 4 1.2.1 Cơ sở quy nạp . . . . . . . . . . . . . . . . . . . . 4 1.2.2 Quy nạp . . . . . . . . . . . . . . . . . . . . . . . 5 1.3 Vận dụng phương pháp quy nạp để giải bài toán không mẫu mực . . . . . . . . . . . . . . . . . . . . . . . . . . . 6 1.4 Bài tập tự giải . . . . . . . . . . . . . . . . . . . . . . . . 23 2 Phương pháp phản chứng 25 2.1 Phép suy luận phản chứng . . . . . . . . . . . . . . . . . 25 2.2 Phương pháp chứng minh bằng phản chứng . . . . . . . 25 2.3 Các bước suy luận trong chứng minh phản chứng . . . . 26 2.4 Vận dụng phương pháp phản chứng để giải các bài toán không mẫu mực . . . . . . . . . . . . . . . . . . . . . . . 27 2.5 Bài tập tự giải . . . . . . . . . . . . . . . . . . . . . . . . 37 3 Phương pháp suy luận 39 3.1 Vài nét về phương pháp suy luận . . . . . . . . . . . . . 39 3.2 Các ví dụ về vận dụng phương pháp suy luận. . . . . . . 40 3.3 Bài tập tự giải . . . . . . . . . . . . . . . . . . . . . . . . 46 4 Phương pháp bảng 50 4.1 Vài nét về phương pháp bảng . . . . . . . . . . . . . . . 50 4.2 Vận dụng phương pháp bảng để giải bài toán không mẫu mực . . . . . . . . . . . . . . . . . . . . . . . . . . . . . 50 4.3 Bài tập tự giải . . . . . . . . . . . . . . . . . . . . . . . . 59 1
  • 3. 5 Phương pháp sơ đồ 63 5.1 Giới thiệu về phương pháp sơ đồ . . . . . . . . . . . . . 63 5.2 Vận dụng phương pháp sơ đồ để giải các bài toán không mẫu mực. . . . . . . . . . . . . . . . . . . . . . . . . . . 63 5.3 Bài tập tự giải . . . . . . . . . . . . . . . . . . . . . . . . 69 6 Phương pháp đồ thị 73 6.1 Một số khái niệm và kết quả cơ bản của lý thuyết đồ thị 73 6.2 Phương pháp đồ thị . . . . . . . . . . . . . . . . . . . . . 74 6.3 Vận dụng phương pháp đồ thị để giải bài toán không mẫu mực . . . . . . . . . . . . . . . . . . . . . . . . . . . . . 75 6.4 Bài tập tự giải . . . . . . . . . . . . . . . . . . . . . . . . 87 Kết luận 91 Tài liệu tham khảo 92 2
  • 4. LỜI NÓI ĐẦU Các bài toán không mẫu mực là các bài toán mà việc giải chúng đòi hỏi suy luận, tư duy độc đáo. Việc giải các bài toán không mẫu mực giúp người thực hiện nâng cao nhanh chóng khả năng tư duy, suy luận và nhiều khi phát hiện ra những phương pháp giải toán độc đáo không ngờ. Bởi vậy rất nhiều em học sinh, đặc biệt là học sinh trường chuyên, lớp chọn thích làm quen với các bài toán này. Luận văn "Một số phương pháp giải bài toán không mẫu mực" trình bày sáu phương pháp chủ yếu để giải các bài toán không mẫu mực. Nhưng do một bài toán không mẫu mực có thể giải đồng thời bằng nhiều phương pháp khác nhau và một vài phương pháp có phần "tương tự" nên việc phân loại phương pháp, ví dụ và bài tập chỉ là tương đối. Các bài toán không mẫu mực là mảng khá lý thú trong toán học nói chung cũng như toán phổ thông nói riêng. Vì vậy, tác giả hi vọng luận văn sẽ trở thành tài liệu có ích cho các em học sinh phổ thông, đặc biệt các em học sinh trường chuyên, lớp chọn, các thầy cô giáo dạy ở cuối cấp tiểu học, các thầy cô giáo dạy toán ở trường phổ thông, các bạn sinh viên và những ai quan tâm đến mảng toán lý thú này. Luận văn được chia làm sáu chương: Chương 1 trình bày về phương pháp quy nạp toán học. Chương 2 trình bày về phương pháp phản chứng. Chương 3 trình bày về phương pháp suy luận. Chương 4 trình bày về phương pháp bảng. Chương 5 trình bày về phương pháp sơ đồ. Chương 6 trình bày về phương pháp đồ thị. Luận văn được hoàn thành dưới sự hướng dẫn, giúp đỡ tận tình của GS.TS Đặng Huy Ruận, em xin gửi tới thầy lòng biết ơn sâu sắc. Em xin gửi lời cảm ơn chân thành đến Ban chủ nhiệm khoa cùng các thầy cô giáo khoa Toán - Cơ - Tin học, Trường Đại học Khoa Học Tự Nhiên - Đại Học Quốc Gia Hà Nội đã tạo điều kiện, dạy bảo và dìu dắt em trong những năm học vừa qua. Xin chân thành cảm ơn sự giúp đỡ của bạn bè, người thân trong thời gian học tập và làm luận văn. Do khả năng nhận thức của bản thân tác giả, luận văn còn nhiều hạn chế, thiếu sót. Kính mong nhận được các ý kiến đóng góp của thầy cô cùng các bạn đọc. Xin chân thành cảm ơn! Hà Nội, tháng 7 năm 2015 3
  • 5. Chương 1 Phương pháp quy nạp toán học Phương pháp quy nạp toán học là một công cụ rất có hiệu lực trong việc chứng minh nhiều bài toán thuộc các lĩnh vực khác nhau của toán học như: số học, đại số, hình học... và đặc biệt là các bài toán không mẫu mực. Đây là một phương pháp chứng minh toán học đặc biệt cho phép ta rút ra những quy luật tổng quát dựa trên cơ sở những trường hợp riêng. Quá trình quy nạp ngược với quá trình suy diễn. Từ "tính chất" của một số cá thể suy ra "tính chất" của tập thể, nên không phải lúc nào cũng đúng. Nó chỉ đúng khi thỏa mãn nguyên lý quy nạp. 1.1 Nguyên lý quy nạp Cho n0 là một số nguyên dương và P(n) là một mệnh đề có nghĩa với mọi số tự nhiên n ≥ n0. Nếu 1.P(n0) đúng và 2. Nếu P(k) đúng từ đó suy ra được P(k + 1) cũng đúng với mọi số tự nhiên k ≥ n0 thì P(n) đúng với mọi số tự nhiên n ≥ n0. 1.2 Phương pháp chứng minh bằng quy nạp Giả sử khẳng định P(n) xác định ∀n ≥ n0. Để chứng minh P(n) đúng ∀n ≥ n0 bằng quy nạp, ta cần thực hiện 2 bước 1.2.1 Cơ sở quy nạp Kiểm tra sự đúng đắn của P(n) với n = n0, nghĩa là xét P(n0) có đúng không. 4
  • 6. 1.2.2 Quy nạp Chứng minh rằng: Nếu với mỗi k ≥ n0, P(k) là mệnh đề đúng, thì suy ra P(k + 1) cũng đúng. Nếu cả 2 bước trên đều thỏa mãn, thì theo nguyên lý quy nạp P(n) đúng với mọi n ≥ n0. Chú ý Trong quá trình quy nạp, nếu không thực hiện đầy đủ cả 2 bước: Cơ sở quy nạp và quy nạp thì có thể dẫn đến kết luận sai lầm. Một số ví dụ sau sẽ chứng tỏ điều này. - Do bỏ qua bước cơ sở quy nạp, ta đưa ra kết luận không đúng: Mọi số tự nhiên đều bằng nhau! Bằng cách quy nạp như sau: Giả sử các số tự nhiên không vượt quá k + 1 đã bằng nhau. Khi đó ta có: k = k + 1 Thêm vào mỗi vế của đẳng thức trên 1 đơn vị sẽ có: k + 1 = k + 1 + 1 = k + 2 Cứ như vậy suy ra mọi số tự nhiên không nhỏ hơn k đều bằng nhau. Kết hợp với giả thiết quy nạp: Mọi số tự nhiên không vượt quá k đều bằng nhau, đi đến kết luận sai lầm: Tất cả các số tự nhiên đều bằng nhau! - Do bỏ qua bước quy nạp nên nhà toán học Pháp P.Fermat (1601 - 1665) đã cho rằng số dạng 22n + 1 đều là số nguyên tố. P.Fermat xét 5 số đầu tiên: Với n = 0 cho 220 + 1 = 21 + 1 = 3 là số nguyên tố. Với n = 1 cho 221 + 1 = 22 + 1 = 5 là số nguyên tố. Với n = 2 cho 222 + 1 = 24 + 1 = 17 là số nguyên tố. Với n = 3 cho 223 + 1 = 28 + 1 = 257 là số nguyên tố. Với n = 4 cho 224 + 1 = 216 + 1 = 65537 là số nguyên tố. Nhưng vào thế kỷ XVIII, L.Euler (1707 - 1783) đã phát hiện với n = 5 khẳng định trên không đúng, bởi vì: 225 + 1 = 4294967297 = 641.6700417 là hợp số. 5
  • 7. 1.3 Vận dụng phương pháp quy nạp để giải bài toán không mẫu mực Phương pháp quy nạp được sử dụng trong tính toán, trong chứng minh và suy luận dưới nhiều dạng khác nhau, nhưng trong phần này chỉ trình bày việc vận dụng phương pháp quy nạp để giải bài toán không mẫu mực. Bài toán 1.3.1. (IMO 1998) Với mọi số nguyên dương n, ta kí hiệu d(n) là số tất cả các ước dương của n (kể cả 1 và n). Hãy xác định tất cả các số nguyên dương k, sao cho d(n2 ) = kd(n), với n là số nguyên dương nào đó. Chứng minh. Giả sử khi phân tích ra thừa số nguyên tố, số n có dạng: n = pa1 1 .pa2 2 ...par r Ta có: d(n) = (a1 + 1)(a2 + 1)...(ar + 1) d(n2 ) = (2a1 + 1)(2a2 + 1)...(2ar + 1) Để d(n2 ) = kd(n) thì ta phải chọn các số ai sao cho: (2a1 + 1)(2a2 + 1)...(2ar + 1) = k(a1 + 1)(a2 + 1)...(ar + 1) (∗) Do (2ai + 1)(1 ≤ i ≤ r) đều là các số lẻ nên k phải là các số lẻ. Ta sẽ chứng minh mệnh đề đảo lại rằng: "Với số lẻ k bất kỳ, ta có thể tìm được các số ai thỏa mãn (*) (tức là tìm được n)". Dùng phương pháp quy nạp theo k. 1. Với k = 1, mệnh đề đúng (n = 1, ai = 0) 2. Giả sử mệnh đề đúng với số k nào đó, ta chứng minh nó cũng đúng cho (2m .k − 1)(m ≥ 1). Lúc đó mệnh đề đúng cho mọi số lẻ vì mọi số lẻ l đều viết được dưới dạng: (2m .l − 1) (với l là số nhỏ hơn l). Đặt ai = 2i−1 [(2m − 1).k − 1], với i = 1, 2, ..., m. Khi đó: 2ai + 1 = 2i (2m − 1)k − (2i − 1) ai + 1 = 2i−1 (2m − 1)k − (2i−1 − 1) = 2ai−1 + 1 Do vậy, tích của các số (2ai + 1) chia hết cho tích các số (ai + 1) với i = 1, m khi (2am + 1) chia hết cho (a1 + 1) hay: [2m (2m − 1)k − (2m − 1)] = (2m − 1).(2m .k − 1) 6
  • 8. chia hết cho (2m − 1)k có nghĩa là: (2m .k − 1) chia hết cho k. Vậy nếu ta chọn các ai như trên với k đã cho, thì mệnh đề trên đúng cho (2m .k −1). Ta có điều phải chứng minh! Bài toán 1.3.2. (USAMTS, 2000 - 2001, Cuộc thi tài năng toán học Mỹ) Hãy tìm số dư khi chia số 17761492! cho 2000. Chứng minh. Trước hết ta chứng minh bổ đề: "Với mọi số nguyên dương n, ta có: 1376n ≡ 1376(mod 2000)" Dùng phương pháp quy nạp: 1. Với n = 1, hiển nhiên có: 13761 ≡ 1376(mod 2000) Với n = 2, ta có: 13762 = 1893376 ≡ 1376(mod 2000) 2. Giả sử mệnh đề đúng với n = k(k ∈ N, k ≥ 1), tức là: 1376k ≡ 1376(mod 2000). Ta chứng minh bổ đề đúng với n = k +1. Thật vậy, từ giả thiết quy nạp ta có: 1376k+1 ≡ 13762 (mod 2000), mà 13762 ≡ 1376(mod 2000), nên 1376k+1 ≡ 1376(mod 2000) Bổ đề được chứng minh! Quay lại bài toán, ta có: 17765 = 1376(mod 2000), nên 17761492! = (17765 ) 1492! 5 Vậy khi chia số 17761492! cho 2000 được số dư là 1376. Bài toán 1.3.3. Hãy tìm chữ số tận cùng của số: An = 22n + 1 với mọi số nguyên n, n ≥ 2. Chứng minh. 1. Với n = 2, số A2 = 222 + 1 = 17, có chữ số tận cùng là 7. 2. Giả sử với n = k, số Ak = 22k + 1 có chữ số tận cùng là 7. Ta sẽ chứng minh Ak+1 có chữ số tận cùng là 7. Thật vậy, do Ak có chữ số tận cùng là 7, nên tồn tại số nguyên dương m để: Ak = 10m + 7, hay: 22k + 1 = 10m + 7 7
  • 9. Tức là: 22k = 10m + 6. Từ đó: Ak+1 = 22k+1 + 1 = 22k .2 + 1 = (22k ) 2 + 1 = (10m + 6)2 + 1 = 100m2 + 120m + 37 = 10(10m2 + 12m + 3) + 7 nên Ak+1 cũng có chữ số tận cùng là 7. Vậy với mọi số nguyên n, n ≥ 2, thì An = 22n + 1 có chữ số tận cùng là 7. Bài toán 1.3.4. (Vô địch toán Canada, 1982) Cho a, b và c là những nghiệm của phương trình: x3 − x2 − x − 1 = 0 Chứng minh rằng số: b1982 − c1982 b − c + c1982 − a1982 c − a + a1982 − b1982 a − b là một số nguyên. Chứng minh. Đặt un = bn −cn b−c , vn = cn −an c−a , wn = an −bn a−b , với n nguyên, n ≥ 1. Ta sẽ chứng minh: un + vn + wn nguyên với mọi n nguyên, n ≥ 1(∗). Trước hết ta thấy rằng: un+3 = un+3 + un+3 + un, ∀n, n = 1. Thật vậy, do b, c là nghiệm của phương trình x3 − x2 − x − 1 = 0 nên: b3 = b2 + b + 1, c3 = c3 + c + 1 Do đó: un+3 = bn+3 − cn+3 b − c = bn .b3 − cn .c3 b − c = bn (b2 + b + 1) − cn (c2 + c + 1) b − c = bn+2 − cn+2 b − c + bn+1 − cn+1 b − c + bn − cn b − c = un+2 + un+1 + un 8
  • 10. Tương tự ta có: vn+3 = vn+2 + vn+1 + vn wn+3 = wn+2 + wn+1 + wn Tiếp theo, ta sẽ dùng phương pháp quy nạp để chứng minh khẳng định (*) 1. Với n = 1, ta có: u1 + v1 + w1 = 1 + 1 + 1 = 3 ∈ Z Với n = 2, ta có: u2 + v2 + w2 = b2 − c2 b − c + c2 − a2 c − a + a2 − b2 a − b = 2(a + b + c) = 2(− −1 1 ) = 2 ∈ Z Với n = 3 ta có: u2 + v2 + w2 = b3 − c3 b − c + c3 − a3 c − a + a3 − b3 a − b = 2(a2 + b2 + c2 ) + (bc + ca + ab) = 2(a + b + c)2 − 3(bc + ca + ab) = 2(− −1 1 )2 − 3( −1 1 ) = 5 ∈ Z Vậy khẳng định đúng với n = 1, 2, 3. 2. Giả sử khẳng định đúng với n = k, k +1, k +2(k ≥ 1). Ta chứng minh khẳng định đúng với n = k + 3. Thật vậy, ta có: uk+3 +vk+3 +wk+3 = (uk+2 +uk+1 +uk)+(vk+2 +vk+1 +vk)+(wk+2 + wk+1 + wk) (uk+2 + vk+2 + wk+2) + ((uk+1 + vk+1 + wk+1) + (uk + vk + wk) Theo giả thiết quy nạp, cả ba số hạng của tổng trên đều nguyên nên (uk+3 + vk+3 + wk+3) cũng nguyên. Khẳng định được chứng minh. Từ đó hiển nhiên tổng ở đề bài là số nguyên. Bài toán 1.3.5. (IMO 1973) Cho −−→ OP1, −−→ OP2, ..., −−−−→ OP2n+1 là các vecto đơn vị trong mặt phẳng. Các điểm P1, P2, P2n+1 đều cùng nằm về 1 phía của đường thẳng qua O. Chứng minh rằng: | −−→ OP1 + −−→ OP2 + ... + −−−−→ OP2n+1| ≥ 1 Chứng minh. 9
  • 11. 1. Với n = 1, mệnh đề hiển nhiên đúng do: | −−→ OP1| = 1 ≥ 1 2. Giả sử mệnh đề đúng với n = k − 1(k ≥ 2), tức là với hệ vecto đơn vị: −−→ OP1, −−→ OP2, ..., −−−−→ OP2k−1 cùng nằm về 1 phía của 1 đường thẳng qua O, ta đã có: | −−→ OP1 + −−→ OP2 + ... + −−−−→ OP2k−1| ≥ 1 Ta chứng minh mệnh đề đúng với n = k, tức là với hệ (2k + 1) vecto −−→ OP1, −−→ OP2, ..., −−−−→ OP2k+1 thỏa mãn các điều kiện trên, ta cũng có: | −−→ OP1 + −−→ OP2 + ... + −−−−→ OP2k+1| ≥ 1 Thật vậy, do vai trò của −−→ OPi(1 ≤ i ≤ 2k + 1) như nhau nên ta có thể sắp xếp lại sao cho −−→ OPi(1 ≤ i ≤ 2k − 1) nằm giữa −−−→ OP2k và −−−−→ OP2k+1 Đặt: −→u = −−−→ OP2k + −−−−→ OP2k+1 −→v = −−→ OP1 + −−→ OP2 + ... + −−−−→ OP2k−1 Khi đó −→u có phương nằm trên tia phân giác góc P2kOP2k+1. Áp dụng quy tắc hình bình hành nhiều lần, ta được −→v nằm giữa −−→ OP1 và −−−−→ OP2k−1, nên nó nằm giữa −−−→ OP2k và −−−−→ OP2k+1. Vậy góc giữa −→u và −→v bé hơn hoặc bằng π 2 Ta lại có: (−→u + −→v )2 = −→u 2 + −→v 2 + 2−→u −→v = −→u 2 + −→v 2 + 2|−→u ||−→v |cos(−→u , −→v ) ≥ −→v 2 (do cos(−→u , −→v ) ≥ 0) Do đó: |−→u + −→v | ≥ |−→v | Mà theo giả thiết quy nạp ta có |−→v | ≥ 1. Vậy |−→u + −→v | ≥ 1 hay | −−→ OP1 + −−→ OP2 + ... + −−−−→ OPn+1| ≥ 1 Mệnh đề đúng với n = k + 1, ta có điều phải chứng minh! Bài toán 1.3.6. (Chứng minh tính chia hết bằng quy nạp) (Định lý Fermat nhỏ) Chứng minh rằng: Nếu p là số nguyên tố, thì với mọi số nguyên dương n, hiệu np − n chia hết cho p. 10
  • 12. Chứng minh. 1. Với n = 1 ta có: 1p − 1 = 0 chia hết cho p. 2. Giả sử khẳng định đúng với số nguyên n = a ≥ 1, nghĩa là ap − a chia hết cho p. Ta chứng minh: (a + 1)p − (a + 1) cũng chia hết cho p. Theo khai triển nhị thức Newton ta có: (a + 1)p − (a + 1) = C0 p.ap + C1 p.ap−1 + C2 p.ap−2 + ... + +Cp−1 p .a + Cp p − a − 1 = (ap − a) + C1 p.ap−1 + C2 p.ap−2 + ... + +Cp−1 p .a Ta có: Ck p = p! k!(p − k)! = p(p − 1)...(p − k − 1) 1.2.3...k (với 1 ≤ k ≤ p − 1) Do p là số nguyên tố nên (p, k) = 1; ∀k, 1 ≥ k ≥ p − 1, suy ra Ck p chia hết cho p với mọi k, 1 ≤ k ≤ p − 1. Mà ap − a cũng chia hết cho p (theo giả thiết quy nạp) Vậy (a+1)p −(a+1) cũng chia hết cho p, định lý được chứng minh! Bài toán 1.3.7. (Vô địch toán Hungari 1932) Chứng minh rằng nếu a, b, n là những số tự nhiên và b chia hết cho an , thì số (a + 1)b − 1 chia hết cho an+1 Chứng minh. (Quy nạp theo n) 1. Với n = 0, ta có (a + 1)0 − 1 luôn chia hết cho a0+1 = a, nên mệnh đề đúng với n = 0. 2. Giả sử mệnh đề đúng với số tự nhiên n = k nào đó, tức là nếu b chia hết cho ak thì (a + 1)b − 1 chia hết cho ak+1 . Ta chứng minh mệnh đề đúng với n = k +1, tức là nếu b1 là số tự nhiên, b1 chia hết cho ak+1 thì (a + 1)b1 − 1 chia hết cho ak+2 . Thật vậy, đặt c = b1 a , thì c chia hết cho ak . Ta có: (a + 1)b1 − 1 = (a + 1)ca − 1 = [(a + 1)c ]a − 1 = [(a + 1)c − 1][(a + 1)c(a−1) + (a + 1)c(a−2) + ... + (a + 1)c + 1] Biểu thức trong dấu ngoặc vuông thứ nhất chia hết cho ak+1 (theo giả thiết quy nạp). 11
  • 13. Biểu thức trong dấu ngoặc vuông thứ hai chia hết cho a vì ta có thể biểu diễn nó dưới dạng: [(a + 1)c(a−1) − 1] + [(a + 1)c(a−2) − 1] + ... + [(a + 1)c − 1] + a (Mỗi số hạng tổng này đều chia hết cho a) Vậy (a + 1)b1 − 1 chia hết cho ak+2 . Mệnh đề được chứng minh! Bài toán 1.3.8. (Chứng minh đẳng thức bằng quy nạp) (Công thức nhị thức Newton) Chứng minh rằng: (a + b)n = n k=0 ck n.ak .bn−k Chứng minh. 1. Với n = 1, dễ thấy công thức đúng. 2. Giả sử công thức đúng với số nguyên dương n, tức là ta có: (a + b)n = n k=0 Ck n.ak .bn−k Ta chứng minh công thức đúng với n + 1. Thật vậy, ta có: (a + b)n+1 = (a + b)n .(a + b) = (an + C1 n.an−1 .b + ... + Ck n.an−k .bk + ... + bn )(a + b) = an+1 + C1 n.an .b + ... + Ck n.an+1−k .bk + ... + a.bn + an .b + C1 n.an−1 .b2 + ... + Ck n.an−k .bk+1 + ... + bn+1 = an+1 + (C0 n + C1 n).an .b + (C1 n + C2 n).an−1 .b2 + ... + (Ck−1 n + Ck n).aa+1−k .bk + ... + bn+1 = an+1 + C1 n+1.an .b + C2 n+1.an−1 .b2 + Ck n+1.an+1−k .bk + ... + bn−1 = n+1 k=0 Ck n+1.ak .bn+1−k Công thức được chứng minh! Bài toán 1.3.9. (Chứng minh bất đẳng thức bằng quy nạp) (Bất đẳng thức Bernoulli) Chứng minh rằng với mọi x > −1, x = 0 và mọi số tự nhiên n ≥ 2, ta có: (1 + x)n > 1 + nx 12
  • 14. Chứng minh. 1. Với n = 2, bất đẳng thức có dạng: (1 + x)2 > 1 + 2x hay x2 > 0. Điều này đúng do x = 0. 2. Giả sử bất đẳng thức đúng với n = k(k ≥ 2), tức là đã có: (1+x)k > (1 + kx) Ta chứng minh nó cũng đúng với n = k + 1. Thật vậy, ta có: (1 + x)k+1 = (1 + x)k (1 + x) > (1 + kx)(1 + x) = 1 + (k + 1)x + kx2 > 1 + (k + 1)x (Vì k > 0 và x = 0) Vậy bất đẳng thức được chứng minh! Chú ý: Bất đẳng thức Bernoulli còn đúng cho mọi số thực α > 1: (1 + x)α > l + α; ∀x, x > −1, x = 0 Bài toán 1.3.10. (Vô địch toán Matxcova 1984) Cho x1, x2, ..., xn là n số không âm (n ≥ 4), tổng của chúng bằng 1. Chứng minh rằng: x1x2 + x2x3 + ... + xnx1 ≤ 4 Chứng minh. Ta sẽ chứng minh bất đẳng thức sau bằng phương pháp quy nạp: (x1 + x2 + .... + xn)2 ≥ 4(x1x2 + x2x3 + .... + xnx1) Với xi ≥ 0, i = 1, n và n ≥ 4 1. Với n = 4, ta có: (x1 − x2 + x3 − x4)2 = x2 1 + x2 2 + x2 3 + x2 4 − 2x1x2 + 2x1x3 − 2x1x4 − 2x2x3 + 2x2x4 − 2x3x4 = (x1 + x2 + x3 + x4)2 − 4(x1x2 + x2x3 + x3x4 + x4x1) ≥ 0 Do đó: (x1 + x2 + x3 + x4)2 ≥ 4(x1x2 + x2x3 + x3x4 + x4x1) nên bất đẳng thức đúng với n = 4. 2. Giả sử bất đẳng thức đúng với n = k(k ≥ 4), tức là ta có: (x1 + x2 + ... + xk)2 ≥ 4(x1x2 + x2x3 + ... + xkx1) Ta chứng minh bất đẳng thức đúng với n = k + 1: (x1 + x2 + ... + xk + xk+1)2 ≥ 4(x1x2 + x2x3 + ... + xkxk+1 + xk+1x1) 13
  • 15. Vì tổng 2 vế của bất đẳng thức này là vòng tròn theo chỉ số, ta có thể giả thiết xk+1 ≥ xi, i = 1, k. Khi đó, từ giả thiết quy nạp ta có: (x1 + x2 + ... + xk + xk+1)2 = (x1 + x2 + ... + (xk + xk+1))2 ≥ 4[x1x2 + x2x3 + ... + xk−1(xk + xk+1) + (xk + xk+1)x1] Mà: [x1x2 + x2x3 + ... + xk−1(xk + xk+1) + (xk + xk+1)x1] = (x1x2 + x2x3 + ... + xkxk+1 + xk+1x1) + xk−1xk+1 + xk(x1 − xk+1) Vì xi ≥ 0 và x1 − xk+1 ≥ 0 nên ta có: [x1x2 + x2x3 + ... + xk−1(xk + xk+1) + (xk + xk+1)x1] ≥ (x1x2 + x2x3 + ... + xk−1xk + xkxk+1 + xk+1x1) Vậy (x1 + x2 + ... + xk + xk+1)2 ≥ 4(x1x2 + x2x3 + ... + xkxk+1 + xk+1x1) nên ta có điều phải chứng minh! Bài toán 1.3.11. (Tính toán bằng quy nạp) Tính bán kính rn, Rn của đường tròn nội tiếp và ngoại tiếp 2n - giác đều chu vi p cho trước (n ≥ 2). Chứng minh. 1. Với n = 2, ta có hình vuông chu vi p. Dễ dàng tính được: r2 = p 8 ; R2 = p √ 2 8 2. Giả sử biết bán kính rn, Rn của các đường tròn nội tiếp và ngoại tiếp 2n - giác đều chu vi p, ta tính các bán kính rn+1, Rn+1 của các đường tròn nội tiếp và ngoại tiếp 2n+1 - giác đều chu vi p. Gọi AB là cạnh của 2n - giác đều chu vi p tâm O. Gọi C, D, E, F lần lượt là trung điểm của cung AB, dây AB, AC, BC, G là trung điểm của EF. Vì (OE, OF) = (OE, OC) + (OF, OC) = 1 2 (OA, OC) + 1 2 (OB, OC) = 1 2 (OA, OB) 14
  • 16. Hình 1.3.11 và EF bằng cạnh của 2n+1 - giác đều nội tiếp đường tròn bán kính OE, nên chu vi của 2n+1 - giác này là: 2n+1 .EF = 2n+1 AB 2 = 2n .AB = p Do đó rn+1 = OG và Rn+1 = OE Mặt khác, ta dễ thấy OC − OG = OG − OD(= 1 2 CD) nên Rn − rn+1 = rn+1 − rn hay rn+1 = Rn+rn 2 Cuối cùng, từ tam giác vuông OEC ta có: OE2 = OC.OG. Nghĩa là: R2 n+1 = Rnrn+1, nên Rn+1 = √ Rnrn+1. Vậy rn+1 = Rn+rn 2 , Rn+1 = √ Rnrn+1. Bài toán 1.3.12. Trên một hình phẳng cho n đường tròn phân biệt, đôi một cắt nhau và không có ba đường tròn nào giao nhau tại 1 điểm. Các mặt phẳng này chia đường tròn thành các miền rời nhau. Tính số miền thu được. Chứng minh. Gọi số miền thu được bởi n đường tròn trong mặt phẳng thỏa mãn điều kiện đề bài là F(n). 1. Với n = 1, dễ thấy F(1) = 2. Với n = 2, ta có 2 đường tròn cắt nhau và F(2) = 4 2. Giả sử với k đường tròn thỏa mãn điều kiện đề bài, chúng chia mặt phẳng ra làm F(k) miền. Xét (k + 1) đường tròn thỏa mãn điều kiện đề bài. Ta tính F(k + 1). Gọi (k + 1) đường tròn đó là (C1), (C2), ..., (Ck+1). Bỏ đi 1 đường tròn bất kỳ trong (k + 1) đường tròn đó, chẳng hạn (Ck+1). Khi đó còn 15
  • 17. Hình 1.3.12 k đường tròn, theo giả thiết quy nạp, số miền thu được là F(k). Bây giờ ta dựng lại Ck+1. Khi đó đường tròn Ck+1 giao với cả k đường tròn ban đầu. Trên Ck+1 có k cặp giao điểm nên cho ta thêm 2k miền. Vậy F(k + 1) = F(k) + 2k. Do đó ta có: F(k) = F(k − 1) + 2(k − 1) F(k − 1) = F(k − 2) + 2(k − 2) ... F(2) = F(1) + 2.1 F(1) = 2 Cộng các đẳng thức trên lại ta được: F(k) = 2[1 + 1 + 2 + 3 + ... + (k − 2) + (k − 1)] = 2[1 + k(k − 1) 2 ] = k2 − k + 2 Vậy số miền thu được từ n đường tròn thỏa mãn đề bài là F(n) = n2 − n + 2. Bài toán 1.3.13. (Chứng minh bằng quy nạp) Chứng minh rằng mọi n - giác lồi(n ≥ 5) đều được chia thành 1 số hữu hạn các ngũ giác lồi. Chứng minh. 1. Với n = 5, mệnh đề hiển nhiên đúng. 2. Giả sử mệnh đề đúng với n = k(k ≥ 5), tức là mọi k - giác lồi đều chia được thành hữu hạn các ngũ giác lồi. Ta chứng minh mệnh đề đúng với n = k + 1, tức là mọi (k + 1) - giác lồi (H) đều chia được thành một số hữu hạn các ngũ giác lồi. 16
  • 18. Thật vậy, xét (k + 1) - giác lồi (H) = A1A2...AkAk+1. Trên các cạnh A1Ak+1 và A3A4 lần lượt lấy các điểm M, N khác các đỉnh. Đoạn MN chia (H) thành 2 đa giác: (H1) = MA1A2A3N và (H2) = MNA4A5...AkAk+1 Rõ ràng (H1) là ngũ giác lồi, còn (H2) là k giác lồi. Theo giả thiết quy Hình 1.3.13 nạp, (H2) chia được thành 1 số hữu hạn các ngũ giác lồi. Mệnh đề được chứng minh! Bài toán 1.3.14. (Chắp hình bằng quy nạp) Cho n hình vuông bất kỳ. Chứng minh có thể cắt chúng (bằng nhát cắt thẳng) làm một số mảnh đa giác, để từ đó có thể ghép lại thành 1 hình vuông lớn. Chứng minh. 1. Với n = 1, mệnh đề hiển nhiên đúng. Với n = 2, gọi độ dài các cạnh 2 hình vuông A1B1C1D1 và A2B2C2D2 tương ứng là x1 và x2. Giả sử x1 ≥ x2. Trên các cạnh của hình vuông A1B1C1D1 ta lấy các điểm M, N, P, Q sao cho A1M = B1N = C1P = D1Q = x1 + x2 2 Cắt hình vuông này theo các đoạn MP và NQ thì MP và NQ cắt nhau tại tâm O của nó và chúng vuông góc với nhau. Các đường đó chia hình vuông thành 4 phần bằng nhau, ta ghép những phần đó vào hình vuông A2B2C2D2 như hình bên. Hình nhận được sẽ là hình vuông vì giá trị tại các góc M, N, P, Q bù nhau; các góc A, B, C, D vuông và AB = BC = CD = DA. 2. Giả sử mệnh đề đúng với n(n ≥ 1) hình vuông. Ta chứng minh mệnh đề cũng đúng với (n + 1) hình vuông. 17
  • 19. Thật vậy, giả sử ta có n + 1 hình vuông (H1), (H2), ..., (Hn), (Hn+1). Ta lấy ra 2 hình vuông bất kỳ, chẳng hạn (Hn), (Hn+1). Theo phần 1, ta có thể cắt một trong hai hình vuông này và ghép các mảnh với hình vuông còn lại để được 1 hình vuông mới (H’). Như vậy, ta có n hình vuông (H1), (H2), ..., (Hn−1), (H ). Theo giả thiết quy nạp, có thể cắt và ghép chúng thành một hình vuông mới. Ta có điều phải chứng minh! Hình 1.3.14 Bài toán 1.3.15. (Tô màu bằng quy nạp) Trên mặt phẳng cho n(n ≥ 1) hình tròn. Chứng minh có thể với bất kỳ cách sắp đặt nào, thì hình nhận được cũng có thể tô bằng 2 màu, để cho 2 phần mặt phẳng kề nhau (có biên chung) cũng được tô bằng hai màu khác nhau. Chứng minh. 1. Cơ sở quy nạp. Với n = 1, trên mặt phẳng chỉ có 1 hình tròn. Ta tô hình tròn bằng màu đen. Khi đó phần mặt phẳng còn lại kề với hình tròn được để trắng, nên hai phần của mặt phẳng kề nhau có màu khác nhau. 2. Quy nạp. Giả sử khẳng định đã đúng với bức tranh n hình tròn. Giả sử trên mặt phẳng cho n + 1 hình tròn tùy ý. Xóa đi 1 trong những hình tròn, sẽ được bức tranh gồm n hình tròn. Theo giả thiết quy nạp, bức tranh này chỉ cần tô bằng hai màu, chẳng hạn, đen, trắng mà hai miền kề nhau đều có màu khác nhau. Khôi phục lại hình tròn đã xóa đi, tức là trở lại hình xuất phát gồm n + 1 hình tròn, rồi theo 1 phía đối với hình tròn vừa khôi phục, chẳng 18
  • 20. Hình 1.5 hạn phía trong của hình tròn này thay đổi các màu đã tô bằng màu ngược lại, sẽ được: bức tranh gồm n+1 hình tròn được tô bằng hai màu, mà hai miền kề nhau tùy ý đều có màu khác nhau. Bài toán 1.3.16. (Dựng hình bằng quy nạp) Trên mặt phẳng cho (2n + 1) điểm. Hãy dựng một (2n + 1) giác để các điểm đã cho là trung điểm các cạnh của đa giác. Chứng minh. 1. Với n = 1, ta dựng tam giác ABC khi biết 3 trung điểm M, N, P bằng cách qua M, N, P lần lượt dựng các đường thẳng song song với NP, MP, MN. Chúng cắt nhau cho ta tam giác ABC. 2. Giả sử đối với 2(n − 1) + 1 điểm tùy ý không thẳng hàng dựng được đa giác (2n − 1) đỉnh có các điểm đã cho là trung điểm các cạnh. Ta chứng minh có thể dựng được (2n + 1) - giác từ trung điểm các cạnh của nó. Xét 2n+1 điểm tùy ý không có ba điểm nào thẳng hàng A1, A2, ..., A2n+1. Giả sử các điểm này là trung điểm các cạnh của (2n+1) - giác cần dựng B1, B2, ..., B2n+1 Xét tứ giác B1B2n−1B2nB2n+1 có A2n−1A2nA2n+1 lần lượt là trung điểm các cạnh B2n−1B2n, B2nB2n+1, B2n+1B1. Gọi A là trung điểm B1B2n−1 thì AA2n−1A2nA2n+1 là hình bình hành. Vì A2n−1, A2n, A2n+1cho trước nên ta dựng được A. 19
  • 21. Hình 1.6 Xét (2n − 1) - giác B1B2...B2n−1 có (2n − 1) trung điểm các cạnh là: A1, A2, ..., A2n−2, A đã xác định nên theo giả thiết quy nạp ta dựng được đa giác này. Từ B1, B2n−1 và các trung điểm A2n−1, A2n+1 ta dựng được nốt B2n vàB2n+1. Vậy ta dựng được (2n + 1) - giác B1B2...B2n+1 khi biết trung điểm các cạnh của nó. Bài toán 1.3.17. (Chia hình bằng quy nạp) Hãy chia một lục giác thành các tam giác đen, trắng sao cho: (i) Hai tam giác tùy ý hoặc rời nhau hoặc có đỉnh chung hoặc có cạnh chung. (ii) Hai tam giác tùy ý có cạnh chung thì có màu khác nhau. (iii) Mỗi cạnh của lục giác đồng thời là cạnh của đa giác đen. Chứng minh rằng đa giác n cạnh có thể chia được theo cách trên khi và chỉ khi n ...3 Chứng minh. Đối với lục giác, có ít nhất hai cách chia theo hình sau thỏa mãn 3 điều kiện trên. Ta chứng minh n - giác được chia theo cách trên khi và chỉ khi n ...3. a) Điều kiện cần: Giả sử n - giác đã chia được thành các tam giác thỏa mãn ba điều kiện trên. Ta chứng minh n ...3. Gọi tổng số cạnh của tất cả các tam giác đen là x, tổng số cạnh của tất cả các tam giác trắng là y 20
  • 22. Hình 1.7 Vì hai tam giác chung cạnh có màu khác nhau và cạnh của đa giác luôn là cạnh của tam giác đen nên: x = y + n Mà x ...3, y ...3 nên n ...3 b) Điều kiện đủ: Giả sử n - giác T có n ...3. Ta cần chỉ ra rằng có thể chia T thành các tam giác thỏa mãn 3 điều kiện trên. Vì n ...3 nên tồn tại số nguyên dương k sao cho n = 3k. Ta chứng minh bằng quy nạp theo k. 1. Với k = 1, ta có tam giác, có thể chia như sau: Hình 1.8 2. Giả sử đối với các đa giác có n = 3k cạnh, ta đã chia được thành các tam giác thỏa mãn 3 điều kiện trên. Ta cần chứng minh đa giác có n = 3(k + 1) cạnh cũng chia tương tự như vậy. Xét đa giác có n = 3(k + 1) cạnh: A1A2...A3k+2A3k+3 Theo giả thiết quy nạp, đa giác có 3k cạnh A1A2...A3k chia được thành các tam giác thỏa mãn ba điều kiện trên. Còn lại ngũ giác A1A3kA3k+1A3k+2A3k+3 trong đó A1A3k đã là cạnh của tam giác đen, ta chia tiếp các ngũ giác này. Có nhiều cách, chẳng hạn lấy M thuộc miền trong ngũ giác, tô tam giác A1MA3k màu trắng; lúc đó lục giácA1MA3kA3k+1A3k+2A3k+3 dễ dàng chia được thành các tam giác thỏa mãn 3 điều kiện trên. Như vậy, đối với đa giác có n = 3(k + 1) cạnh ta cũng chia được theo cách trên. Bài toán được chứng minh! 21
  • 23. Hình 1.9 Bài toán 1.3.18. Chứng minh rằng: Nếu có một số tiền nguyên (nghìn) đồng Việt Nam lớn hơn 6.000đ thì luôn luôn có thể đổi ra những tờ tiền lẻ loại 2.000đ và 5.000đ. Chứng minh. Ta sẽ giải quyết bài toán này bằng phương pháp quy nạp: 1. Cơ sở quy nạp. Nếu trong túi có số tiền ít nhất, tức là 7.000đ thì ta đổi 1 tờ tiền loại 5.000đ và 1 tờ tiền loại 2.000đ. Khi đó: 7.000đ = 5.000đ + 2.000đ (1) 8.000đ = 2.000đ + 2.000đ + 2.000đ + 2.000đ (2) 2. Quy nạp. Giả sử với k(k ≥ 7.000đ) nghìn đồng ta đã đổi được sang các tờ tiền loại 5.000đ và 2.000đ Nếu ta thêm vào đẳng thức (1) và (2) tờ 2.000đ thì: 9.000đ = 7.000đ + 2.000đ (3) 10.000đ = 8.000đ + 2.000đ (4) Tiếp tục thêm 2.000đ vào 2 đẳng thức (3) và (4) thì ta có: 11.000đ = 9.000đ + 2.000đ 12.000đ = 10.000đ + 2.000đ Ta còn tiếp tục được cho bất kỳ số nguyên nào. Ta thấy rằng ở bước trước có 2 đẳng thức và suy ra ở bước sau có 2 đẳng thức. Như vậy với mọi số k nguyên đồng nào đó dù là số chẵn hay số lẻ khi k − 2 cũng sẽ đổi ra được 2 loại tiền 2.000đ và 5.000đ. Suy ra nó cũng đổi được thành 2.000đ và 5.000đ. Như vậy khẳng định của mệnh đề là đúng. 22
  • 24. 1.4 Bài tập tự giải Bài toán 1.4.1. Tính số tam giác (T(n)) của một đa giác n đỉnh được chia bởi các đường chéo không cắt nhau. Hướng dẫn. T(n) = n − 2. Tính bằng quy nạp theo số đỉnh của đa giác. Bài toán 1.4.2. Cho n + 1(n ≥ 1) số nguyên dương a0, a1, a2, ..., an. Biết rằng a1 ≥ a0, a2 = 3a1 − 2a0, a3 = 3a2 − 2a1, ..., an = 3an−1 − 2an−2. Chứng minh rằng: an > 2n−1 . Hướng dẫn. Theo giả thiết có a1 − a0 ≥ 1, ai = 2(ai−1 − ai−2) + ai−1(2 ≤ i ≤ k) Nhân vế với vế k + 1 đẳng thức trên ta suy ra ak ≥ 2k−1 . Nhờ bất đẳng thức vừa nhận được và quy nạp theo k chứng minh được ak ≥ 2k−1 . Từ đây có an ≥ 2n−1 . Bài toán 1.4.3. Trong mặt phẳng cho n đường thẳng. Biết rằng hai đường thẳng bất kỳ không song song, ba đường thẳng bất kỳ không đồng quy. Hỏi n đường thẳng này chia mặt phẳng thành bao nhiêu miền. Hướng dẫn. Dùng phương pháp quy nạp theo n, ta có kết quả số miền có được là F(n) = n2 + n + 2 2 Bài toán 1.4.4. Cho cấp số cộng vô hạn với các thành phần là các số nguyên dương. Một trong những thành phần của cấp số cộng này là số chính phương. Chứng minh rằng trong cấp số cộng này có một số vô hạn các số chính phương. Hướng dẫn. Giả sử cấp số cộng có công sai là d và một trong những thành phần chính phương của nó là a = m2 , trong đó m là một số tự nhiên. Khi đó ∀k ∈ N số bk = (m + kd)2 = m2 + 2mdk + k2 d2 = a + d(2mk + k2 d) cũng là một thành phần của cấp số cộng được xét. Bởi vậy cấp số cộng đã cho chứa một số vô hạn các thành phần là số chính phương. 23
  • 25. Bài toán 1.4.5. Chứng minh rằng với mọi số tự nhiên n, số 23n +1 chia hết cho 3n+1 và không chia hết cho 3n+2 . Hướng dẫn. Chứng minh khẳng định đúng với n = k 23k+1 = (23k )3 + 1 = (23k + 1)[(23k )2 − 23k + 1] = M.3k+1 [(23k + 1)2 − 3.23k ] = 3k+2 .M[32k+1 .M2 − 23k ] Do đó 23k+1 chia hết cho 3k+2 Chứng minh biểu thức [32k+1 .M2 − 23k ] không chia hết cho 3 32k+1 .M2 chia hết cho 3 23k ≡ ±1(mod3) hay 23k không chia hết cho 3. Nghĩa là 23k+1 không chia hết cho 3k+3 . Bài toán 1.4.6. Chứng minh rằng mỗi số tự nhiên không vượt quá n! đều phân tích được thành tổng gốm không quá n số, sao cho 2 số bất kỳ đều khác nhau và mỗi số này là ước của n!. Chứng minh. 1. Cơ sở quy nạp. Với n = 1, khẳng định hiển nhiên thỏa mãn. 2. Giả sử khẳng định đã đúng với n = k. Giả sử a là số tự nhiên tùy ý và a < (a + 1)!. Chia a cho n+1 với số dư r và thương d. Khi đó: a = d(n+1)+r, trong đó d ≤ n!, r < n + 1. Theo giả thiết quy nạp d = d1 + d2 + ... + dl, trong đó di(1 ≤ i ≤ l) là các số tự nhiên khác nhau từng đôi 1, và là ước của n!, đồng thời l ≤ n. Khi đó: a = d1(n + 1) + ... + dl(n + 1) + r và trong tổng này có không quá n +1 số khác nhau từng đôi một và đều là ước của (n + 1)! 24
  • 26. Chương 2 Phương pháp phản chứng 2.1 Phép suy luận phản chứng Phép suy luận phản chứng là quá trình ta đưa ra một giả thiết (giả thiết này đối lập với điều cần tìm) rồi đi tìm đến sự vô lý để loại trừ giả thiết ta vừa đặt ra. 2.2 Phương pháp chứng minh bằng phản chứng Phương pháp chứng minh bằng phản chứng là phương pháp sử dụng phép suy luận phản chứng để chứng minh, diễn giải những khẳng định toán học. Trong lịch sử toán học, phương pháp chứng minh bằng phản chứng đã được sử dụng từ rất sớm. Người ta sử dụng nó trong chứng minh nguyên lý Dirichlet. Bài toán 2.2.1. (Nguyên lý Dirichlet) Người ta nhốt m con thỏ vào n cái lồng, (m > n). Chứng minh rằng có ít nhất 2 con thỏ được nhốt trong cùng một lồng nào đó. Chứng minh. Giả sử mi là số con thỏ được nhốt vào lồng thứ i, (i = 1, n). Khi đó ta có m1 + m2 + ... + mn = m. Giả sử ngược lại, mỗi lồng chỉ nhốt nhiều nhất một con thỏ, tức là 0 ≥ mi ≥ 1, (i = 1, n). Khi đó m = m1 + m2 + ... + mn ≤ 1 + 1 + ... + 1 n số 1 = n Điều này mâu thuẫn với giả thiết m > n. 25
  • 27. Vậy điều ta giả sử là sai, nghĩa là phải có ít nhất 2 con thỏ được nhốt trong một lồng nào đó. 2.3 Các bước suy luận trong chứng minh phản chứng Trong toán học, phương pháp phản chứng rất thường xuyên được sử dụng, nó là công cụ đắc lực trong chứng minh một số bài toán khó. Vậy câu hỏi đặt ra là: Phương pháp chứng minh phản chứng sử dụng khi nào? Cách chứng minh phản chứng như thế nào? Phương pháp chứng minh phản chứng sử dụng khi nào? Khi gặp những bài toán khẳng định một hệ thức đúng, khẳng định nghiệm của phương trình, hệ phương trình hoặc bất đẳng thức ... trong đại số, hình học, số học, giải tích hay các bài toán không mẫu mực. Đặc biệt khi cần chứng minh tính tồn tại duy nhất của một đối tượng người ta hay dùng phản chứng để chứng minh. Nhận xét trên đây chỉ là kinh nghiệm của tác giả trong quá trình giải một số bài toán. Tùy vào từng bài toán, tình huống cụ thể khác mà người giải toán vận dụng phương pháp này một cách linh hoạt. Các bước trong chứng minh phản chứng: Ta chứng minh mệnh đề P là đúng. Bước 1. Giả sử mệnh đề P là sai (tức là chúng ta đi phủ định mệnh đề cần chứng minh). Bước 2. Từ điều giả sử ta suy ra một số tính chất hoặc quan hệ mới mà những tính chất này dẫn tới điều vô lý. Bước 3. Ta kết luận điều giả sử ban đầu là sai. Vậy mệnh đề P là mệnh đề đúng. Chú ý: Trong ba bước suy luận phản chứng nêu trên, bước 1 rất quan trọng vì chúng ta cần tạo ra mệnh đề phủ định điều cần chứng minh phải chính xác. Ở bước 2 điều vô lý có thể thuộc một trong các dạng sau: Điều trái với giả thiết đã cho Điều trái với một trong các kiến thức đã biết. Điều trái với giả thiết phản chứng đặt ra. 26
  • 28. 2.4 Vận dụng phương pháp phản chứng để giải các bài toán không mẫu mực Bài toán 2.4.1. Chứng minh tồn tại vô số số nguyên tố. Chứng minh. Giả sử ngược lại, tồn tại hữu hạn số nguyên tố p1, p2, ..., pn. Ta xét tích N = p1p2...pn + 1. Hiển nhiên N phải có ít nhất một ước số nguyên tố p nào đó. Khi đó, do p1, p2, ..., pn là tất cả các số nguyên tố nên tồn tại i, (1 ≤ i ≤ n) sao cho p = pi. Từ đó suy ra được p|1 hay p = 1 (mâu thuẫn với p là số nguyên tố). Vậy điều ta giả sử là sai. Nghĩa là có vô số số nguyên tố. Bài toán 2.4.2. Trong lớp học có 30 học sinh. Khi viết chính tả, em An phạm 13 lỗi còn các em khác ít hơn. Chứng minh rằng trong lớp có ít nhất 3 em học sinh đã mắc một số lỗi như nhau khi viết chính tả (kể cả những người không mắc lỗi nào). Chứng minh. Ta có thể vận dụng nguyên lý L. Dirichlet để giải bài toán này. Ở đây, thỏ tức là các em học sinh còn lồng là số lỗi đã phạm phải khi các em viết chính tả. Ta lập 14 lồng được đánh số từ 0 đến 13. Lồng số 0 "nhốt" các em viết chính tả phạm 0 lỗi; Lồng số 1 "nhốt" các em viết chính tả phạm 1 lỗi; Lồng số i, (0 ≤ i ≤ 13) "nhốt" các em viết chính tả phạm i lỗi; Chỉ có em An phạm 13 lỗi khi viết chính tả nên lồng số 13 chỉ có một mình em An, 29 em còn lại được "nhốt" vào các lồng từ 0 đến 12, tức là 29 em được "nhốt" vào 13 lồng, nên phải có ít nhất một lồng nhốt từ 3 em trở lên. Chẳng hạn lồng i(0 ≤ i ≤ 12) có ít nhất 3 em. Khi đó 3 trong các em ở lồng i cùng phạm i lỗi khi viết sai chính tả. Bài toán được chứng minh! Bài toán 2.4.3. Tìm các cặp số (m, n) nguyên dương sao cho m!+n! = mn . Chứng minh. Trước hết ta sẽ chỉ ra rằng các cặp số nguyên dương (m, n) muốn thỏa 27
  • 29. mãn bài toán thì m ≤ n. Phản chứng. Giả sử ngược lại, m > n. Khi đó ta có m! + n! = n![m(m − 1)...(n − 1) + 1] = mn . Do (m, m(m − 1)...(n + 1) + 1) = 1 nên đẳng thức trên không xảy ra. Vậy điều ta giả sử là sai nên m ≤ n. Xét trường hợp m > 2. Vì m! + n! = (m − 2)!(m − 1)m[1 + (m + 1)(m + 2)...(n − 1)n] nên ta có (m − 2)!(m − 1)m[1 + (m + 1)(m + 2)...(n − 1)n] = mn . Đẳng thức trên không xảy ra vì (m, m − 1) = 1 nên vế trái của đẳng thức trên chia hết cho (m − 1), còn vế phải của đẳng thức không chia hết cho (m − 1). Như vậy m ∈ 1, 2. Với m = 1 thì 1! + n! = 1. Không tồn tại số n thỏa mãn. Với m = 2 thì 2! + n! = 2n . Bây giờ ta đi tìm các giá trị của n thỏa mãn 2! + n! = 2n . Nếu n ≥ 4 thì n! = 1.2.3.4...n > 2.2.22 ...2 = 2n nên không thỏa mãn 2! + n! = 2n . Suy ra n = 2 hoặc n = 3. Thử trực tiếp ta có: 2! + 2! = 22 , 2! + 3! = 23 . Như vậy các cặp số thỏa mãn bài toán là (2, 2) và (2, 3). Bài toán 2.4.4. (Olympic Châu Á Thái Bình Dương 1998) Chứng minh rằng với mọi số a, b nguyên dương, số (36a + b)(a + 36b) không thể là một lũy thừa của 2. Chứng minh. Phản chứng. Giả sử ngược lai, tồn tại cặp số nguyên dương (a, b), sao cho (36a + b)(a + 36b) là một lũy thừa của 2. Trong số những cặp (a, b) như vậy, ta xét (m, n) là cặp có tổng m+n bé nhất. Vì (36m+n)(m+36n) là một lũy thừa của 2 nên suy ra (36m+n) 28
  • 30. và (m+36n) cũng là các lũy thừa của 2. Lúc đó m, n đều là các số chẵn. Ta cũng thấy rằng (36m + n) ≥ 37, (m + 36n) ≥ 37 Bây giờ, ta đặt 36m + n = 2r , m + 36n = 2s , m = 2p và n = 2q. trong đó r, s, p, q là các số nguyên dương. Ta có r, s > 5. Khi đó (36p + q)(p + 36q) = 36m + n 2 x m + 36n 2 = 2r+s−2 cũng là một lũy thừa của 2. Nhưng ta lại có p + q = m + n 2 < m + n Điều này mâu thuẫn với giả thiết về việc chọn cặp số (m, n) có tổng bé nhất như trên. Mâu thuẫn này chứng tỏ rằng trong các cặp (a, b) mà (36a + b)(a + 36b) là một lũy thừa của 2, ta không thể chọn được cặp số có tổng bé nhất: vô lý! Vậy số (36a + b)(a + 36b) không thể là một lũy thừa của 2. Bài toán 2.4.5. Cho 20 số tự nhiên a1, a2, ..., a20, không vượt quá 70. Chứng minh rằng trong các hiệu aj − ak(j > k) tìm được ít nhất 4 số giống nhau. Chứng minh. Phản chứng.Giả sử khẳng định trên không đúng, tức là trong các hiệu aj − ak(j > k) có không quá 3 hiệu giống nhau. Khi đó trong 19 số tự nhiên a20 − a19, a19 − a18..., a3 − a2, a2 − a1 (1) có không quá 3 hiệu giống nhau. Bởi vậy không một số nào trong các số 1, 2, 3, 4, 5, 6 trong dãy (1) lặp quá 3 lần, mà dãy (1) có 19 số, bởi vậy phải có ít nhất một trong các số thuộc (1) phải lớn hơn 6 (Trường hợp ngược lại, nếu tất cả các dãy số thuộc dãy (1) đều không quá 6, khi đó các số đều không lặp quá 3 lần, nên dãy (1) chỉ có thể có tối đa 18 số chứ không phải 19 số như thực tế). 29
  • 31. Khi đó Ít nhất 3 trong 18 số còn lại lớn hơn 5. Ít nhất 3 trong 15 số còn lại lớn hơn 4. Ít nhất 3 trong 12 số còn lại lớn hơn 3. Ít nhất 3 trong 9 số còn lại lớn hơn 2. Ít nhất 3 trong 6 số còn lại lớn hơn 1. Bởi vậy tổng các số thuộc dãy (1) a20 − a1 = (a20 − a19) + (a19 − a18) + ... + (a3 − a2) + (a2 − a1) ≥ 7 + (6 + 6 + 6) + (5 + 5 + 5) + (2 + 2 + 2) + (1 + 1 + 1) = 70 Mặt khác a20 − a1 ≤ 70 − 1 = 69 (3) So sánh (2) và (3) ta đi đến mâu thuẫn. Bởi vậy trong các hiệu aj −ak(j > k) phải có ít nhất 4 số giống nhau. Bài toán 2.4.6. (Tuyển tập 5 năm Tạp chí Toán học & Tuổi trẻ 1991 - 1995) Trong một hình vuông cạnh bằng 1 cm, lấy 51 điểm tùy ý. Chứng minh rằng luôn luôn có 3 điểm đã lấy ra nằm trong một hình tròn bán kính bằng 1 7 . Chứng minh. Bài toán được giải theo 2 bước. 1) Chia mỗi cạnh hình vuông thành 5 phần bằng nhau, rồi nối các điểm chia tương ứng đối diện bằng các đoạn thẳng song song với cạnh hình vuông. Khi đó hình vuông được chia thành 25 hình vuông nhỏ bằng nhau và có cạnh bằng 1 5 cm. Giả sử mỗi hình vuông nhỏ chứa không quá 2 điểm đã được lấy. Khi đó số điểm đã được lấy sẽ không vượt quá 50, nên nhỏ hơn 51 điểm. Ta đã đi tới mâu thuẫn với giả thiết: Các điểm đã được lấy bằng 51. Bởi vậy phải có ít nhất một hình vuông con chứa không ít hơn 3 điểm đã lấy. Giả sử hình vuông con ở góc trên tận cùng bên phải chứa 3 điểm đã chọn ra. Ta ký hiệu hình vuông này bằng ABCD. 2) Bao hình vuông ABCD bằng hình tròn bán kính 1 7 cm. Hình vuông ABCD có hai đường chéo cắt nhau tại điểm giữa của mỗi đường và vuông góc với nhau. Dùng O để ký hiệu giao điểm hai đường chéo và x là độ dài nửa đường chéo. 30
  • 32. Tam giác AOB vuông tại O có cạnh huyền AB dài 1 5 . Khi đó ta có OA 2 + OB 2 = 2x2 = AB 2 = 1 5 2 = 1 25 nên x2 = 1 50 < 1 49 = 1 7 2 Do đó x < 1 7 cm. Bởi vậy, ta có thể bao hình vuông ABCD bằng hình tròn bán kính 1 7 Hình 2.4.6 cm, nên ba điểm đã chọn ra thuộc hình vuông ABCD nằm trong hình tròn bán kính 1 7 cm. Bài toán 2.4.7. Cho ba số a, b, c ∈ (0, 1). Chứng minh rằng ba số a(1 − a), b(1 − b), c(1 − c) không thể đồng thời lớn hơn 1 4 . Chứng minh. Phản chứng. Ta giả sử ngược lại, cả ba số cùng lớn hơn 1 4 , tức là a(1 − b) > 1 4 , b(1 − c) > 1 4 , c(1 − a) > 1 4 Nhân vế với vế của các bất đẳng thức đó, ta được a(1 − b)b(1 − c)c(1 − a) > 1 64 (1) 31
  • 33. Mặt khác, từ giả thiết a ∈ (0, 1) ta có 0 < a(1 − a) = 1 4 − (a − 1 2 )2 ≥ 1 4 (2) Tương tự, ta có 0 < b(1 − b) ≥ 1 4 ; 0 < c(1 − c) ≥ 1 4 (3) Từ (2) và (3) suy ra a(1 − b)b(1 − c)c(1 − a) ≤ 1 64 (4) Ta thấy (1) và (4) mâu thuẫn nhau, chứng tỏ điều giả sử ban đầu là sai. Vậy ta có điều phải chứng minh. Bài toán 2.4.8. Cho n+1 số tự nhiên khác nhau và nhỏ hơn 2n. Chứng minh rằng từ các số này luôn luôn chọn ra được ba số, mà một trong ba số này bằng tổng của hai số còn lại Chứng minh. Giả sử n + 1 số đã cho là a1, a2, ..., an+1 và a1 < a2 < ... < an < an+1 Ta lập n hiệu từ các số đã cho h1 = a2 − a1, h2 = a3 − a1, ..., hn = an+1 − a1 Vì a1 nhỏ hơn tất cả các số còn lại, nên tất cả các hiệu hi(1 ≤ i ≤ n) đều dương. Do các số a2, a3, ..., an+1 khác nhau từng đôi một, nên hiệu h1, h2, ..., hn đều khác nhau. Vì an+1 < 2n, nên tất cả các hiệu hi(1 ≤ i ≤ n) đều nhỏ hơn 2n. Như vậy, ta có cả thảy 2n + 1 số nguyên dương nhỏ hơn 2n. a1, a2, ..., an, an+1, h1 = a2 − a1, h2 = a3 − a1, ..., hn = an+1 − a1 nên, theo nguyên lý phản chứng, phải có hai số bằng nhau. Nhưng các số ai(1 ≤ i ≤ n + 1) và các số ht(1 ≤ t ≤ n) đều khác nhau, nên chỉ có thể có một số ak nào đó phải bằng một số hs, tức ak = as − a1 Từ đó ta có: as = ak + a1 và khẳng định được chứng minh. 32
  • 34. Bài toán 2.4.9. Chứng minh rằng từ một tập bất kỳ gồm n số tự nhiên đều có thể tách ra một tập con (không trống), mà tổng của các số thuộc tập con này chia hết cho n. Chứng minh. Giả sử A = {a1, a2, ..., an} là tập các số tùy ý đã chọn ra. Phản chứng. Giả sử khẳng định của đề bài không đúng (*), nghĩa là không có một tập con nào gồm các phần tử thuộc tập A, mà tổng chia hết cho n. Khi đó trong n tổng S1 = a1, S2 = a1 + a2, ..., Sn = a1 + a2 + ... + an−1 + an không có một tổng nào chia hết cho n. Bởi vậy, khi chia n tổng S1, S2, ..., Sn cho n chỉ nhận được tối đa n − 1 loại số dư. Bởi vậy, theo nguyên lý phản chứng, phải có ít nhất hai trong các số trên chia cho n có cùng số dư. Chẳng hạn Si, Sj với j > i có cùng số dư khi chia cho n. Do đó ai+1 + ai+2 + ... + aj = Sj − Si chia hết cho n. Ta đã chứng tỏ khẳng định (*) là sai, nên khẳng định của đề bài là đúng. Bài toán 2.4.10. Gọi a là số lớn nhất của các hình tròn rời nhau (không có điểm chung) có đường kính bằng 1 và tâm nằm trong đa giác F cho trước, còn b là số nhỏ nhất những hình tròn có bán kính bằng 1 mà bằng chúng có thể phủ kín đa giác F. Hỏi a ≥ b hay a < b? Chứng minh. Bằng phản chứng ta sẽ khẳng định được a ≥ b. Do a là số lớn nhất các hình tròn rời nhau (từng cặp không có điểm chung) có đường kính bằng 1, mà tâm nằm trong đa giác F, nghĩa là trên đa giác F không còn một khoảng nào có thể đặt gọn được một hình tròn có đường kính bằng 1 nữa. Ta lấy tâm của mỗi hình tròn này làm tâm của một hình tròn mới đường kính bằng 2, thì tập tất cả các hình tròn này sẽ phủ kín hình F. Giả sử ngược lại, có điểm O nào đó thuộc F, nhưng không nằm trên một hình tròn nào trong các hình tròn ta vừa vẽ. Khi đó O cách tất cả các tâm của hình tròn bán kính bằng 1 2 một khoảng lớn hơn 1, nên khi lấy O làm tâm ta có thể đặt thêm 1 hình tròn đường kính bằng 1 vào hình F, mà nó không trờm lên bất kỳ một hình tròn bán kính 1 2 nào. 33
  • 35. Như vậy, ta có thể đặt không phải a, mà ít nhất a + 1 hình tròn đường kính đơn vị vào hình F, mà chúng vẫn rời nhau. Như vậy ta đã đi tới mâu thuẫn với giả thiết: a là số lớn nhất các hình tròn rời nhau có đường kính bằng 1 và tâm nằm trong đa giác F. Từ đó suy ra a ≥ b. Chú ý. 1) Nếu F là hình vuông cạnh bằng 1 2 , thì a = b. Khi đó mỗi loại có một hình tròn, tâm của hai hình tròn này chung nhau và chung với tâm hình vuông. 2) Nếu F là hình vuông cạnh bằng 1, thì ta cần phủ bằng một hình tròn có bán kính bằng 1, tâm trùng với tâm của hình vuông nhưng lại có thể vẽ được tâm của 4 hình tròn bán kính bằng 1 2 với tâm ở 4 đỉnh. Vậy khi đó ta có a = 4 > 1 = b Hình 2.4.12 Bài toán 2.4.11. Gọi a là số lớn nhất các hình tròn rời nhau (không có điểm chung), có bán kính bằng 1 và tâm thuộc hình F, còn b là số nhỏ nhất các hình tròn bán kính bằng 1, mà các hình tròn này có thể phủ kín hình F. Hỏi a ≤ b hay a > b? Chứng minh. Bằng phản chứng ta có thể khẳng định được a ≤ b. Giả sử a > b Gọi C1, C2, ..., Ca là tập hợp nhiều nhất các hình tròn rời nhau có tâm tại O1, O2, ..., Oa thuộc hình F và có bán kính bằng 1. Gọi D1, D2, ..., Db là tập ít nhất các hình tròn bán kính bằng 1 và phủ kín hình F. Ta cắt các hình tròn D1, D2, ..., Db bằng giấy xanh rồi đem phủ kín lên hình F. Sau đó, ta cắt các hình tròn C1, C2, ..., Ca bằng giấy đỏ rồi đặt chúng lên hình F, sao cho chúng rời nhau. 34
  • 36. Vì a > b, nên số hình tròn C1, C2, ..., Ca có tâm nằm nằm trên hình F nhiều hơn số hình tròn D1, D2, ..., Db có ít nhất một phần nằm trên hình F (Vì b là số ít nhất các hình tròn phủ hình F, nên những hình tròn không có phần phủ F đã bị loại bỏ). Bởi vậy sẽ có hình tròn Di(1 ≤ i ≤ b) nào đó chứa tâm của hai hình tròn Cj, Ck(k = j) nào đó. Khi đó do các hình tròn D1, D2, ..., Db đều có bán kính bằng 1, nên khoảng cách giữa Oj và Ok nhỏ hơn 2 (OjOk < 2). Do đó hai hình tròn Cj, Ck không rời nhau. Ta đi tới mâu thuẫn với tính chất rời nhau của các hình tròn C1, C2, ..., Ca. Bởi vậy phải loại bỏ giả thiết a > b mà kết luận a ≤ b. Bài toán 2.4.12. Cho 7 đoạn thẳng. Độ dài mỗi đoạn lớn hơn 10cm, bé hơn 100cm. Chứng minh rằng luôn luôn tồn tại ba đoạn thẳng là các cạnh của một tam giác. Chứng minh. Dùng ai(1 ≤ i ≤ 7) để ký hiệu 7 đoạn thẳng đã cho và giả sử độ dài của chúng được xếp theo thứ tự: 10cm < a1 ≤ a2 ≤ a3 ≤ a4 ≤ a5 ≤ a6 ≤ a7 < 100cm Ta biết rằng ba đoạn thẳng là cạnh của một tam giác khi và chỉ khi tổng độ dài của hai đoạn tùy ý lớn hơn độ dài đoạn còn lại và trị tuyệt đối của hiệu độ dài hai cạnh tùy ý không lớn hơn độ dài cạnh còn lại. Phản chứng. Giả sử không có ba đoạn thẳng nào trong các đoạn thẳng đã cho lập thành một tam giác. Khi đó bất kỳ bộ ba i, j, k nào mà 1 ≤ i; j, k ≤ 7, thì ai + aj ≤ ak, nên có: a1 + a2 ≤ a3, a2 + a3 ≤ a4, a3 + a4 ≤ a5, a4 + a5 ≤ a6, a5 + a6 ≤ a7 Bởi vậy: 100cm ≥ a7 ≥ a5 + a6 ≥ 2a5 + a4 ≥ 2(a3 + a4) + a4 = 2a3 + 3a4 ≥ ≥ 2a3 + 3(a2 + a3) = 5a3 + 3a2 ≥ 5(a2 + a1) + 3a2 = 8a2 + 5a1 ≥ ≥ 8a1 + 5a1 = 13a1 ≥ 13.10cm = 130cm Ta đi tới mâu thuẫn, nên phải tồn tại ít nhất một bộ ba p, q, r ∈ N+ (1 ≤ p < q < r ≤ 7) để ap + aq > ar. Từ đó ap + ar > ar > aq, aq + ar > ar > ap. Khi đó bộ ba ap, aq, ar lập thành một tam giác. Khẳng định được chứng minh. 35
  • 37. Bài toán 2.4.13. Chứng minh rằng một đa giác lồi có 2015 cạnh không thể chia ra thành hình bình hành. Chứng minh. Phản chứng. Ta giả sử ngược lại, có một đa giác lồi 2015 cạnh có thể chia cắt thành những hình bình hành. Khi đó mỗi cặp cạnh của đa giác sẽ là đôi một song song cùng nhau (ba cạnh không thể song song cùng nhau vì đa giác là lồi). Nhưng trong trường hợp như vậy, đa giác sẽ có số cạnh là số chẵn. Giả thiết bài toán cho cạnh của đa giác lồi là 2015 (là một số lẻ) nên không thỏa mãn. Suy ra điều ta giả sử là sai. Vậy đã giác lồi có 2015 cạnh không thể chia ra thành những hình bình hành. Bài toán 2.4.14. Bảy đa giác có diện tích bằng nhau và bằng 1 nằm trong một hình vuông có độ dài cạnh là 2. Chứng minh rằng có ít nhất hai đa giác cắt nhau với diện tích phần chung không nhỏ hơn 1 7 . Chứng minh. Ta kí hiệu: Pi, i = 1, 7 là bảy đa giác có diện tích bằng 1; V là hình vuông có diện tích bằng 4 chứa bảy đa giác trên. Phản chứng. Giả sử mệnh đề: "Ít nhất có hai đa giác cắt nhau với diện tích chung không nhỏ hơn 1 7 " là sai. Kí hiệu SA là diện tích của hình A. Ta có SPi∩Pj < 1 7 ∀i, j : 1 ≤ i < j ≤ 7. Từ đây suy ra SPi∪Pj = SP1 + SP2 − SP1∩P2 > 1 + 1 − 1 7 . Hơn nữa SP1∪P2∪P3 = SP1∪P2 + SP3 − S(P1∪P2)∩P3 = SP1∪P2 + SP3 − S(P1∩P3)∪(P2∩P3) ≥ SP1∪P2 + SP3 − SP1∩P3 − SP2∩P3 > (2 − 1 7 ) + 1 − ( 1 7 + 1 7 ) = 3 − ( 1 7 + 2 7 ). 36
  • 38. Lặp lại cách làm trên ta nhận được SP1∪P2∪P3∪P4∪P5∪P6∪P7 > 7 − ( 1 7 + 2 7 + ... + 6 7 ) = 4. Điều này không thể xảy ra vì các đa giác nằm trong hình vuông V có diện tích bằng 4. Vậy phải có ít nhất hai hình đa giác cắt nhau mà phần chung diện tích lớn hớn hoặc bằng 1 7 . 2.5 Bài tập tự giải Bài toán 2.5.1. Chứng minh rằng từ 8 số tự nhiên tùy ý luôn luôn chọn ra được 2 số, mà hiệu của chúng chia hết cho 7. Hướng dẫn. Chia 8 số tự nhiên tùy ý đã chọn ra cho 7 được 8 số dư tương ứng, nhưng chỉ thuộc 7 loại: 0, 1, 2, 4, 5, 6 nên phải có ít nhất 2 số dư bằng nhau. Khi đó hiệu của 2 số tương ứng chia hết cho 7. Bài toán 2.5.2. Chứng minh với mọi số nguyên a, b, c luôn tìm được số nguyên dương n, sao cho số f(n) = n3 + an2 + bn + c không phải là số chính phương. Hướng dẫn. Nhận xét rằng khi chia cho 4, số chính phương chỉ có thể dư 0 hoặc dư 1. Phản chứng. Giả sử f(n) là số chính phương. Khi đó f(4) − f(2) ≡ 2b(mod 4) Mà 2b là số chẵn, theo nhận xét thì f(4) − f(2) chỉ có thể đồng dư với 0, 1, −1 theo modun 4, nên suy ra 2b ≡ 0(mod 4) f(3) − f(3) ≡ (2b + 2)(mod 4) Tương tự trên ta cũng có 2b + 2 ≡ 0(mod 4) nên suy ra 2 ≡ 0(mod 4) (vô lý). Vậy điều giả sử là sai. Bài toán 2.5.3. Chứng minh không số hữu tỷ nào là nghiệm của phương trình x3 + x + 1 = 0. 37
  • 39. Hướng dẫn. Phản chứng. Giả sử ngược lại, có số hữu tỷ p q (q = 0, (p, q) = 1, p, q ∈ Z) là nghiệm của phương trình. Khi đó p3 q3 + p q + 1 = 0 ⇒ p3 + p.q2 + q3 = 0 Xét các trường hợp p, q cùng lẻ, p, q cùng chẵn, p chẵn, q lẻ hoặc p lẻ, q chẵn để chỉ ra vô lý của điều giả sử. Bài toán 2.5.4. Chứng minh rằng từ n số tự nhiên bất kỳ a1, a2, ..., an đều có thể tách ra một nhóm, gồm ít nhất một số, mà tổng tất cả các số thuộc nhóm này chia hết cho n. Hướng dẫn.Xét n tổng S1 = a1, S2 = a1 + a2, ..., Sn = a1 + a2 + ... + an (1) 1) Nếu một trong những tổng thuộc dãy (1) chia hết cho n, thì khẳng định được chứng minh. 2) Các tổng thuộc dãy (1) đều không chia hết cho n. Khi đó chia tất cả các số thuộc dãy (1) cho n, được n số dư, nhưng chỉ thuộc tối đa n − 1 loại. Khi đó theo nguyên lý phản chứng, phải có ít nhất 2 số dư bằng nhau. Giả sử j > i và Si, Sj có cùng số dư khi chia cho n. Khi đó: A = Sj − Si = ai+1 + ai+2 + ... + aj chia hết cho n. Vậy khẳng định được chứng minh. Bài toán 2.5.5. Cho 9 đường thẳng. Mỗi đường đều chia hình vuông thành 2 tứ giác với tỷ số diện tích bằng 2 3 . Chứng minh rằng có ít nhất 3 trong 9 đường thẳng này phải đi qua 1 điểm. Hướng dẫn. Mỗi đường trung bình của hình vuông có 2 điểm chia nó theo tỷ lệ 2 3 . Do đó trong hình vuông có 4 điểm chia đường trung bình theo tỷ lệ 2 3 . Mặt khác đường thẳng bất kỳ cắt hình vuông thành 2 hình thang hoặc 2 hình chữ nhật với tỷ lệ diện tích 2 3 . 38
  • 40. Chương 3 Phương pháp suy luận 3.1 Vài nét về phương pháp suy luận Các bài toán không mẫu mực (không có cách giải nhất định), thường có nhiều cách giải khác nhau, trong đó có phương pháp suy luận. Phương pháp suy luận đã có từ xa xưa và để giải các bài toán logic người ta chỉ có duy nhất phương pháp này (sau này mới có thêm các phương pháp khác). Các bài toán không mẫu mực đa dạng về đề tài, phong phú về chủng loại đòi hỏi chúng ta phải biết suy luận đúng đắn, chặt chẽ trên cơ sở vận dụng những kiến thức cơ bản và kinh nghiệm sống của mình. Vì vậy, cần phải luyện tập óc quan sát, cách lập luận, cách xem xét các khả năng có thể xảy ra của một sự kiện và vận dụng những kiến thức đã học vào các tình huống muôn hình muôn vẻ trong cuộc sống hàng ngày. Đôi khi để giải những bài toán loại này, chỉ cần những kiến thức toán học đơn giản nhưng lại đòi hỏi khả năng chọn lọc trường hợp, suy luận chặt chẽ, rõ ràng. Sự phát triển của toán học, chẳng hạn giải tích tổ hợp, phương pháp quy nạp, phản chứng góp phần phong phú thêm phương pháp suy luận logic trực tiếp. Và nhờ những kiến thức toán học này người ta có thể giải các bài toán không mẫu mực một cách nhanh hơn, tốt hơn và chặt chẽ hơn. Điều cơ bản của phương pháp này là thông qua việc phân tích các điều kiện của bài toán cần tìm ra mối quan hệ logic giữa các mệnh đề.Sau đây là một vài ví dụ về vận dụng phương pháp suy luận trực tiếp. 39
  • 41. 3.2 Các ví dụ về vận dụng phương pháp suy luận. Bài toán 3.2.1. (Tạp chí Toán Học & Tuổi Trẻ số 379, tháng 1 - 2009) Hãy tưởng tượng bạn đang tham gia trò chơi "Chiếc nón kỳ diệu" trên truyền hình và bạn đang có cơ hội nhận được một phần quà có giá trị. Phần quà này nằm ở 1 trong 3 chiếc hộp (hai chiếc hộp còn lại rỗng). Giả sử bạn đã chọn chiếc hộp nào đó nhưng chưa mở ra, lúc này người dẫn chương trình (là người biết rõ hộp nào chứa phần quà) sẽ mở 1 chiếc hộp rỗng trong hai chiếc còn lại ra (nếu 2 hộp đều rỗng thì chọn hộp bất kỳ). Sau đó anh ta hỏi bạn có đổi chiếc hộp vừa chọn lấy chiếc hộp còn lại chưa mở không? Hỏi rằng nếu bạn đồng ý thì xác suất chọn đúng chiếc hộp có quà là bao nhiêu? Theo tôi nghĩ khi người dẫn chương trình bỏ đi một chiếc hộp rỗng thì sẽ còn lại 2 chiếc hộp. Nên nếu đồng ý đổi thì xác suất chọn đúng hộp có quà tặng là 1 2 . Bạn có nghĩ như tôi không? Chứng minh. Rõ ràng xác suất chọn đúng hộp có quà tặng bằng 1 2 là không đúng. Có thể thấy rằng xác suất để chiếc hộp chứa quà mà người chơi chọn ban đầu (khi có 3 hộp) là 1 3 . Xác suất để một trong hai chiếc hộp còn lại có quà là 2 3 . Khi người dẫn chương trình loại đi một chiếc hộp rỗng thì chiếc hộp còn lại (mà người chơi không chọn) có xác suất chứa quà là 2 3 . Do đó xác suất có chứa quà trong hai chiếc hộp còn lại là khác nhau. Vì vậy nếu người chơi đồng ý đổi chiếc hộp đã chọn với chiếc hộp còn lại thì xác suất chọn đúng hộp quà của người chơi sẽ tăng lên (= 2 3 ). Như vậy nếu người dẫn chương trình tạo một cơ hội như trên thì người chơi không nên bỏ lỡ. Bài toán 3.2.2. Có 12 đội bóng tham gia tranh giải. Mỗi đội phải đấu với nhau một trận: Thắng được 1 điểm, hòa được 1 2 điểm, thua không được điểm. Biết rằng số điểm của đội đứng thứ 2 trong giải bằng tổng số điểm của 5 đội đứng cuối cùng trong giải và số điểm của đội đứng thứ 9 không nhỏ hơn tổng số điểm của 3 đội đứng cuối cùng trong giải. Đồng thời không có ba đội nào cùng bằng điểm nhau. Hỏi kết quả của 40
  • 42. trận giữa đội đứng thứ 8 và đội đứng thứ 9 như thế nào? Chứng minh. Đội thứ nhất đấu 11 trận với 11 đội còn lại và số điểm không vượt quá 11 điểm. Do đó, đội thứ 2 có số điểm không quá 10,5 điểm. Nếu đội thứ 2 được 10,5 điểm, thì đội thứ hai hòa 1 trận và thắng 10 trận, nghĩa là trận đấu giữa đội xếp thứ nhất và đội xếp thứ 2 hòa nhau vì thế đội xếp thứ nhất chỉ đạt số điểm nhiều nhất là 10,5 điểm, nhưng như vậy có hai đội đạt cùng một điểm (trái với giả thiết). Vậy số điểm của đội thứ 2 không vượt quá 10 điểm (1). Năm đội cuối cùng đã đấu với nhau số trận là 5 × 4 2 = 10 trận và đạt tổng số điểm là 10 điểm, nên tổng số điểm của 5 đội xếp cuối cùng phải lớn hơn hoặc bằng 10 điểm (2). Vì đội thứ hai đạt số điểm bằng số điểm 5 đội xếp cuối bảng, nên từ (1) và (2) ta có số điểm của 5 đội xếp cuối cùng là 10 điểm. Nghĩa là các đội này chỉ đạt điểm khi họ đấu với nhau còn họ thua tất cả các đội còn lại, nghĩa là mỗi đội trong số 5 đội này đều thua các đội còn lại. Kí hiệu A, B, C, D, E lần lượt là số điểm của đội thứ 8, thứ 9, thứ 10, thứ 11, thứ 12. Ta đã biết A + B + C + D + E = 10. Nếu ba đội đấu với nhau thì bao giờ cũng có tổng số điểm là 3, nên nếu kể cả điểm của ba đội đó đạt được khi thi đấu với 2 dội còn lại thì ta có: C +D +E ≥ 3 (3). Theo giả thiết B ≥ C + D + E (4), suy ra B ≥ 3. Đội đứng thứ 8 có số điểm A ≤ 4. Từ đó suy ra A = 4 (Vì nếu A = 3 × 5 thì B = 3 × 5), B = C + D + E = 3. Vậy đội đứng thứ 8 thi đấu 4 trận với 4 đội xếp cuối cùng, đạt 4 điểm nên đội đứng thứ 8 đã thắng đội đứng thứ 9. Bài toán 3.2.3. (Tạp chí Toán Học & Tuổi Trẻ số 388, tháng 10 năm 2009) Trong một cuộc họp gồm 56 đại biểu. Biết rằng mỗi đại biểu đều quen biết với không ít hơn 8 đại biểu khác (quy ước rằng A quen B thì B quen A). Chứng minh rằng ban tổ chức có thể sắp xếp ít nhất một bàn gồm 4 đại biểu cùng bàn với nhau sao cho mỗi đại biểu quen ít nhất 2 trong số 3 đại biểu còn lại. Chứng minh. Xét 1 đại biểu A bất kỳ. Kí hiệu A1, A2, ..., An là tất cả các đại biểu quen với A. Theo giả thiết n ≥ 8 và mỗi Ai(i = 1, n) ngoài A ra đều quen ít nhất 7 đại biểu khác. Do đó nếu với mỗi i = 1, n ta 41
  • 43. liệt kê tất cả các đại biểu khác A và quen với A1 thì tổng số đại biểu liệt kê ra được kể cả lặp không ít hơn 7n ≥ 56. Mà ngoài A chỉ còn 55 đại biểu khác nhau nên suy ra có ít nhất một đại biểu được liệt kê ít nhất hai lần. Nghĩa là phải có hai đại biểu Ai, Aj(i = j, j ∈ {1, 2, ..., n}) cũng quen với đại biểu B, khác A (có thể B ∈ {A1, A2, ..., An}). Hiển nhiên khi xếp A1, Ai, Aj, B cùng bàn ta sẽ được chiếc bàn thỏa mãn đề bài. Bài toán 3.2.4. (Tạp chí Toán Học & Tuổi Trẻ, số 5 năm 1992) Trong dịp hè Cường, Hùng, Dũng tham gia câu lạc bộ thể dục thể thao, luyện tập 6 môn bơi, bóng bàn, bóng đá, võ dân tộc, cờ quốc tế và cầu lông.Mỗi người luyện tập 2 môn và không có 2 người nào luyện tập cùng một môn. Hãy xác định xem ai luyện tập những môn nào? Biết rằng 1) Bạn luyện tập cờ quốc tế và bạn luyện tập bóng đá là đôi bạn thân từ nhỏ. 2) Bạn Cường cao to nhất bọn. 3) Bạn Dũng, bạn tạp bóng bàn và bạn tập bóng đá thường rủ nhau đến câu lạc bộ. 4) So với bạn học võ dân tộc, bạn chơi bóng bàn thấp hơn chút ít. 5) Ba bạn: bạn tập bơi, bạn học võ dân tộc và bạn Cường cùng tham dự kỳ thi học sinh giỏi toán năm vừa qua. Chứng minh. Từ giả thiết (2) và (4) ta suy ra bạn Cường không tập bóng bàn. Từ (3) ta có Dũng cũng không tập bóng bàn. Vậy bạn Hùng tập bóng bàn (6) và bạn Cường tập bóng đá (7) Từ giả thiết (4) và (6) suy ra Hùng không tập võ dân tộc. Từ (5) ta có Cường không tập võ dân tộc. Vậy Dũng tập võ dân tộc (8). Từ (5) và (8) suy ra Hùng tập bơi (9). Từ (1) và (7) suy ra Cường không học chơi cờ quốc tế. Từ (6) và (9) ta cũng có Hùng không học chơi cờ quốc tế. Vậy Dũng chơi cờ quốc tế và Cường tập cầu lông. Kết luận: Cường tập cầu lông và bóng đá. Dũng học võ dân tộc và chơi cờ quốc tế. Hùng tập bóng bàn và tập bơi. Bài toán 3.2.5. Có một bảng gồm 3×3 ô vuông và 9 cái thẻ kích thước bằng một ô vuông có viết các số nào đó. Hai người chơi lần lượt đặt từng thẻ một lên các ô của bảng. Sau khi đặt hết các thẻ phủ kín bảng, người thứ nhất (người đặt thẻ đầu tiên) tính tổng của 6 ô đứng ở dòng trên 42
  • 44. cùng và dưới cùng, người thứ 2 tính tổng của 6 ô đứng ở cột bên trái và cột bên phải. Ai có tổng của mình lớn hơn là người thắng cuộc. Chứng minh rằng nếu người thứ nhất chơi đúng thì người thứ hai sẽ không thể thắng cuộc. Chứng minh. Rõ ràng kết quả của trò chơi chỉ phụ thuộc vào các số đứng ở ô số 1, ô số 2, ô số 3, ô số 4. Người thứ nhất thua nếu tổng các chữ số ở ô số 1 và ô số 3 nhỏ thua tổng các chữ số ở ô số 2 và ô số 4. Ta hãy đề ra chiến thuật người thứ nhất chơi không thua cuộc. Giả sử các số ghi trên các thẻ là a1, a2, ..., a9 thỏa mãn a1 ≤ a2 ≤ a3 ≤ ... ≤ a9. Hình 3.2.5 - Nếu a1 + a9 ≥ a2 + a8, khi đó người thứ nhất đặt a9 ở ô số 1 và lần thứ hai đặt a2 (hoặc a1) vào ô số 2 (hoặc ô số 4). Rõ ràng tổng ở các ô số 1 và số 3 sẽ không bé thua a1 + a9, còn tổng các chữ số của ô số 2 và ô số 4 không vượt quá a2 + a8. - Nếu a1 + a9 < a2 + a8, khi đó trong lần đầu đi đầu tiên người thứ nhất đặt a1 vào ô số 2 và lần đi thứ 2 đặt a8 (hoặc a9) vào ô số 1 hoặc ô số 3. Rõ ràng tổng các số ở ô số 1 và ô số 3 không nhỏ thua a2 + a8 còn tổng các số ở ô số 2 và ô số 4 không vượt quá a1 + a9. Như vậy trong cả hai trường hợp thì người thứ nhất không thua, nghĩa là người thứ 2 không thể thắng cuộc. Tất nhiên kết quả này không phụ thuộc vào số viết trên thẻ. Bài toán 3.2.6. Có 3 trường học, mỗi trường học có n học sinh. Mỗi học sinh bất kỳ có tổng số người quen từ hai trường kia là n + 1. Chứng minh rằng có thể chọn ở mỗi trường một học sinh, sao cho ba học sinh này quen lẫn nhau. Chứng minh. Trong số tất cả 3n học sinh chọn ra một học sinh có số người quen lớn nhất (k) từ một rong hai trường học khác (hay một từ các học sinh như vậy). Giả sử đó là học sinh A từ trường số 1 mà học sinh này quen k học sinh, ví dụ ở trường số 2. Khi đó A quen với n+1−k học sinh ở trường số 3, ngoài ra n + 1 − k ≥ 1 vì k ≤ n. 43
  • 45. Xét học sinh B của trường số 3 quen với A. Nếu B quen với một học sinh C trong số k người quen của A ở trường số 2, thì các học sinh A, B, C tạo thành bộ ba cần tìm. Nếu B không quen C trong số k người quen của A ở trường số 2 thì trong trường hợp này B sẽ quen với không nhiều hơn n − k học sinh ở trường số 2. Nghĩa là trong trường hợp số 1, B quen với không ít (n + 1) − (n − k) = k + 1 học sinh. Điều này trái với việc chọn số k. Suy ra khẳng định được chứng minh. Bài toán 3.2.7. Ai cao ai thấp? Có 200 học sinh xếp hàng 10. Lần thứ nhất, theo mỗi hàng dọc, người ta chọn ra một học sinh cao nhất. Sau đó trong số 10 học sinh được chọn ấy người ta lại chọn ra một học sinh thấp nhất. Lần thứ hai, theo mỗi hàng ngang, người ta chọn ra một học sinh thấp nhất. Sau đó trong số 20 học sinh được chọn ấy người ta lại chọn ra 1 học sinh cao nhất. Ai trong số 2 học sinh được chọn ra cao hơn, biết rằng sau lần chọn thứ nhất các học sinh lại về vị trí cũ. Chứng minh. Gọi A là học sinh được chọn trong lần chọn thứ nhất và B là học sinh được chọn trong lần chọn thứ hai. Gọi C là học sinh đứng cùng hàng dọc với A và cùng hàng ngang với B (như hình vẽ). Vì A là học sinh cao nhất trong hàng dọc của mình nên A cao hơn C: h(A) ≥ h(C) (1). Vì B là học sinh thấp nhất trong hàng ngang của mình nên B thấp hơn C:h(B) ≤ h(C) (2), với h() là kí hiệu chiều cao của mỗi học sinh. Từ (1) và (2) suy ra h(A) ≥ h(B), nghĩa là học sinh được chọn lần đầu không thấp hơn học sinh được chọn lần hai. Dấu "=" xảy ra ở cả hai đẳng thức (1) và (2) khi và chỉ khi A cao bằng B. Bài toán 3.2.8. (Tạp chí Toán học & tuổi trẻ, tháng 10 năm 2007) Ai bắng trúng vòng mười? Ba anh bộ đội Mạnh, Hùng, Dũng tập bắn bia. Mỗi người bắn ba phát vào cùng một bia. Kết quả như sau: Tất cả các vòng bia từ 2 đến 10 đều 44
  • 46. được bắn trúng và ba người đều có tổng số điểm bằng nhau. Biết rằng Hùng bắn trúng vòng 4, Dũng bắn trứng vòng 7. Các bạn hãy xem xét xem ai đã bắn trúng vòng 10 và mỗi người đã bắn trúng những vòng nào? Chứng minh. Vì có 9 phát bắn và cả 9 vòng bia đều được bắn trúng nên mỗi vòng bia từ 2 đến 10 chỉ có một phát trúng. Như vậy tổng số điểm của cả ba người là 2 + 3 + 4 + 5 + 6 + 7 + 8 + 9 + 10 = 54 Số điểm mỗi người đạt được là 54 : 3 = 18. Vì Hùng bắn trúng vòng 4 nên Hùng có thể bắn trúng vòng 5, 4, 9 hoặc 4, 6 ,8. Vì Dũng bắn trúng vòng 7 nên Dũng có thể bắn trúng vòng 3, 8, 7 hoặc 7, 2 ,9. Vậy Mạnh bắn trúng vòng 10 và kết quả có thể theo các khả năng như bảng sau. Bài toán 3.2.9. Nhập học vào lớp 10 năm nay có 50 em biết nhạc, 50 em biết đá bóng và 50 em biết đánh bóng bàn (một em có thể biết nhiều môn). Hãy chia các em học sinh khối 10 thành 5 lớp (có thể sĩ số của các lớp khác nhau) sao cho mỗi lớp có đúng 10 em biết nhạc, 10 em biết đá bóng và 10 em biết đánh bóng bàn. Chứng minh. 1. Chia 50 em biết nhạc thành 50 nhóm A1, A2, ..., A49, A50 2. Bổ sung người biết đá bóng (nếu Ai có em biết đá bóng thì không bổ sung nữa) để được các nhóm B1, B2, ..., B49, B50 (Bi có đúng một em biết nhạc, một em biết đá bóng). 3. Bổ sung các em biết bóng bàn. Ta bổ sung các em biết đánh bóng bàn chưa tham gia nhóm Bi vào các nhóm Bi sao cho mỗi nhóm nhận được có không quá 2 m biết đánh 45
  • 47. bóng bàn. Sau 3 bước ta chọn được các nhóm C1, C2, ..., C49, C50 (mỗi nhóm Ci, 1 ≤ i ≤ 50 có đúng một em biết nhạc, một em biết đá bóng và không quá 2 em biết đánh bóng bàn.) Gọi số nhóm Ci có 2 em biết đánh bóng bàn là x. Gọi số nhóm Ci không có em nào biết đánh bóng bàn là y. Gọi số nhóm Ci có 1 em biết đánh bóng bàn là z. Suy ra số nhóm Ci là x + y + z = 50 Số em biết đánh bóng bàn là 2x + 0y + z = 50 Suy ra hệ: x + y + z = 50 2x + z = 50 ⇒ x = y z chẵn Vì số nhóm không có em nào biết đánh bóng bàn bằng số nhóm có hai em biết đánh bóng bàn và số nhóm có một em biết đánh bóng bàn là số chẵn nên ta nhập một nhóm không em nào biết đánh bóng bàn với một nhóm có hai em biết đánh bóng bàn. Nhập hai nhóm có một em biết đánh bóng bàn lại với nhau. Suy ra có 25 nhóm như vậy và ký hiệu là: D1, D2, ..., D25. Mỗi nhóm Di có đúng hai em biết đánh bóng bàn, hai em biết nhạc và hai em biết đá bóng. Ta lại nhập 5 nhóm Di lại thành một nhóm rồi bổ sung các em còn lại vào các nhóm này thì sẽ được 5 lớp, mỗi lớp có đúng 10 em biết nhạc, 10 em biết bóng bàn và 10 em biết đá bóng. 3.3 Bài tập tự giải Bài toán 3.3.1. Điều mâu thuẫn ở đâu? Trong một tòa nhà chỉ có những cặp vợ chồng và những con nhỏ chưa lập gia đình. Ban điều tra dân số yêu cầu báo cáo về số người sống trong tòa nhà, đại diện là một anh thợ thích đùa báo cáo như sau: Sống trong tòa nhà bố mẹ nhiều hơn con cái. Mỗi con trai đều có 1 chị hay em gái. Số con trai nhiều hơn số con gái. Mỗi cặp vợ chồng đều có con. Người ta không thể chấp nhận được báo cáo đó dù là đùa vui vì trong đó xảy ra mâu thuẫn. Bạn hãy chỉ ra điều mâu thuẫn trong báo cáo trên. Hướng dẫn. Vì mỗi gia đình đều có con, mỗi con trai đều có một chị hoặc em gái. 46
  • 48. Suy ra số con gái ít nhất bằng số gia đình. Mặt khác số con trai nhiều hơn số con gái nên tổng số con nhiều hơn hai lần số gia đình, hay nhiều hơn số bố mẹ, điều này cho ta thấy mâu thuẫn trong báo cáo của anh thợ thích đùa ở câu đầu tiên:"bố mẹ nhiều hơn con cái" với các câu tiếp theo. Bài toán 3.3.2. Có xếp được không? Một bàn cờ đô - mi - nô có 64 ô vuông (bàn cờ vuông, mỗi cạnh 8 ô vuông). Các quân đô - mi - nô hình chữ nhật, có kích thước sao cho có thể chồng vừa khít lên hai ô vuông liền nhau. Như vậy có thể dễ dàng xếp được 32 quân cho vừa khít cả bàn cờ. Có thể xếp được 31 quân sao cho vừa khít 62 ô còn lại không? Nếu được thì xếp như thế nào? Nếu không, hãy chứng minh là không xếp được. Hướng dẫn. Trả lời: Không. Với bất kỳ cách xếp đặt nào, mỗi quân cờ sẽ phủ kín hai ô khác màu, vì vậy 30 quân cờ sẽ phủ kín 30 ô đen và 30 ô trắng. Còn lại 2 ô cùng một màu, do đó hai ô xa nhau sẽ hoặc đen hoặc trắng. Bài toán 3.3.3. Thứ tự xếp hàng. Có 5 người là A, B, C, D, E xếp thành hàng mua vé xem phim. Biết rằng A mua được vé sau E, trước B, C và D không đứng kề sau. D không đứng kề với E, A và C. Tìm thứ tự xếp hàng của 5 người đó. Hướng dẫn. Thứ tự sắp xếp: E, A, C, B, D. Vì D không đứng kề với E, A và C nên D đứng kề với B. Do đó D đứng đầu hàng hoặc cuối hàng, nhưng A mua vé trước D. Do vậy D đứng cuối hàng và B đứng thứ 4. Mà E đứng trước A, C và E không đứng kề nhau nên C đứng thứ 3. A đứng thứ 2 còn E đứng thứ nhất. Bài toán 3.3.4. Bao nhiêu đại biểu Trong một hội nghị có 500 đại biểu tham dự, mỗi đại biểu có thể sử dụng một trong ba thứ tiếng: Nga, Anh hoặc Pháp. Theo thống kê của ban tổ chức, có 60 đại biểu chỉ nói được một trong ba thứ tiếng, 180 đại biểu nói được hai thứ tiếng Anh và Pháp, 150 đại biểu nói được cả hai thứ tiếng Anh và Nga, 170 đại biểu nói được cả tiếng Nga và tiếng Pháp. Hỏi có bao nhiêu đại biểu nói được cả ba thứ tiếng trên? 47
  • 49. Hướng dẫn. Có 60 đại biểu nói được cả ba thứ tiếng. Bài toán 3.3.5. Ai đi xem xiếc? Gia đình Châu có 5 người: bố, mẹ, Châu cùng hai em Anh và Dũng. Sáng chủ nhật cả nhà thích đi xem xiếc, nhưng chỉ mua được hai vé. Mọi người trong gia đình đề xuất 5 ý kiến 1. Châu và Dũng đi. 2. Bố và mẹ đi. 3. Bố và Anh đi. 4. Mẹ và Châu đi. 5. Bố và Châu đi. Cuối cùng mọi người đồng ý với đề nghị của Dũng vì theo đề nghị đó mỗi đề nghị của bốn người còn lại đều được thỏa mãn một phần và bác bỏ một phần. Bạn hãy cho biết ai đi xem xiếc hôm đó? Hướng dẫn. Nếu chọn đề nghị thứ nhất thì đề nghị thứ 2 không thỏa mãn. Điều này trái với đề bài. Tương tự cũng không thể chọn đề nghị thứ hai, thứ ba và thứ tư. Nếu chọn đề nghị thứ 5 thì mỗi ý kiến được thỏa mãn 1 phần và bác bỏ 1 phần. Vậy bố và Châu đi xem xiếc. Bài toán 3.3.6. Người không chinh phục được cá sấu. Cho trước các mệnh đề: 1. Trẻ sơ sinh có tư duy logic. 2. Chúng ta không dám coi thường những người chinh phục được cá sấu. 3. Chúng ta coi thường những người chưa có tư duy logic. Từ các mệnh đề trên bạn hãy suy ra kết luận: Trẻ sơ sinh không chinh phục được cá sấu. Hướng dẫn. Giả sử "Trẻ sơ sinh chinh phục được cá sấu". Từ (2) suy ra "chúng ta không dám coi thường trẻ sơ sinh". Từ (3) suy ra "Trẻ sơ sinh có tư duy logic". Điều này mâu thuẫn với (1). Bài toán 3.3.7. Các giáo sư dạy môn gì? Ba giáo sư Kim, Quân, Hà dạy ba môn toán, lý, hóa. Trong ba mệnh đề: 1. Giáo sư Kim dạy môn hóa. 2. Giáo sư Quân không dạy môn hóa. 3. Giáo sư Hà không dạy môn lý. chỉ có một mệnh đề đúng. Bạn hãy xác định mỗi giáo sư dạy môn gì? 48
  • 50. Hướng dẫn. Giả sử mệnh đề (1) đúng suy ra (2) đúng. Điều này vô lý, vậy mệnh đề (1) là sai. Nếu (2) đúng ⇒ Giáo sư Hà dạy hóa và (3) đúng. Điều này vô lý, vậy mệnh đề (2) là sai. Vậy giáo sư Quân dạy hóa, giáo sư Hà dạy toán và giáo sư Kim dạy lý. 49
  • 51. Chương 4 Phương pháp bảng 4.1 Vài nét về phương pháp bảng Nhiều bài toán logic có thể giải bằng cách lập bảng mô tả mối quan hệ giữa các đối tượng được cho trong bài toán. Đối với một số bài toán logic trong đó xuất hiện 2 hay nhiều tệp và các cặp phần tử nói lên mối quan hệ giữa các tệp, người ta có thể thiết lập một hay nhiều bảng, để mô tả mối quan hệ giữa các tệp. Mỗi bảng này có hàng trên cùng ghi các phần tử của một tệp, còn cột tận cùng bên trái ghi các phần tử thuộc tập kia và các vị trí trong bảng ghi mã số quan hệ giữa các phần tử thuộc các tệp. Căn cứ vào các điều kiện đã cho trong bài toán gạch bỏ đi những cặp phần tử không thích hợp, từ đó đi đến lời giải của bài toán. Giải bài toán logic bằng phương pháp bảng đôi khi vấp phải bảng cần lập có chiều khá lớn hoặc phải kết hợp nhiều bảng mới đi đến kết quả. Sau đây là một vài ví dụ vận dụng phương pháp bảng để giải bài toán không mẫu mực. 4.2 Vận dụng phương pháp bảng để giải bài toán không mẫu mực Bài toán 4.2.1. Trong buổi học nữ công, ba bạn Cúc, Đào, Hồng làm ba bông hoa: cúc, đào, hồng. Bạn làm hoa hồng nói với bạn Cúc:"Thế là trong chúng ta không có ai làm loại hoa trùng với tên mình". Hãy xác định tên hoa mà mỗi bạn đã làm. Chứng minh. Bài toán này có hai tệp đối tượng. Tệp thứ nhất gồm các bạn làm hoa, 50
  • 52. tệp thứ 2 gồm các bông hoa được làm. Nó có thể giải bằng phương pháp bảng. 1) Lập bảng. Bảng cần lập gồm 4 hàng và 4 cột. Hàng đầu, từ cột thứ 2 ghi lần lượt tên các bông hoa được làm, còn trên cột tận cùng bên trái từ hàng hai ghi lần lượt các bạn tham gia làm hoa. 2) Điền mã số quan hệ vào các vị trí của bảng. a. Căn cứ vào giả thiết: Mỗi bạn đều không làm hoa trùng với tên mình, mà điền mã "không" vào các ô nằm trên đường chéo chính. Bảng 4.2.1 b. Căn cứ vào câu "Bạn làm hoa hồng nói với bạn Cúc" suy ra bạn Cúc không phải làm hoa hồng, mà ghi mã "không" vào ô nằm ở hàng Cúc, cột hồng. 3) Loại bỏ vị trí không thỏa mãn quan hệ để nhận được lời giải. Trong bảng trên cột cuối vị trí 1 và 3 bị gạch bỏ nên vị trí duy nhất còn lại là vị trí 2 phải thỏa mãn quan hệ giữa người làm hoa và hoa được làm. Do đó bạn Đào làm hoa hồng. Vì trên hàng 2 (Đào) đã có vị trí thỏa mãn quan hệ thì toàn bộ hàng này phải loại bỏ ra ngoài diện xét. Bởi vậy cột Cúc chỉ còn vị trí cuối cùng trong diện xét. Bởi vậy nó phải thỏa mãn quan hệ giữa người làm hoa và hoa được làm, nên bạn Hồng làm hoa Cúc. Từ đó suy ra người còn lại là bạn Cúc phải làm hoa đào. Vậy bạn Cúc làm hoa đào, bạn Đào làm hoa hồng và bạn Hồng làm hoa cúc. Bài toán 4.2.2. Trên bàn là 3 cuốn sách giáo khoa: Văn, Toán và Địa Lí được bọc 3 màu khác nhau: xanh, đỏ, vàng. Cho biết cuốn bọc bìa màu đỏ nằm giữa cuốn Văn và Địa Lí, cuốn Địa Lí và cuốn màu xanh mua 51
  • 53. cùng một ngày. Bạn hãy xác định mỗi cuốn sách bọc bìa màu gì? Chứng minh. Ta có bảng sau Bảng 4.2.2 Theo đề bài "Cuốn màu đỏ đặt giữa 2 cuốn Văn và Địa Lý". Vậy cuốn Văn và Địa Lý đều không bọc màu đỏ nên cuốn Toán sẽ bọc màu đỏ. Ta ghi số 0 vào ô 4 và 6, đánh dấu "x" vào ô 5 Mặt khác "cuốn Địa Lí và cuốn màu xanh mua cùng ngày". Điều đó có nghĩa là cuốn Địa Lí không bọc màu xanh. Ta ghi số 0 vào ô 3. Nhìn vào cột 4 ta thấy cuốn Địa Lí không bọc màu xanh, cũng không bọc màu đỏ. Vậy cuốn Địa Lí bọc màu vàng. Ta đánh dấu "x" vào ô số 9. Nhìn vào cột 2 ta và ô 9 ta thấy cuốn Văn không bọc màu đỏ cũng không bọc màu vàng. Vậy cuốn Văn bọc màu xanh, ta đánh dấu "x" vào ô 1. Kết luận: Cuốn Văn bọc màu xanh, cuốn Toán bọc màu đỏ, cuốn Địa Lí bọc màu vàng. Bài toán 4.2.3. Bốn bạn Xuân, Hạ, Thu, Đông nhận được điểm của bài kiểm tra cuối học kỳ. Bạn Lan cùng lớp muốn biết điểm của từng người. Khi Lan hỏi thì được các bạn trả lời úp mở như sau: Xuân nói: "Bạn Đông được 7, Hạ được 8, Thu được 9". Hạ nói: "Bạn Đông được 10, Thu được 8, Xuân được 9". Thu nói: "Cả 3 bạn đều được 7". Đông nói: "Cả 3 bạn đều được 8". Biết rằng không có 2 bạn nào được 2 bạn nói cùng đúng với số điểm của mình và mỗi câu trả lời ở trên chỉ nói đúng số điểm của một người mà thôi. Hãy tìm điểm của mỗi người. Chứng minh. Ta ghi điểm của từng người trong các câu trả lời vào bảng 4.2.3. 52
  • 54. Căn cứ vào điều kiện: Không có bạn nào được hai bạn nói đúng điểm của mình, ta suy ra Hạ không được điểm 8. Thu không được điểm 8. Đông không được điểm 7. Bảng 4.2.3 Căn cứ vào điều kiện: Mỗi câu trả lời chỉ đúng với số điểm của một người, ta suy ra: Xuân được 8, Thu được 9, Đông được 10, Hạ được 7. Bài toán 4.2.4. Trong trại hè thiếu nhi quốc tế, một tổ gồm ba bạn thiếu niên: Một bạn người Anh, một bạn người Pháp, một bạn người Nga. Mỗi người trong số ba người này đều đang học một trong ba ngoại ngữ: Anh, Pháp, Nga. Biết rằng bạn học ngoại ngữ Anh lớn hơn bạn người Pháp 1 tuổi. Hãy xác định xem bạn nước nào học ngoại ngữ gì? Chứng minh. Trong bài toán này có hai nhóm đối tượng: Nhóm các bạn thiếu niên: Người Anh, người Pháp, người Nga. Nhóm các ngoại ngữ: Tiếng Anh, tiếng Pháp, tiếng Nga. Ta lập một bảng có hàng trên là quốc tịch các bạn thiếu niên và cột bên trái là các ngoại ngữ. Theo đề bài: "Bạn học ngoại ngữ Anh lớn hơn người Pháp một tuổi", ta suy ra: Bạn học người Pháp không học tiếng Anh. Do đó ta điền O vào ô (tiếng Anh, Pháp). Rõ ràng tiếng Pháp không phải ngoại ngữ của bạn người Pháp, tương tự với bạn người Anh và người Nga nên ta điền O vào ô (tiếng Pháp, Pháp), (tiếng Anh, Anh) và ô (tiếng Nga, Nga). Tiếp đến, nhìn vào hàng tiếng Anh, ta thấy hai ô O, do đó ô còn lại phải là có nên ta đánh dấu "x" vào ô (tiếng Anh, Nga). 53
  • 55. Bảng 4.2.4 Lại thấy cột Pháp đã có 2 ô O, nên ô còn lại phải là có nên ta đánh dấu "x" vào ô (tiếng Nga, Pháp). Suy ra ô (tiếng Pháp, Anh) là có nên ta đánh dấu "x". Vậy bạn người Pháp học tiếng Nga, bạn người Nga học tiếng Anh, bạn người Anh học tiếng Pháp. Bài toán 4.2.5. Nhiều nhất bao nhiêu người dự đoán đúng? Khi đội tuyển của trường Lương Ngọc Quyến đi thi học sinh giỏi văn và toán toàn quốc, sáu bạn ở nhà đã dự đoán: An: Trường ta sẽ đạt giải vô địch môn văn. Hải: Trường ta sẽ không đạt giải vô địch môn toán. Lan: Trường ta đạt giải vô địch môn toán nhưng không đạt giải vô địch môn văn. Phan: Đạt giải vô địch văn nhưng không đạt giải vô địch toán. Cường: Đạt giải vô dịch văn và đạt giải vô địch toán. Thịnh: Đạt giải vô địch văn hoặc đạt giải vô địch toán nhưng không đạt cùng một lúc cả hai giải vô địch. Bạn hãy cho biết nhiều nhất bao nhiêu bạn dự đoán đúng và bao nhiêu bạn dự đoán sai? Chứng minh. Giả sử bạn Lan dự đoán đúng, thế thì An và Hải đều dự đoán sai. Nếu Lan dự đoán sai và một trong hai bạn An hoặc Hải dự đoán sai thì cũng có hai bạn dự đoán sai. Còn nếu cả hai bạn dự đoán đúng, thì Cường dự đoán sai. Vậy trong mọi trường hợp vẫn có 2 bạn dự đoán sai hay nói một cách khác, số bạn dự đoán đúng không vượt qua 4. Mặt khác, nếu An và Hải dự đoán đúng thì cả Phan và Thịnh cũng dự đoán đúng. Vì vậy số người dự đoán đúng nhiều nhất không nhỏ hơn 54
  • 56. 4. Từ đó suy ra nhiều nhất có 4 bạn dự đoán đúng. Gọi dự đoán của các bạn An, Hải, Lan, Phan, Cường, Thịnh theo thứ tự là các mệnh đề A, H, L, P, C, T. Mệnh đề đúng ký hiệu là 1, mệnh đề sai ký hiệu là 0. Theo bài ra ta có bảng sau đây: Bảng 4.2.5 Nhìn vào bảng trên ta cũng thấy số mệnh đề đúng nhiều nhất bằng 4 và số mệnh đề sai nhiều nhất cũng bằng 4. Bài toán 4.2.6. Có ba cậu bé từ Quy Nhơn, Đà Nẵng và Huế tới dự trại hè. Chúng bằng tuổi nhau và chúng cùng thích thể thao. Chỉ có Châu và cậu bé đến từ Quy Nhơn chơi tennis, Tốc và cậu bé tới từ Đà Nẵng chơi bóng, Dung chơi cờ tướng và lớn hơn cậu bé từ Đà Nẵng. Các cậu bé chơi tennis không chơi cờ. Cậu bé chơi cờ lớn nhất. Hỏi cậu bé nào đến từ thành phố nào? Và thích môn thể thao nào? Ai lớn hơn ai? Chứng minh. Trước tiên ta xác định xem ai là người thành phố nào? Thích môn thể thao nào? Ta lập bảng hai chiều: Chiều thẳng đứng ghi tên môn thể thao và thành phố, chiều ngang ghi tên các cậu bé và thành phố. Bảng này chứa tất cả các khả năng có thể đề kết hợp tên người và tên môn thể thao ưa thích, tên người với tên thành phố, tức là những tích đề các của hai tập hợp: A × X và A × Y với A = {Châu, Tốc, Dung} X = {Tennis, Bóng bàn, Cờ} Y = {Qui Nhơn, Đà Nẵng, Huế} Từ giả thiết ta có: Do Châu và cậu bé từ Quy Nhơn chơi tennis, suy ra Châu không ở Quy 55
  • 57. Nhơn, vì vậy ô (Quy Nhơn, Châu) ghi O. Do Tốc và cậu bé từ Đà Nẵng chơi bóng đá, suy ra Tốc không ở Đà Nẵng, vì vậy ô (Đà Nẵng, Tốc) ghi O. Suy ra Dung không ở Q vì cậu bé từ Q chơi tennis, mà chơi tennis thì không chơi cờ, do đó Dung ở H. Vì vậy ô (H, Dung) đánh dấu "x". Suy ra các ô còn lại ở dòng Huế, các ô (Quy Nhơn, Dung) và (Đà Nẵng, Dung) ghi O. Dòng Quy Nhơn còn lại duy nhất ô (Quy Nhơn, Tốc) trống, nên ô này ghi dấu "x". Dòng Đà Nẵng còn lại duy nhất ô (Đà Nẵng, Châu) trống nên ô này ghi dấu "x". Cũng từ giả thiết ta có: ô (Tennis, Châu), (Tennis, Quy Nhơn) ghi dấu "x"; ô (Bóng bàn, Tốc), (Bóng bàn, Đà Nẵng) ghi dấu "x"; ô (Cờ tướng, Dung) ghi dấu "x". Mặt khác cậu bé chơi cờ lớn nhất, suy ra Dung lớn nhất. Dung lại lớn hơn cậu bé từ Đà Nẵng, do đó Dung lớn hơn Châu. Suy ra Tốc bé nhất. Vậy: Châu chơi tennis và đến từ Đà Nẵng. Tốc chơi bóng và đến từ Quy Nhơn, Dung chơi cờ và đến từ Huế. Bài toán 4.2.7. Trong một buồng trên toa tàu có 6 hành khách A, B, V, G, D, E từ 6 thành phố M, L, K, T, X, O. Dọc đường họ phát hiện ra: 1. A và người từ M là bác sỹ, D và người từ L là giáo viên, V và người từ T là kỹ sư. 2. B, E và một người từ K là cựu chiến binh, còn người từ T chưa nhập ngũ bao giờ. 3. Người từ X già hơn A, người từ O già hơn V, còn E là người trẻ nhất hội. 4. B và người từ M đi K, còn V và người từ X đi O. 56